Jump to content

Wikipedia:Reference desk/Science: Difference between revisions

From Wikipedia, the free encyclopedia
Content deleted Content added
SineBot (talk | contribs)
m Signing comment by 116.71.54.217 - "→‎Azimuth sweep angle?: new section"
Line 720: Line 720:
== Azimuth sweep angle? ==
== Azimuth sweep angle? ==


In the game Call of Duty 4, in one mission we control the guns on a C-130 Spectre Gunship. A guy on the plane says a couple of times: "Recalibrate azimuth sweep angle, adjust elevation scan." What does it mean? And yes I'm the guy who asked the Coriolis effect question <ref>http://en.wikipedia.org/wiki/Wikipedia:Reference_desk/Science#Coriolis_effect_and_sniping</ref>. Just a big fan of COD4. And thanks for your answers. <span style="font-size: smaller;" class="autosigned">—Preceding [[Wikipedia:Signatures|unsigned]] comment added by [[Special:Contributions/116.71.54.217|116.71.54.217]] ([[User talk:116.71.54.217|talk]]) 08:52, 20 April 2009 (UTC)</span><!-- Template:UnsignedIP --> <!--Autosigned by SineBot-->
In the game Call of Duty 4, in one mission we control the guns on a C-130 Spectre Gunship. A guy on the plane says a couple of times: "Recalibrate azimuth sweep angle, adjust elevation scan." What does it mean? And yes I'm the guy who asked the Coriolis effect and sniping question . Just a big fan of COD4. And thanks for your answers. <span style="font-size: smaller;" class="autosigned">—Preceding [[Wikipedia:Signatures|unsigned]] comment added by [[Special:Contributions/116.71.54.217|116.71.54.217]] ([[User talk:116.71.54.217|talk]]) 08:52, 20 April 2009 (UTC)</span><!-- Template:UnsignedIP --> <!--Autosigned by SineBot-->

Revision as of 08:53, 20 April 2009

Welcome to the science section
of the Wikipedia reference desk.
Select a section:
Want a faster answer?

Main page: Help searching Wikipedia

   

How can I get my question answered?

  • Select the section of the desk that best fits the general topic of your question (see the navigation column to the right).
  • Post your question to only one section, providing a short header that gives the topic of your question.
  • Type '~~~~' (that is, four tilde characters) at the end – this signs and dates your contribution so we know who wrote what and when.
  • Don't post personal contact information – it will be removed. Any answers will be provided here.
  • Please be as specific as possible, and include all relevant context – the usefulness of answers may depend on the context.
  • Note:
    • We don't answer (and may remove) questions that require medical diagnosis or legal advice.
    • We don't answer requests for opinions, predictions or debate.
    • We don't do your homework for you, though we'll help you past the stuck point.
    • We don't conduct original research or provide a free source of ideas, but we'll help you find information you need.



How do I answer a question?

Main page: Wikipedia:Reference desk/Guidelines

  • The best answers address the question directly, and back up facts with wikilinks and links to sources. Do not edit others' comments and do not give any medical or legal advice.
See also:



April 14

DNA

Can one do a DNA test from vomit? —Preceding unsigned comment added by 12.216.168.198 (talk) 00:21, 14 April 2009 (UTC)[reply]

Sure, though not necessarily all instances of vomit. The vomit itself -- that is to say, that charming combination of stomach acid, half-digested food and whatnot -- probably doesn't contain any, but it's not uncommon for the vomit to have bits of stomach lining in it, for example. Likewise, people generally do a lot of spitting in the process. (That said, Nigel Tufnel famously opined that you can't dust for vomit, but I grant that he may not be the greatest authority on this subject. Or any subject.) -- Captain Disdain (talk) 01:57, 14 April 2009 (UTC)[reply]
I bet he knows a thing or two about the glutæus maximus, particularly large ones. ;) Rockpocket 02:15, 14 April 2009 (UTC)[reply]
Vomit may represent the worst sort of specimen for DNA testing, because a large proportion of the DNA in it may represent a source (food) other than the person you mean to be testing. Ideally, of course, if you can ensure your testing only detects human DNA then you'll only be confused if you're testing a cannibal. --Scray (talk) 02:35, 14 April 2009 (UTC)[reply]

stars outside the boundaries of their constellation

is there a name for a star that is named as being a member of one constellation, but actually exists inside the boundaries of another? does such a thing even exist? i had a dream that a star like that was called a "tapuo" or something, so now i'm curious if it actually exists. 99.245.16.164 (talk) 00:35, 14 April 2009 (UTC)[reply]

I have a copy of A Field Guide to the Stars and Planets, one of the books mentioned in the constellation article. I'm pretty sure that its maps don't show any such thing as a star named as part of a different constellation. --Anonymous, 00:53 UTC, April 14, 2009.
The star Sirrah might be considered an example of what OP refers to: It forms the north-eastern corner of the big square that visually defines the constellation Pegasus (constellation), and historically, it was called δ Pegasi. According to the modern definition of the constellations it belongs to Andromeda, and is even known as α Andromedae. Another example might be β Tauri/γ Aurigae. --Wrongfilter (talk) 07:54, 14 April 2009 (UTC)[reply]
Bayer designation (whose language I tweaked yesterday) mentions some examples. —Tamfang (talk) 06:34, 15 April 2009 (UTC)[reply]

Are diamonds ionic compounds?

Diamonds have some properties of ionic compunds (they are hard, have high melting and boiling points and can conduct electricity) but other properties (such as diamonds not being able to dissolve in water and not being brittle) are not properties of ionic compounds. —Preceding unsigned comment added by 174.6.144.211 (talk) 01:52, 14 April 2009 (UTC)[reply]

No, the carbon-carbon bonds inside a diamond are covalent bonds. Dauto (talk) 02:09, 14 April 2009 (UTC)[reply]

But if diamonds are covalent compoounds then why are they hard, have high melting and boiling points are able to conduct electricity? —Preceding unsigned comment added by 174.6.144.211 (talk) 02:18, 14 April 2009 (UTC)[reply]

Usually covalent compounds will have covalent bonds (which are very strong) within a molecule but those molecules will bind to each other by week Van der Waals atraction. I diamond has strong covalent bonds all over the whole crystaline structure. In simple terms, it is one single macroscopic molecule. Dauto (talk) 03:11, 14 April 2009 (UTC)[reply]
Look up diamond. most diamonds are insulators and they more burn rather than melt. The reason they are so hard is how well they electronic/bonding structure matches their packing crystal lattice. Diamonds also have a lot in common with ionic materials. Diamond aren't so much a molecule or polymer as a material made of repeating unit cells like a ionic solids.--OMCV (talk) 02:24, 14 April 2009 (UTC)[reply]
I'm stunned to note that we don't have an article on network solids. I'm too swamped to write one at the moment, but if someone wants to dive in, please do! TenOfAllTrades(talk) 12:34, 14 April 2009 (UTC)[reply]

More generally, lots of things you learn in school are general guidelines or observed trends, not the actual reasons. "Ionic has property X, non-ionic has property not-X" is only true if being ionic is the exact and only cause of property X. But that's not true for this case: the actual cause of "high-melting" isn't "ionic", but rather it's related to how the molecules are held together. Ionic molecules are merely a common type that are held together in a way to give high-mp. For that reason, "ionic gives high mp" doesn't mean you can say "therefore non-ionic will not be high mp": being ionic is not the only type of molecule that is held together in this certain way. Same goes for any property and any characteristic: need to see how/why the trend works, not use the observed trend as the actual cause and black'n'white separation (correlation vs causation). DMacks (talk) 14:18, 14 April 2009 (UTC)[reply]

To expand a little bit, the binary "covalent vs. ionic compounds" thing they teach you in basic high school chemistry is a bit oversimplified, and it does not apply to many substances including diamonds. Reorganize your thinking this way. Compounds either exist as a collection of discrete molecules or they exist as a network of roughly equally bonded particles.
Molecules are the "special case" for us to consider. In the case of molecular substances, you have a small number of atoms which are strongly tied to each other, making a tiny packet which itself is electrically neutral; meaning it does NOT much attracted to other substances, and only subject to smallish forces like dispersion forces and dipole interactions. Therefore molecular substances (calling them "covalent" compounds is misleading, see explanation below) will tend to have low melting and boiling points, be mostly gasseous or low boiling liquids, etc. etc. The bonding inside of the molecule is covalent bonding, but the bonding between the molecules are relatively weak, and THAT is what makes molecular substances behave like they do.
All other compounds consist of billions of atoms all bonded to each other in a highly stable crystal. The strength of these bonds is comparible to the strength of the internal bonds in a molecule, thus ripping apart a salt crystal or a diamond is roughly on the order of physically pulling the hydrogen atoms off of a water molecule; it requires huge forces, which is why "non-molecular substances" are universally strong, hard, and have high boiling and melting points. Now, these non-molecular substances can be further classified according to the type of bonding holding their atoms together, but these distinctions are not as large as one might imagine. The type of bonding in non-molecular substances falls into three types: covalent bonding, ionic bonding, and metallic bonding.
For covalent and ionic bonding, look at the crappy MS Paint diagrams I uploaded. The both consist of the force of attraction between a positive atomic center and an negatively charged electron cloud. It's only the location of that cloud (either between the atoms or around one of them) that determines the difference between the two. Since BOTH consist of the same maginitude of electrostatic attractions, BOTH are going to be on the same order of strength. Thus so-called "network covalent" solids, like diamond and zinc blende are about as strong as so-called "ionic" solids like sodium chloride and potassium nitrate.
For metallic bonding, the electron cloud, instead of being localized either between or around some of the atoms, is instead shared roughly equally between all atoms, creating a so-called "sea of electrons" which explains metals electrical conductive properties. The material is hard because ALL of the electrons in bulk are holding the whole mess together, but the individual electrons are not tied to any individual nucleii, so they are free to move around easily given a little "push".
I hope that clears things up a bit for you. So, you see that diamond is infact a covalently bonded substance, but it is not a molecular substance, which is why it does not behave like a molecular substance. --Jayron32.talk.contribs 01:24, 15 April 2009 (UTC)[reply]

Atropine as anaesthesia pre-med

Atropine is historically used as an anaesthetic, and apparently is used as a premedication for general anaesthesia today, I can see why you would want to have a lower heart rate, bronchodilation and the like, but as atropine is a CNS stimulant, my question is, why would you use it as an pre-anaesthesia. I know hyoscine would be a better choice. MedicRoo (talk) 11:14, 14 April 2009 (UTC)[reply]

You may find your answer to why hyoscine (scopolamine) has limited usage compared to atropine at Muscarinic_antagonist#Effects, which states that scopolamine crosses the blood-brain barrier, whereas atropine does not. Therefore, scopolamine can cause increased CNS effects such as amnesia, which can in some cases be beneficial, but not all. Also, atropine given intramuscularly initially results in bradycardia. —Cyclonenim | Chat  11:43, 14 April 2009 (UTC)[reply]
Slightly irrelevant to the OP's question, but are you sure atropine doesn't cross the BBB? In Atropine#Side-effects and overdose it states "hallucinations" and presumably the typical delerium-like effects of anticholinergics. Looking at the chemical structure, it seems fairly non-polar. --Mark PEA (talk) 19:54, 14 April 2009 (UTC)[reply]
I understand most of the effects and differences, more I'm not clear on why atropine would be used for anaesthesia when it causes CNS excitation instead of depression.MedicRoo (talk) 13:15, 14 April 2009 (UTC)[reply]
I'm not too sure, all I know is that there is a weak agonist effect in low doses of atropine, which lowers the heart rate. Presumably, this agonist effect spreads throughout the entire parasympathetic nervous system, not just the heart. —Cyclonenim | Chat  13:42, 14 April 2009 (UTC)[reply]

The main reasons for using anticholinergic drugs as pre-medication are:-

  1. Drying of secretions
  2. Prevent bradycardia
  3. Anti-emetic
  4. Prevent laryngospasm

Other possible benefits are relatively unimportant. The sedative effect of hyoscine is not usually regarded as beneficial when giving general anaesthesia. Indeed the sedative effect of hyoscine may be tricky to predict, especially in the elderly. Other anaesthetic drugs give a more reliable response. Axl ¤ [Talk] 17:34, 14 April 2009 (UTC)[reply]

Bugger, somewhere along the line me and atropine went seperate ways and I thought it was right behind me. Thanks for the help.MedicRoo (talk) 18:05, 14 April 2009 (UTC)[reply]
I know that atropine is frequently used when doing surgery on rats, because it dries them out and makes it less likely that they will choke on their saliva. It isn't an anesthetic as far as I know. Looie496 (talk) 01:35, 15 April 2009 (UTC)[reply]

Separating Water(H2O) and Hydrogen Peroxide(H2O2)

The hydrogen Peroxide(H2O2) you buy at pharmacies are so low concentrated and are dissolved in water(H2O), and I want more pure hydrogen Peroxide, but don't want to use so much money to buy the expensive high concentration hydrogen Peroxide(H2O2) from large companies.Can I remove the water by evaporation(H2O2's boiling point is 150.2 °C but water's boiling point is 100 °C)? Please tell me other methods too!The Successor of Physics 15:19, 14 April 2009 (UTC)[reply]

Have you read our excellent article on hydrogen peroxide ? Especially the part that says "Above roughly 70% concentrations, hydrogen peroxide can give off vapor that can detonate above 70 °C (158 °F) at normal atmospheric pressure. This can then cause a boiling liquid expanding vapor explosion (BLEVE) of the remaining liquid. Distillation of hydrogen peroxide at normal pressures is thus highly dangerous." Maybe there's a good reason it's only generally available in low concentrations .... Gandalf61 (talk) 16:42, 14 April 2009 (UTC)[reply]


Makes pretty good rocket fuel though. The higher cost of high purity H2O2 would be because it is difficult to make safely. 65.121.141.34 (talk) 18:10, 14 April 2009 (UTC)[reply]
The main reason it's only available at low concentrations is because high concentrations are very dangerous. It looks exactly like water, but if you take a swig of highly concentrated H2O2 and swallow it before you know what you're doing, say goodbye. Looie496 (talk) 01:38, 15 April 2009 (UTC)[reply]
I know what Looie496 and 65.121.141.34 said but forgot what Gandalf61 said! Well if I can't seperate them by evaporation, then how about cooling(I'm not even sure is there such a separation method)(H2O2's melting point is -0.41 °C but water's melting point is 0 °C)?The Successor of Physics 13:05, 15 April 2009 (UTC)[reply]

H2O2 has a density about 40% greater then water, perhaps a centrifuge could separate them? 65.121.141.34 (talk) 15:39, 15 April 2009 (UTC)[reply]

You can't use evaporation to concentrate hydrogen peroxide above the point where it becomes an azeotrope. Does anyone here know what that point is? — DanielLC 16:31, 15 April 2009 (UTC)[reply]
I know this is a very old thread, but in response to DanielLC, H2O2 and water do not form an azeotrope. Complete (to all intents and purposes) separation is possible with a sufficiently tall distillation column. Preferable vacuum distillation so you don't blow everything up.

4th Dimension...again

http://www.math.union.edu/~dpvc/math/4D/projections/welcome.html

According to this link, what would we look like to a entity that can see in 4D? --Reticuli88 (talk) 15:36, 14 April 2009 (UTC)[reply]

Well, we would look like ... a 3D entity. Equendil Talk 17:08, 14 April 2009 (UTC)[reply]

Ok. What would we look like as a 4D entity? --Reticuli88 (talk) 18:45, 14 April 2009 (UTC)[reply]

If we were (somehow) a 3D object in a 4D world, we'd look kinda "flat" - much like a drawing on a 2D sheet of paper looks to us in the 3D world. But this is basically a silly question. If the universe had 4 spatial dimensions (and not "curled up" dimensions as string theory supposes) then we would be 4D entities with perceptual systems to match. With 4D brains, we'd have evolved to be entirely different creatures than we really are...in all likelyhood, there is some deep, dark, physics-ish reason why things like people, DNA, planets, stars and galaxies might simply not be able to exist in a 4D world. Certainly, such things are impossible in a 2D world. So this "what would it be like if..." trail our OP is on is entirely fruitless. All we can say (and even then, not entirely uncontroversially) is what a 4D world might look like to us...as we are right now with our 2D eyes...assuming that the phenomenon of light is possible in a 4D universe, assuming we don't explode into fundamental particles the moment we pop into existence in the 4D world, etc, etc. SteveBaker (talk) 19:41, 14 April 2009 (UTC)[reply]


The reason I brought this up was because of the link below:

http://sprott.physics.wisc.edu/pickover/fourth.html

The author describes what a 4D entity would appear to us: "....resemble flesh-colored balloons constantly changing in size." Also, as scene in the movie Contact, Jodie Foster's character experiences a sort of 4D experience, I think. I mean the scene when she exclaims "They're alive!" the first time; the scene where it appears she splits in two. Does anyone know this reference? I'm just trying to get some handle what 4D would appear to us lowly humans. --Reticuli88 (talk) 20:40, 14 April 2009 (UTC)[reply]

You might like to read Flatland; it's a good way to start thinking about these things. It uses a common approach to these things; what would 3D objects look like to 2D entities? Then extrapolate. For example, (and this is probably something like what the author of that link is getting at) a 2D entity experiencing a 3D entity would itself really be a 3D entity which only experienced 2 dimensions (like an ant that never looks up or down). If a sphere passed through the 2D entity's plane, the 2D entity would experience it as a circle which got bigger and bigger, then smaller and smaller. Equally, a 4D hypersphere passing through our 3D space would appear as a 3D sphere that got bigger and bigger, then smaller and smaller. More complicated shapes would change in more complicated ways.
The entity would only appear as flesh-coloured balloons if they were flesh-coloured (whose flesh?) and bulbous. 217.43.141.59 (talk) 21:52, 14 April 2009 (UTC)[reply]
I don't understand why people keep recommending Flatland - I can't believe that anyone who has actually read the book would ever wish it on anyone else! It's a simply awful book! It's turgid, sexist, racist, class-ist and everything-else-ist. It's really painful to read. It tells you nothing meaningful about different numbers of dimensions and the author is clearly quite confused about his explanations of the 2D world (his description of houses is particularly ill-thought-out. So - forget that piece of junk and read Planiverse by A.K. Dewdney - this is an entirely self-consistent world in two dimensions - it has plot, it has amazing diagrams of 2D gadgets (if you have any interest in mechanical gadgets, you'll spend ages staring at the wonderful 2D steam engine - and thinking SURELY there is a way to have 2D wheels that work usefully (there isn't)...the 2D house and 2D space-station are also pretty interesting) - and it has a comprehensive appendix that explains in detail the problems with there being an actual 2D universe. It's written by a mathematician so you can be reasonably sure that it's properly thought through. It remains one of very few works of fiction that survives on my "top 20 books" bookshelf! Read it! NOW!!!! SteveBaker (talk) 22:42, 14 April 2009 (UTC)[reply]
Brilliant! Here's a link to the steam engine diagram for anyone who can't find it themselves. SpinningSpark 00:14, 15 April 2009 (UTC) although lighting a fire strikes me as a bad idea on a world that only exists on paper SpinningSpark 00:24, 15 April 2009 (UTC)[reply]
One aspect of the steam engine diagram that may not be evident without reading the book is the question of how 2D beings could assemble such a machine? When you look at it at first sight, it seems impossible. However, what you may not appreciate is that the parts that look like the artist shaded them with a bunch of thin parallel lines - actually represent a glued laminated material that the people of the 2D world have developed to solve this exact problem. By building up the steam engine from layers of thin material, glued together, they are able to fabricate the machine perfectly well. When you understand THAT - you've gotta go back and stare at the diagram for ANOTHER 10 minutes to convince yourself that's really true. I just flipped through the book again - the stuff about the 2D computer is really quite clever...how do you make a computer when wires can't cross each other without shorting out? Also how does a 2D being eat and excrete without falling in half? All of these things are quite nicely explained. Like I said - it's an amazingly well put-together book. SteveBaker (talk) 22:40, 15 April 2009 (UTC)[reply]


Flatland is a satire. You're not supposed to like the sexist and classist society it describes. As a satire I think it's pretty clever. But I don't understand either why people keep recommending it as a book about life in two dimensions. Abbott obviously didn't much care about that side of the story. There's nothing in the book about two-dimensional physics or biology, and what little it says about geometry could fit on one page. For that you have to slog through 150 pages of the narrator complaining about the baseness of women and the isosceles class. Think of it as part five of Gulliver's Travels, though, and it's a good read. -- BenRG (talk) 12:55, 15 April 2009 (UTC)[reply]
Yeah - I've heard it described as a satire - but it doesn't seem in any way to be poking fun at these ideas. If you told me it was a stinging diatribe about the fickle nature of women and the uselessness of the working classes - I'd have been more inclined to believe it. SteveBaker (talk) 17:57, 15 April 2009 (UTC)[reply]
Well, consider that as the dimensionality decreases, so does one's respect for the inhabitants. The King of Lineland isn't much when compared to A Square, and the sole inhabitant of Pointland is a horrid egomaniac. By analogy then, the implication is that the figures in Flatland are not only physically, but morally, inferior to we Spacelanders. And, BTW, the Sphere, itself, is a bit on the pompous side. (Perhaps the Hyperspacers have higher ethics.) B00P (talk) 23:10, 15 April 2009 (UTC)[reply]

In this 4D world our 3D world would appear like the stiff flat cross section showing at 1:48 to 2:04. Cuddlyable3 (talk) 00:08, 17 April 2009 (UTC)[reply]

orion

A friend of mine told me that you can see the constellation Orion in Australia, but that it looks upsidedown then what is normal in the northern hemisphere. Was he just joking with me, or is there truth to this? 65.121.141.34 (talk) 16:12, 14 April 2009 (UTC)[reply]

He is not joking. Dauto (talk) 16:23, 14 April 2009 (UTC)[reply]
You can currently catch Orion for about three hours after sunset, looking westwards, tilted about 90° clockwise compared to the northern hemisphere (with Sirius above) after which it disappears below the horizon to reappear later during the day lost in the sun glare (and "upside down" for a while). Equendil Talk 17:03, 14 April 2009 (UTC)[reply]
It's true - from the point of view of someone looking at Earth from the outside, someone in the Southern hemisphere is "standing on their head" relative to someone in the Northern hemisphere. That means that, from their perspective, anything outside the Earth is upside-down compared to what someone in the North sees. --Tango (talk) 18:24, 14 April 2009 (UTC)[reply]
Obviously, all of the constellations, the sun, the moon and the planets are "upside down" - but not many of the constellations are visible from both the far northern and southern hemispheres...but I guess Orion is one of them. The MUCH more noticeable thing for me is that the moon is "upside down". A new moon in the northern hemisphere looks like ')' but in the southern hemisphere it looks like '('. On the equator, it is an upward curving arc. I was born and lived most of my life in the UK - which is pretty far north. A waning moon looks like '(' - and I'd read many times that some cultures believed the waning moon was a boat that carried the gods around (or some such twaddle). I could never understand why they thought that...until I moved to Texas where the waning moon is rolled over at 45 degrees...and on the equator...it looks just like a boat (well, a canoe, perhaps). Even now - after living 16 years in Texas - the moon always looks "wrong" to me. SteveBaker (talk) 19:22, 14 April 2009 (UTC)[reply]
I recall looking at the full moon in New Zealand and trying to figure what was wrong with it. Not sure why but I was totally unable to recognize the "patterns" on the surface at all (I wasn't aware that it was simply upside-down at the time), as if it had been randomized and now I was forced to find another character or face to associate it with. 124.154.253.25 (talk) 03:43, 15 April 2009 (UTC)[reply]
A new moon looks like a ' ' Are you thinking of a waxing crescent? — DanielLC 16:23, 15 April 2009 (UTC)[reply]
Orion's head and feet are at 10 degrees north and 10 degrees south respectively, so he's partly visible everywhere on Earth and fully visible (not counting some dim stars) within 80 degrees of the equator. —Tamfang (talk) 06:32, 15 April 2009 (UTC)[reply]

Note that it's not as simple as "right-side-up or upside-down". Even at the same location, the same constellation will be seen in different orientations at different times of day when it is in different parts of the sky. Similarly, it is not true that "down" is exactly the opposite direction for people in different hemispheres unless they happen to be in antipodal locations. I'm in Toronto; compared to someone in Buenos Aires I'm not upside-down, I'm sideways.

For a simple example that doesn't require you to wait for nightfall, look at the moon at different times on the same day. The center of the lighted side always points toward the sun -- therefore it points above the horizon when the sun is up, but below the horizon when the sun is down. But the same features on the moon's face are visible, so you can see it's a different way up with respect to you. Well, for someone in a different part of the Earth it's different again. --Anonymous, expanded 04:55 UTC, April 15, 2009.

Muscle pain associated with infection

What is the cause of muscle pain associated with the flu or other infections? I looked at myalgia - no help there. -71.239.110.144 (talk) 18:38, 14 April 2009 (UTC)[reply]

How about the Causes section of Fibromyalgia? If that isn't any help, asking a doctor should be your next step. Livewireo (talk) 18:54, 14 April 2009 (UTC)[reply]
The OP is thinking of the pain which is shown as part of sickness behavior. I have little knowledge of the subject, but a quick read on the fever article states "A trigger of the fever, called a pyrogen, causes a release of prostaglandin E2 (PGE2)". A quote from Eicosanoid#Role in inflammation states: "Pain — The cytokines increase COX-2 activity. This elevates levels of PGE2, sensitizing pain neurons". Ibuprofen is a COX-2 inhibitor and thus should treat this pain. --Mark PEA (talk) 22:40, 14 April 2009 (UTC)[reply]

Dilution

Hello.

A stock solution of HCl(aq) has a density of 1.19 g/mL and a percentage purity of 39.1%. A lab requires 7.25 L of 0.500 mol/L solution. What volume of stock solution do I need before dilution?

I divided the amount of hydrochloric acid needed (3.625 mol) by [HCl(aq)]39.1% (12.76165661 mol/L), reaching the conclusion of 284 mL. The answer key says 303 mL. This is not homework counting for marks. Otherwise, my teacher would not have given me the answers. Did I do something wrong? Thanks in advance. --Mayfare (talk) 20:58, 14 April 2009 (UTC)[reply]

I'm getting 284 mL as well. Perhaps we're both making the same obvious error (anyone?), or perhaps there's a typo in the book. TenOfAllTrades(talk) 21:26, 14 April 2009 (UTC)[reply]
If there is an obvious mistake out there, you can count on me to make it, I get the same answer as Mayfare also (well 284.7 to be exact). The book answer requires a molecular weight of 38.8. The value I am using is 36.45. SpinningSpark 22:18, 14 April 2009 (UTC)[reply]
One thing to double check is what is meant by "39.1% purity" - is it w/w, w/v, v/v, v/w, mol/mol (mole percent) or something else? I would usually expect it to be w/w in this context, but you never can tell. I tried to run the calculation several other ways but none gave me 303 mL, though. -- 128.104.112.117 (talk) 23:14, 14 April 2009 (UTC)[reply]

burning of stone

can stones get fire as wood (at any temprature)i.e as wood cause to increase fire ,can stones do same thing —Preceding unsigned comment added by True path finder (talkcontribs) 22:01, 14 April 2009 (UTC)[reply]

No. In order to catch fire, the object has to be able to combine with oxygen and give off energy. Wood has energy to give off - and the carbon in the wood can combine with the oxygen in the air to make carbon dioxide. Rocks have no spare energy to give away - the chemicals they contain don't easily combine with oxygen - so with rare exceptions (Coal, for example), rocks don't catch fire. SteveBaker (talk) 22:27, 14 April 2009 (UTC)[reply]
Does something that burns (rapidly oxidizes) always have to be exothermic ? I've noticed that aluminum foil seems to "burn" in a hot enough flame, yet doesn't appear to give off energy, since the flame is not self-sustaining (if you try this experiment, please do it outside with a torch, as aluminum fumes can be toxic). As for the rocks, the two alternatives would seem to be melting and burning. So, do all other rocks melt at high temps (or perhaps some sublimate) ? StuRat (talk) 15:25, 15 April 2009 (UTC)[reply]
Aluminum can give off huge amounts of heat when it burns. See thermite. Perhaps it doesn't have enough oxygen when you do it that way? — DanielLC 16:14, 15 April 2009 (UTC)[reply]
I guess it doesn't have to be exothermic - but for the fire to be self-sustaining, it would. SteveBaker (talk) 17:51, 15 April 2009 (UTC)[reply]

Effects of fasting

what r the permanet, bad r good effects of Muslim Fasting (which means without water and any otherthing else for average 12 houres ) —Preceding unsigned comment added by True path finder (talkcontribs) 22:06, 14 April 2009 (UTC)[reply]

I would start at fasting. Sifaka talk 22:17, 14 April 2009 (UTC)[reply]
It will definitely depend on what is eaten before the fasting starts. The obvious possible negative effects would be dehydration, hypoglycaemia and possibly electrolyte imbalance. See: Fasting#Health effects for a list of some benefits of fasting. Also, the ramadan article states that "Fasting is meant to teach the person patience, sacrifice and humility", so that could be a good effect of fasting, although whether it will teach those things is quite hard to test. --Mark PEA (talk) 22:22, 14 April 2009 (UTC)[reply]
editing Wikipedia tests all three of those. SpinningSpark 22:31, 14 April 2009 (UTC)[reply]

Paradox

How would someone be able to eat their whole own self? Since eating one's whole self requires eating their mouth, how would they be able to eat afterwards? And since it requires eating their digestive system, how would they be digested? 58.165.25.29 (talk) 23:26, 14 April 2009 (UTC)[reply]

It's not a paradox - it's impossible! Duh! SteveBaker (talk) 23:36, 14 April 2009 (UTC)[reply]
Step 1: Create clone. Step 2: Buy fava beans and a nice chianti. Step 3: Eat clone (maybe with ice cream). Clarityfiend (talk) 23:52, 14 April 2009 (UTC)[reply]
No, fava beans and a nice chianti are only for the liver, not for the whole thing. Still, your approach is perfectly valid. I suggest a simpler, alternative approach. Step 1: spew generous amount of digestive juices. Step 2: digest self. Or, a slightly fancier alternative approach. Step 1: make a cocoon. Step 2: undergo histolysis. Step 3: remember that you are not a butterfly, stop. Step 4: Have a nice day :) . --Dr Dima (talk) 00:46, 15 April 2009 (UTC)[reply]
If you attempt to eat yourself, by all means do it with relish, however defined. Edison (talk) 04:45, 15 April 2009 (UTC)[reply]
This reminds me of Ouroboros.--Lenticel (talk) 07:51, 15 April 2009 (UTC)[reply]
Most human cells are not particularly long-lived, so if you were to consume all of your waste products, hair, nail trimmings, and sloughed skin cells, you will have eventually eaten most of yourself, in a manner. --Sean 16:36, 15 April 2009 (UTC)[reply]
Put on your will that your body be sent back in time to the present, then, assuming time travel exists and people are willing to fulfill your will, you can eat it. — DanielLC —Preceding undated comment added 01:22, 16 April 2009 (UTC).[reply]

Mailbox Key question.

In the neighbourhood where I live, all of the houses share a single mail pick-up point - it's like an apartment complex - there are no mailboxes outside each house. Instead, over by the park, there are rows of tiny locked mailboxes - one for each house - for which you have a key. There is also a row of much larger mailboxes for packages and such. When you get a large parcel, the mailman puts it into one of the large boxes, locks it and places the key into your individual mailbox. Now - here's the thing. The large mailboxes are set up so that you put the key into the lock - turn it to get your package out - but the key won't turn back again - and it won't come out of the keyhole so nobody can steal it. There is a second keyhole that the postman puts a special key into that releases the first key.

I've been trying to figure out how this fiendish mechanism works - the mailbox key looks like a perfectly ordinary key. It's obviously a ratchet or something - but curiosity demands that I know how it ACTUALLY works. Does anyone know of a diagram or something?

SteveBaker (talk) 23:46, 14 April 2009 (UTC)[reply]

This seems to be the patent of the mechanism in question, but after briefly skimming it, I'm no wiser about how the capture of the key is actually accomplished. Maybe I missed it, or maybe they just don't specify because it was a trade secret. APL (talk) 01:40, 15 April 2009 (UTC)[reply]
Oh, Here we go. I forgot to check the references. Here's the patent of the keylock. APL (talk) 02:15, 15 April 2009 (UTC)[reply]
Awesome. Figures 13 though 15 of the patent explain it quite well. My curiosity is now satisfied. Many thanks! SteveBaker (talk) 20:52, 15 April 2009 (UTC)[reply]
I also recall seeing that same mechanism in lockers in bus/train depots and airports. You would pay a quarter and get to use the locker/key once. Of course, as a result of recent terrorism, such bomb-friendly lockers have probably disappeared. StuRat (talk) 15:17, 15 April 2009 (UTC)[reply]


April 15

Congenital absence of appendix

The article on the appendix says that congenital absence of the appendix does not affect anything, but this case study published online shows differently. I'm looking for a second opinion. http://74.125.113.132/search?q=cache:yvGgkxsdJgEJ:www.bhj.org/journal/2008_5002_april/download/page-293-294.pdf+congenital+absence+of+appendix&cd=1&hl=en&ct=clnk&gl=ca —Preceding unsigned comment added by 99.227.94.24 (talk) 02:03, 15 April 2009 (UTC)[reply]

It's hard to make much of a single case study, described in somewhat broken English in the Bombay Hospital Journal. Looie496 (talk) 03:04, 15 April 2009 (UTC)[reply]
The authors of the paper have not explicitly stated the reason for the patient's symptoms (i.e. the diagnosis). One could debate whether this patient has "Congenital absence of appendix" or "Rudimentary appendix". If the latter, perhaps the diagnosis is indeed appendicitis of the rudimentary appendix? The authors do not describe their further management of the patient. Did they remove the rudimentary appendix? Did the patient improve?
The main problem here is with the definitions of "congenital absence of the appendix" and "rudimentary appendix". If they were defined more clearly, we could give a more helpful answer. Axl ¤ [Talk] 08:28, 15 April 2009 (UTC)[reply]

What is a naturally occurring isotope?

How are Antimony-121 and Antimony-123 two naturally occuring isotopes? Also, why is Antimony-121 more abundant than Antimony-123? —Preceding unsigned comment added by 174.6.144.211 (talk) 02:41, 15 April 2009 (UTC)[reply]

You might have a look at Isotope#Occurrence_in_nature and then let us know if something specific is unclear. --Scray (talk) 03:14, 15 April 2009 (UTC)[reply]
(ec)You can start by looking at Isotope#Occurrence in nature and Nucleosynthesis. In general, the various processes that transform the elements yield nuclei with various combinations of protons and neutrons. Only certain combinations are stable. Others will almost immediately decay into other elements. Still others will be relatively stable and may take years millennia, or longer to decay. An element is defined by the number of protons contained in the nucleus, and a given element may only be stable or have a measurable life with a certain number of neutrons. For example, Carbon (see also Isotopes of carbon) have three isotopes with a half life of more than a few seconds, but those three isotopes – with six protons, and six, seven, or eight neutrons – have very long lives (in human terms). Those are the isotopes general identified as naturally occurring. -- Tcncv (talk) 03:26, 15 April 2009 (UTC)[reply]
(Sorry, I guess I got off track) To generalize, there is no reason why an element cannot have more than one stable isotope. hydrogen has two, oxygen has three, and as you found antimony has two. As to why they have different abundances, I expect that is due to the different isotopes being formed through different mechanisms, or if formed through the same method, that method (such a supernovas) just happens to produce more of one isotope than another. -- Tcncv (talk) 03:56, 15 April 2009 (UTC)[reply]
Does the varying longevity of isotopes of an element have to do with the curve of binding energy? Edison (talk) 04:41, 15 April 2009 (UTC)[reply]
It does, to some extent. Loosely, radioactive decay can be thought of as a Quantum tunelling phenomena and the tunneling probability is dependent on the released energy. However, the transition rate is more heavily influenced by various selection rules governing the decay mode. For example, in beta decay there are selection rules which arise out of conservation of angular momentum and parity. The influence of these selection rules can be huge - it is energetically possible for Calcium-48 to undergo single beta decay but it has never been observed to decay via. single beta decay (it has been observed to undergo double beta decay though).
Going back to the original question, it is possible to have two stable (or effectively stable) isotopes of an element as long as there is no significant decay mode to another element or isotope. A nuclear decay chain can be thought of as a waterfall, where nuclei keep "falling" into lower states until there's nothing to fall into. It just so happens that both Antimony-121 and Antimony-123 are at the bottom. Someone42 (talk) 09:09, 15 April 2009 (UTC)[reply]

Precambrian craton formation

Hi, I've been studying a little about the geological history of the Earth preceding the ordovician period, and I've come across some information on the cratons that are known to have existed, as well as an animation of how they were believed to have been positioned at the point when the crust first formed into plates. There is an animation on Wikipedia here (warning, may enduce seizures), but I find the morphing process is much easier to see in this video clip (youtube, starts at about 2:15), though I can only assume they are working from similar data.

I'm specifically concerned with the peculiar string of "islands" linking the Siberia craton and the Baltica craton. By the Carboniferous period they seem to have all been gobbled up by the larger cratons as pangea is formed. I assume that the reason they have been separated from Siberia and Baltica in the first place is because of some difference in the mineral/fossil record that indicates separation before Carboniferous, but I'm curious about the positioning of these mini-cratons, and the size. Are they being placed in a string simply because it is impossible to tell exactly where they were located, or is there some reason to believe that they were arranged in such a manner? And if the land fossil record for that period is as sparse as I think it is, how can they rule out that these mini-cratons were not in fact much larger, or merged together into a larger mass?

I don't know if anybody can help with these questions, but opinions are welcome! Thanks ahead! 124.154.253.25 (talk) 03:35, 15 April 2009 (UTC)[reply]

It sounds like you are talking about a volcanic arc. I don't know why geologist believe that there was an arc link the cratons you are talking about. Volcanic arcs, like the one at the caribean or aleutian islands for instance, are actually quite common and they do indeed align along a string of islands as you described. Dauto (talk) 04:36, 15 April 2009 (UTC)[reply]

Yes, it does look very like a volcanic arc, and if it is, the real question is what kind of evidence led scientists to their conclusion? I thought perhaps that they may have evidence for a certain number of small cratons separate from Siberia and Baltica, likely created by a similar event (such as a volcanic hot spot under a floating plate) assuming they are geologically similar. Fossil evidence could then further show that they were separated by water, or alternately the cratons could have been discovered very far from each other. I'm not educated enough in any field to make wild guesses like that though! 124.154.253.25 (talk) 06:35, 15 April 2009 (UTC)[reply]
The evidence nowadays would be a line of acid volcanics such as dacite or andesite from that period, possibly embedded in marine sediments. There is likely to be folding in marine (or back arc) sediments, as the volcanic arc was fused onto a craton. In this particular case I don't know the specific evidence. Graeme Bartlett (talk) 06:46, 15 April 2009 (UTC)[reply]
This paper by Robin Cocks and Trond Torsvik [1] includes a fairly detailed discussion of all the 'bit and pieces' or terranes caught up between Baltica and Siberia. I'm assuming that you're mainly talking about 'Kazakhstania'. I think there is some faunal evidence, i.e. an analysis of high v. low latitude types of fossil and possibly palaeomagnetic data allowing the construction of an apparent polar wander path for at least some of the terranes. Remember that geoscientists have difficulty reconstructing continents where there are no 'magnetic stripes' preserved to help close up intervening oceans (that's pretty much anything before the middle Jurassic or about 200 million years ago); several distinct reconstructions may be possible from the same data. Mikenorton (talk) 10:22, 16 April 2009 (UTC)[reply]
Thanks for the link! Really interesting and there's loads there to delve into. 124.154.253.25 (talk) 02:00, 21 April 2009 (UTC)[reply]

Pharmaceutical naming

How do drugs get their common or generic names? I'm thinking here of lorazepam, minoxidil, sildenafil, etc. They're somewhat random syllables, although I'm pretty sure that anything ending in -mab is a monoclonal antibody, so there's some correlation sometimes. Is there a convening body to dispense these names, is there some centralized random generator everyone can go to, or what?

Related question, when the actual corporations do their branding, are they wide open, or do they obey some rules also? For the examples above, how do we arrive at Ativan/Temesta; Rogaine/Regaine; Viagra/Revatio?

Do we already have an article on this? I tried some possibly lame searches, but I'm stumped. Thanks! Franamax (talk) 07:24, 15 April 2009 (UTC)[reply]

Medicines will often have more than one name:
  • a generic name, which is the active ingredient of the medicine
  • a brand name, which is the trade name the manufacturer gives to the medicine.
The generic name is the official medical name for the active ingredient of the medicine.
The brand name is chosen by the manufacturer, usually on the basis that it can be recognised, pronounced and remembered by health professionals and members of the public. An example would be Viagra - this is the well-known brand name given by Pfizer to the generic medicine sildenafil. (Brand names are capitalised; generic names are not.) More here. Cuddlyable3 (talk) 09:56, 15 April 2009 (UTC)[reply]
We've discussed these issues before on the ref desk, e.g. generic names and trade names. These are just 2 of the first few hits I got after I searched the RefDesk archive (above) for "drug names". BTW, I was told 5 or 6 years ago that focus group research had indicated that "v", "x", and "z" were "strong" letters to include, and since then we've seen "Vioxx", "Zyvox", and the list goes on... --Scray (talk) 11:28, 15 April 2009 (UTC)[reply]
Franamax, your own screen nama also kind of sound a little bit like a brand name. Dauto (talk) 12:24, 15 April 2009 (UTC)[reply]
Hey, you noticed! :) The marketing committee was up all night on that one. It's a combo of my given name and "max", connoting large or major. If the Pope can be pontifex maximus I figured I could follow the same strategy. Sadly, I am not yet considered infallible. ;) Franamax (talk) 18:31, 15 April 2009 (UTC)[reply]
Heh, I've occasionally thought of using the handle Antonissimo. —Tamfang (talk) 19:06, 15 April 2009 (UTC)[reply]
Ooohh, an absolute superlative in singular masculine form - that's good! The other strategy is to go for self-deprecation. When I first saw User:Mild Bill Hiccup, I laughed for quite a while. (For those not familar with American culture, the contrast is here). Franamax (talk) 20:03, 15 April 2009 (UTC)[reply]

Thanks for the pointers! The World Health Organization Expert Panel on the International Pharmacopoeia and Pharmaceutical Preparations handles the generic names - why didn't I think of that. Boy, the United Nations has an article on everything! Franamax (talk) 18:31, 15 April 2009 (UTC)[reply]

resistivity of vacuum

what is the resistivity of vacuum? is it zero? --harish (talk) 11:20, 15 April 2009 (UTC)[reply]

No, it's infinite. Since ρ is defined as electric field divided by current density, and there is no current in a vacuum (since there are no charge carriers), ρ is infinite. Note that this is only hypothetical, since nobody has ever created or discovered a perfect vacuum. --Heron (talk) 13:07, 15 April 2009 (UTC)[reply]
Are you sure? If the electric field was strong enough I would expect electrons to be ripped off the negatively charged object and be attracted to the positively charged object, so a current would flow. (Although, I guess technically it wouldn't be a vacuum then.) --Tango (talk) 14:46, 15 April 2009 (UTC)[reply]
How about electron flow through a CRT? Glowworm. —Preceding unsigned comment added by 98.17.38.45 (talk) 14:19, 15 April 2009 (UTC)[reply]
A CRT has electrons flowing through it, and is therefore not a perfect vacuum. — DanielLC 16:08, 15 April 2009 (UTC)[reply]

A "perfect vacuum" is merely" a philosophical construct that is never observed in practice" per the article. Outer space is 1×10-6 to <3×10-17 torr, compared to zero torr for a perfect vacuum. Intergalactic space could have as little as 1 molecule per cubic meter. But a heated filament in that region would emit electron toward a positively charged plate and the current would be as great as in a common vacuum tube(valve) or cathode ray tube, since these devices do not depend on residual gases to carry the current. That current, known as the "Edison effect" or thermionic emission, can be characterized by a curve relating current and voltage. It does not have a constant resistance, and the space does not have a constant restivity. It does not follow Ohm's Law any more than a semiconductor does. The tubes are evacuated to the greatest extent possible. Do electrons travelling through a region of otherwise pure vacuum count as "particles" and how much pressure would each electron in, say, one cubic centimeter create? Edison (talk) 16:08, 15 April 2009 (UTC)[reply]

Electrons have mass, so when an electron strikes a surface it exerts a force – a pressure. The amount of force depends on the speed of the electron. In the case of a radio vacuum tube the situation is special because the electrons are not moving at random. They are all moving in one direction – toward the anode. – GlowWorm. —Preceding unsigned comment added by 98.17.38.45 (talk) 22:01, 15 April 2009 (UTC)[reply]
A perfect vacuum is not merely philosophical, since I can define some arbitrarily small volume of space which contains no particles at the time I measure it. But at the next instant, those damn physicists insist it will get filled up again - nature abhors a vacuum and Casimir keeps putting in new particles.
Nevertheless, if the definition of a perfect vacuum is zero particles and you send a particle (or wave that may also be a particle, whatever) into that space - it is definitely not a perfect vacuum anymore!
And "pressure" is a statistical concept, so I'm not sure how valid it is to talk about a single particle exerting a "pressure" - wouldn't we instead term that an "impact force"? Franamax (talk) 22:25, 15 April 2009 (UTC)[reply]
To mend the gaping flaw in my reasoning, for "particle" substitute "particles possessing mass". EM waves pervade the universe and are indeterminate in location. Erk! Franamax (talk) 22:41, 15 April 2009 (UTC)[reply]
I spoke of a single electron, but I had in mind the statistical effect of many electrons. – GlowWorm.
While it's true that it is not possible to create a perfect vacuum, that does not make a merely philosophical construct. It is a physical idealisation. There is nothing wrong with talking about the properties of physical idealisations. They are very usefull for thought experiments, for instance.
The OP asks about the vacuum resistivity. Paradoxically, the answer to this question can be both zero and 'infinity' depending on how we chose to understand the question. If you shoot electrons creating a streem, these electrons would continue to move unoposed even in the absence of an external field, zero resistivity! I seriously suspect that this was the setup that the op had in mind. But that's not what people usually understand by measument of resistivity. The normal understanding is to apply an external field and see what happens. If you do that, you get no current because there are no carriers, infinite resistivity! —Preceding unsigned comment added by Dauto (talkcontribs) 03:45, 16 April 2009 (UTC)[reply]
It's zero. The more fundamental understanding of resistance is not the ratio of potential difference to current (for one thing, that works only for DC), but rather a measure of the energy dissipated when a given current flows through the medium (power dissipation is I2R, where I is the current). Since vacuum dissipates no energy, it has no resistance. --Trovatore (talk) 04:01, 16 April 2009 (UTC)[reply]
I disagree with the "zero resistance" claim. A superconductor will carry a certain amount of current with no voltage across it and no energy dissipate, so zero resistance applies. A perfect vacuum (no electrons or ions) can have substantial voltage across it with no current. Don't tell me a vacuum tube diode with the filament cold has zero resistance when I apply 12 volts across the cathode to anode gap and no current flows. Edison (talk) 19:05, 16 April 2009 (UTC)[reply]
I stand by what I said. The vacuum has no resistance at all. That does not, of course, make it a superconductor, or even a conductor; it means only that no energy is lost to scattering when charge carriers flow through it. --Trovatore (talk) 18:36, 17 April 2009 (UTC)[reply]


Regarding your difficulty in making the current actually flow from the cold filament, well, the fact that it depends on the temperature of the filament should give you a clue that this is not a property of the vacuum. If the issue were related to the vacuum, then heating the filament wouldn't change it.
Rather, the discrepancy between potential difference and current is properly addebited to the non-ohmic contact between the metal and the vacuum. --Trovatore (talk) 19:48, 17 April 2009 (UTC)[reply]


I don't think the resistivity will be infinite even in the absence of carriers, because beyond a certain field strength the vacuum itself will become a carrier: electron-positron pairs will be created out of the void and split apart, going in opposite directions and producing a net current. This phenomenon is a consequence of quantum electrodynamics. Looie496 (talk) 05:11, 16 April 2009 (UTC)[reply]
According to the indentation, you're responding to me, but I didn't say the resistivity was infinite. I said it was zero. --Trovatore (talk) 05:47, 16 April 2009 (UTC)[reply]
Looie, that which you talk about is called the dielectric strength. It's not the samething as resistivity. You are right. Vacuum's dielectric stregth is not infinite. A simple back of envelop calculation (Or better yet, a dimentional analysis) gives the vacuum dielectric strength which is 9 to 10 orders of magnitude higher than the dielectric strength of any material. Dauto (talk) 17:57, 16 April 2009 (UTC)[reply]
There is a voltage drop across a resistor if another resistor is in series with it to form a voltage divider. (Without the second resistor, the full voltage of the power supply will appear across the resistor, and this would not be called a voltage drop.) A voltmeter will show a voltage drop between the filament and plate of a radio vacuum tube. This indicates that the vacuum in a vacuum tube is a resistor, even though it dissipates no power.(There is a resistor in the plate circuit to form a voltage divider.) – GlowWorm. —Preceding unsigned comment added by 98.17.41.25 (talk) 10:35, 16 April 2009 (UTC)[reply]

If you have two electrodes in a perfect vacuum with some constant potential difference then there will be a nonzero tunnel current. Count Iblis (talk) 13:25, 16 April 2009 (UTC)[reply]

formation of pure alkyl nitrile

Hi, I need to find a way to synthesise the following product: it's a n-hexane with a CN group attached to the 2-carbon. My "base" reactant has to start with a hydrocarbon, though presumably I can employ a whole host of special reagents after that. I can't find a suitable process? I must obtain a single pure product, no isonitrile stuff (unless you can tell me an easy way to purify it without jumping through hoops). I don't think Kolbe nitrile synthesis will give me that, and all the other processes I've looked up either seem to apply to aromatic compounds or they leave oxygen groups on the carbon chain. John Riemann Soong (talk) 11:28, 15 April 2009 (UTC)[reply]

Working forwards, what reactions do you know that convert an alkane (is it correct to assume "hydrocarbon" means alkane?) into something-else (i.e., what kinds of functional groups can you make in Step 1)? Working backwards, what are ways you can make a nitrile (what would be a precursor) and what are ways you can add a "prebuilt" nitrile or nitrile-containing structure onto some other structure (i.e., what could be the precursor for the final step)? Do you have to start with a 6-carbon hydrocarbon? DMacks (talk) 04:23, 16 April 2009 (UTC)[reply]

Invasive species

(Not homework.) Dog (something)bine. Southern Ontario. Grows in underbrush of dead pine forest. A short human-height plant, wood-like or stalk-like (don't remember which). Nearby red pine dying from fungual infection. Any ideas? Thanks. ~AH1(TCU) 11:33, 15 April 2009 (UTC)[reply]

Are Woodbine#Climbing_plants or "dogwood vine" or "creeping dogwood" helpful? --Sean 16:43, 15 April 2009 (UTC)[reply]

Do scientists have to pay to have their work published in a peer-reviewed journal?

I was wondering if scientists have to pay to have a paper published in a peer-reviewed journal? The Open Chemical Physics Journal says that they "aim at providing the most complete and reliable source of information on current developments in the field" but charge anywhere from $450 to $900 to have a paper published. This sounds like a vanity press. The fees are listed towards the bottom of the page. [2] A Quest For Knowledge (talk) 12:18, 15 April 2009 (UTC)[reply]

No they don't and that's a vanity journal - seems to popular with 9/11 cranks. --Cameron Scott (talk) 12:25, 15 April 2009 (UTC)[reply]
Yes, and that's partially why I'm asking. In particular, Bentham's Open Chemical Physics Journal published an article [3] authored by 9/11 conspiracy theorists. The most prominent is Steven E. Jones who was relieved of his teaching duties and placed on paid leave at Brigham Young University after he published an article on BYU's web site promoting 9/11 conspiracy theories.[4] They claim that the destruction of the World Trade Center was not the result of terrorist attacks. Instead, they claim that the US government had planted explosives inside the WTC prior to 9/11 and that the towers were brought down by controlled demolition. The article Bentham published claims that active thermitic material was discovered in the dust from the 9/11 World Trade Center remains which 9/11 conspiracy theorists argue are the result of a controlled demolition. If Bentham is a vanity press, that would explain how this paper was published. If, however, this is a legitimate peer-reviewed scientific journal, how did it get past a peer review? A Quest For Knowledge (talk) 13:07, 15 April 2009 (UTC)[reply]
While I know nothing in this case it is rather an open secret that peer review depends very heavily on the opinions of the editor, who is in charge of assigning reviewers, shepherding the article through to publication, etc. Peer review is not without its problems. (And not all important articles are peer reviewed.) --98.217.14.211 (talk) 01:15, 16 April 2009 (UTC)[reply]
I don't know whether this journal is vanity or not, but with open access journals often the costs are put upon the authors of the articles (more usually their institutions), and the scholars in question usually have money for this written into their grant proposals for this purpose if they are planning to go in this direction. For example, the Public Library of Science, which is very reputable, works this way. --98.217.14.211 (talk) 12:31, 15 April 2009 (UTC)[reply]
Journals – open access and otherwise – often defray some of their costs through page and color charges (extra fees for color figures). Page charges are higher for open access journals which enjoy little to no other income from subscription fees. As the anon above notes, the very reputable PLoS family of journals charges substantial publication fees: [5].
That said, it's worth looking closely at any journal which charges page or publication fees — some of them are vanity presses. Reputable journals will usually have an indication that they waive fees for authors who can't afford them; the PLoS statement is in the second paragraph of that link, above the prices:
We offer a complete or partial fee waiver for authors who do not have funds to cover publication fees. Editors and reviewers have no access to author payment information, and hence inability to pay will not influence the decision to publish a paper.
Be very wary of any journal that isn't prepared to make a similar guarantee. TenOfAllTrades(talk) 12:43, 15 April 2009 (UTC)[reply]
The journal(s) in question (Bentham) began in 2007. To quickly fill up about 300 "open" journals, Bentham spammed universities looking for papers. Anything that was thrown at them that appeared remotely interesting was "peer reviewed" and published online as quickly as possible. The main goal was to get a lot of hits. The second goal was to fill the journals with articles as quickly as possible. The third (perhaps fourth, fifth, or sixth) goal was to ensure the articles had a respectable peer review. -- kainaw 13:29, 15 April 2009 (UTC)[reply]

I think there are plenty of legitimate journals with page charges. Not the most high profile journals, but nonetheless respectable places for researchers in specialty fields to publish their work. I believe my colleagues who work in leather chemistry pay page charges. Some fields are very small, and their journals just don't sell a ton of subscriptions. I don't think this necessarily says anything about the quality of the work or the journal. ike9898 (talk) 13:39, 15 April 2009 (UTC)[reply]

There seems to be an inherent conflict of interest there. That is, if the journal is short on funds and can get them by publishing articles, they may be tempted to publish total crap just so they can meet their payroll. StuRat (talk) 15:01, 15 April 2009 (UTC)[reply]
If its a peer reviewed journal, then that shouldn't happen as total crap should not pass review. Besides, journals don't chase money that way, because its ultimately self defeating. What they really chase is impact factor. The higher your IF, the more people want to publish in your journal and the more people that want to read it, which means more money for you. If you publish crap, your IF will decrease which means less people will want to publish in your journal which means less money for you. There is possible COI in a peer reviewed journal, though, as publishers can manipulate IF's in devious ways. Rockpocket 19:15, 15 April 2009 (UTC)[reply]
What you say makes sense for the long-term future of the journal, but short-term concerns like avoiding immediate liquidation tend to outweigh lofty long-term goals. StuRat (talk) 16:14, 19 April 2009 (UTC)[reply]

FWIW, I'm more concerned about the fact that they published an article promoting 9/11 conspiracy theories. A Quest For Knowledge (talk) 15:06, 15 April 2009 (UTC)[reply]

I found a second Bentham article promoting 9/11 conspiracy theories in a discussion from the archives of the Reliable Sources Noticeboard regarding another Bentham journal, The Bentham Open Civil Engineering Journal. [6] Unfortunately, the discussion of the journal's reliability doesn't go very far. A Quest For Knowledge (talk) 15:52, 15 April 2009 (UTC)[reply]


The existence of page charges does not provide any evidence of a journal being a less than reliable source. Googling "journal page charge" showed "article processing charges" of $1400-$1550 (US) for BiomedCentral, which in turn tabulates the page charges for a number of journals, including Cambridge University Press ($2700), Blackwell ($3000), American Physiological Society ($3000), Elsevier ($3000), Wiley ($3000), and Springer ($3000). Some journals have lower page fees. If you do not like Jones and do not believe his analysis of the building collapses on 9/11 (notably the only modern highrises which ever collapsed from a fire or impact), do not try to discredit him or his analysis by claiming that page charges or "processing charges" make a journal "vanity press." Edison (talk) 15:53, 15 April 2009 (UTC)[reply]

It should be noted that it costs more to publish if the figures are in colour. Agree with Edison that a journal should not be considered a vanity press due to page charges. David D. (Talk) 15:59, 15 April 2009 (UTC)[reply]
Note that Nature (journal) (which is about as respectable as you can get) charges fees for colour pages (though not non-colour pages), but its rather difficult to publish a paper in Nature without colour. Scientific publishing has a strange business model. Most journals have a very limited circulation, so to cover costs (and make a profit) the publishes charge at both sides - they charge the scientists to publish and they charge the scientists to read. Amazingly, the scientists also provide most of the content (the manuscripts), yet the publishers often demand the scientists hand over the copyright of that work also!
For such smart people, the scientists appear complicit in getting screwed over every which way. Why? Well, its because the scientific publishing system is so important to progressing in a scientific career, that scientists are willing to do whatever it takes to get their papers published. One could take a principled stand and refuse to comply, but ultimately your paper will not get published in a top journal and your career will not progress. There are efforts to change the system, such as the PLoS and BioMed Central journals, where you typically pay for publishing only. These are similar to Wikipedia is spirit, freely disseminating knowledge is the goal. What distinguishes them from vanity publishing is, as other point out above, they all have clauses saying you only have to pay if you can. If you genuinely don't have the grants funds to cover the costs of publishing, then you don't have to pay. Rockpocket 16:27, 15 April 2009 (UTC)[reply]

If I were trying to get a job in academia or to get tenure or to get a research grant renewed, $3000 for a publication would seem like chump change, besides the fact that the grant is likely to pay it. How many hours and dollars went into the research and writing, do you suppose? Anything to pad the resumé. Edison (talk) 18:52, 15 April 2009 (UTC)[reply]

Many would argue is that publication is just one of the many costs of doing research. Just like presenting your work at conferences - some people get invited and have their way paid, but others are on their own to find funds they can use for this. ike9898 (talk) 19:50, 15 April 2009 (UTC)[reply]

Just to clarify, I've never heard an employed scientist spending their own personal money to publish. Sometimes you can use grant money or department funds or something else. It's just that you'd rather save that money if you could for supplies, equipment and salaries. ike9898 (talk) 20:03, 15 April 2009 (UTC)[reply]

When I was an unemployed scientist, I would certainly have paid page charges to get a publication in a good journal. Maybe you cash in some U.S. Savings bonds, or hit up the parents for some cash. It could lead to becoming an employed scientist. The alternative might be writing off many years of college and graduate school and accepting a less desirable career path. Do not underestimate the value of a publication if the resumè is a bit anemic. Edison (talk) 18:59, 16 April 2009 (UTC)[reply]
I agree. It also demonstrates true dedication to your work. ike9898 (talk) 13:56, 17 April 2009 (UTC)[reply]
Many journals that have page charges are willing to waive them in cases of need. Looie496 (talk) 16:34, 19 April 2009 (UTC)[reply]

Rate Constant, k

Hi all I after having read the article on rate constant I am still a little confused over what k is. Is it the rate of the reaction when the reactants are at a concentration of 1 mol dm-3 I've also heard that k isn't even a constant anyway as it if affected by the likes of temperature. How bizarre to call it a constant then! Another thing which particularly confuses me is writing the rate equation for a zero order reaction. I believe it would be written: Rate = k. Which means rate = rate constant. What does this mean? I hope what I've written makes sense - I find chemistry very confusing. Thanks in advance to all who help. —Preceding unsigned comment added by 92.10.162.104 (talk) 12:29, 15 April 2009 (UTC)[reply]

The arrhenius equation states what k is. Yes k is not really a constant, rather a coefficient. If the order of reaction is 0 with respect to all reactants, then yes, rate = k (mol dm-3 s-1).
Clearly, if the total order of reaction (m + n) is 0, then changing the concentration of reactants will have no effect on how fast/slow the reaction is (because any number to the power of 0 equals 1). --82.21.28.65 (talk) 12:58, 15 April 2009 (UTC)[reply]
They may call it a "constant" since, in this particular equation, it is constant with respect to reaction progress. When you use the rate equations, you pick a value of k and stick with it. But yes, it's not really constant - I think maybe "rate coefficient" would probably have been a better name, but I think we're stuck now :) --Bennybp (talk) 13:34, 15 April 2009 (UTC)[reply]

Armour made from tantalum-niobium alloy - how effective?

Any metallurgists here? In Terminator: The Sarah Connor Chronicles, it has been stated that the armoured parts of the more recent Terminator T-8xx models are constructed from an alloy of tantalum and niobium, as it was discovered that this was more effective on the battlefield of the future than the previously-used titanium, due to its greater heat-resistance (in response to the development of laser cannons and particle beam weapons, I suppose, as the war has been going on for decades in the story), comparible resistance to bullets and lighter weight (I think).

So, in the world of Real Life, how would armour plating made from an alloy of tantalum and niobium really measure up in a war situation? Not nescessarily when attached to a robotic soldier, just in general, I mean. Thanks. --81.77.155.81 (talk) 15:13, 15 April 2009 (UTC)[reply]

My first thought is that the armour would be vastly more expensive than titanium, since there appears to be three orders of magnitude difference in crustal abundance.
Looking at the various properties: Ta/Nb is much more dense than Ti, so a given thickness would be much heavier; the specific heat capacities are comparable, but Ta/Nb has greater conductance, so it would be better at absorbing laser input; also Ti becomes brittle on heating in presence of oxygen, if you can get a good laser shot in, that bit will turn blue and fracture on mechanical impact; Ta/Nb would probably be better with particle beams, due to its higher density; the hardnesses are comparable but I have a suspicion that Ti might be more susceptible to brittle fracture than Ta/Nb, not sure there - recall that the SG-1 team beat the replicators by using bullets, which shattered them.
If you're giving me unlimited funds, I'd probably pick the Ta/Nb solution. In fact, I'm not sure why you made us use Ti in the first place. In the end, it depends on what you want to use the armour against, and how fast you want the thing you're protecting to move. The best way to withstand an enemy shot is to move out of the way, heavy armour restricts that ability. If you're going to stand and take fire, there's a lot to be said for the sandwich technique using depleted uranium.
And when it comes right down to it, I'll take whatever Cameron is made of - those are very impressive construction materials! :) Franamax (talk) 20:33, 15 April 2009 (UTC)[reply]
The high density of tantalum and uranium as well makes it a problematic alloy to use in large scale, for a humanoid cyborg, but if you do not care if one terminator is only capable to run ultra hard surface and simply would get stuck in a normal madow it is good. Normally a superalloy similar to the stuff used in turbines, a alloy based on nikel or cobalt with aditions of other refractory metals like tantalum and rhenium gives the best performance when heated. But a woven fabric of carbon nanotubes as interior of a composit material would be my favorite. For a particle beam weapon the exact sort of particle is important, protons, neutrons, electrons, alpha particles are positrons would all need a material capable to with stand the negative effects of nuclear conversions of the material. For Protons the answer can be found in the normal satelite business, because solar wind is made mostly from protons.--Stone (talk) 21:01, 15 April 2009 (UTC)[reply]
The OP is asking about the present-day performance of the armour, so perhaps I've led us astray discussing particle-beam weapons. Laser and microwave weapons are reality or close to it, so that's where we should focus, along with projectile and heat munitions (and vacuum bombs, which render the issue moot if you don't have a secondary air source). I think we agree on the tradeoff between weight and mobility.
As to a nanotube inner lining, would this be part of the breathable atmosphere inside the armour? There is growing evidence that carbon nanotubes are as toxic as asbestos, so I think maybe I'd rather have a chrysotile blanket instead (not amphibolic!) At least then I could quantify the risk. Franamax (talk) 21:50, 15 April 2009 (UTC)[reply]

Hair & Facial Hair Question

Hello, if this has been asked before, I apologize, I couldn't find where it had been asked. Anyway on to the question: How is it possible for some body hair to be a completely different color than all the other hair on someone? For example I am blonde most of my arm and leg hair is blonde (dirty blonde to be exact). But my facial hair is RED. I'm not really upset about it and I'm not looking for answers on how to dye or change the color of my hair, I'm just curious about the origins and reasons behind this.

Thanks! 12.204.178.35 (talk) 15:33, 15 April 2009 (UTC)[reply]

Us blondes have hair of many colours, on our heads and elsewhere on our bodies. You will probably have red hair on your head as well as blue (honestly), black and red. It's just that the blonde is more prominent on your head, whereas the red is probably more prominent on your face. Nothing to worry about - it's a quirk of nature. For the record, the only blonde hair on my body is on my head!--TammyMoet (talk) 15:48, 15 April 2009 (UTC)[reply]
Yes I've noticed the array of colors I seem to have! I wasn't too sure if it was kosher to talk about but my pubic hair isn't blonde..that's for sure. It's good to hear this isn't uncommon, but I'm still wondering why this is? Aside from artificial coloring you don't seem many dark haired people with such a wide variety of colors. 12.204.178.35 (talk) 16:19, 15 April 2009 (UTC)[reply]
Blue?! —Tamfang (talk) 19:17, 15 April 2009 (UTC)[reply]

I'm afraid TammyMoet's response isn't quite true. The gene responsible for red hair in most people is MC1R. When this was first being studied, the scientists found a strange correlation. They found that if you have two null alleles (two copies of a gene variant that doesn't function fully) then you have an increased likelihood of having red hair. However they also found that if you have just one null allele (what we call heterozygous for that gene), then you may have a different hair colour on your head, but that you have an an increased likelihood of having red body hair. I was one of the scientists working on that project, and having a reddish beard myself, I decided to sequence my own MC1R gene to see if I was a heterozygote myself. Turns out I am.

Quite why being a heterozygote results in this effect isn't quite clear. However it may be something to do with different effects of haploinsufficiency on different hair types. More simply, that the hairs on your head manage to make enough eumelanin with just a single copy of MC1R, but the hairs on the rest of your body need more MC1R. If you want to read the scientific data behind this, let me know and I'll find it for you. Rockpocket 17:25, 15 April 2009 (UTC)[reply]

I don't see how what you said contradicts what I said: indeed, it seems to be the science behind the phenomenon. I was sure there was a genetic component to it, however I was also sure there'd be a geneticist along in a while - and I was right! Thanks.--TammyMoet (talk) 20:45, 15 April 2009 (UTC)[reply]
A "Blue Heeler"
A "Blue Picardy Spaniel"
The thing I took issue with was the suggestion people have blue hair. There are no blue pigments naturally occurring in mammals, so (healthy) humans don't make blue hairs. Rockpocket 21:35, 15 April 2009 (UTC)[reply]
However, healthy humans might look at a hair and perceive it as being blue. I run into that terminology problem all the time. Franamax (talk) 21:58, 15 April 2009 (UTC)[reply]
There are plenty of dog breeds called "Blue somethingorother" - they aren't literally blue either - but the problem (again) is that blue is just a word - and it means whatever people want it to mean in some specific context. SteveBaker (talk) 22:25, 15 April 2009 (UTC)[reply]
Even blue jays lack blue pigment. -GTBacchus(talk) 22:31, 15 April 2009 (UTC)[reply]
In fact, for some people there's no such separate concept as blue. The world is painted in shades of grue! Franamax (talk) 22:45, 15 April 2009 (UTC)[reply]
Yikes, I better light a torch then! ArakunemTalk 22:50, 15 April 2009 (UTC)[reply]
Just to be clear, the preceding reflects two very different things. Blue jays have no blue pigment, but are coloured blue due to a process referred to as structural colour. Grue refers to the different ways languages classify colours. See here for the actual article on the topic. Matt Deres (talk) 13:33, 16 April 2009 (UTC)[reply]

Calculating Kc equilibrium constant

Hi I am really stuck with the following problem. The reaction needed for the question is: Cu2+ + H2A (a fictional substance)(equilibrium arrows - don't know how to insert them) CuA + 2H+. The question states that at the beginning there are 0.005 moles of Copper ions and 0.005 moles of H2A. At equilibrium there is 0.0049 moles of CuA. Therefore: Kc = [CuA] [H+]2/ [Cu2+] [H2A] The question asks me to calculate the number of moles of all substances at equilibrium. So I already know that equilbrium the concentration of CuA is 0.0049 moles. Therefore at equilbrium there is (0.005-0.0049) 0.0001 moles of Copper ions left. Now comes the part I'm not too sure on. Looking at the answer the concentration of H+ = 0.00492 = 2.401x10-5 mol. My question is how can it be known for sure that the concentration of H+ ions is the square of the concentration of CuA? Is it not possible that the concentration of H+ ions would not be the square of 0.0049 moles at equilibrium? How do we know this is the case? Sorry for asking silly questions and thanks in advance to anyone who can help. —Preceding unsigned comment added by 92.8.198.137 (talk) 18:51, 15 April 2009 (UTC)[reply]

I don't think squaring the concentration of CuA will give you the concentration of protons. Check the stoichiometry of the reaction (and note the link I provided), specifically the ratio of CuA to H+ in the equation, and ask yourself: for every mole of CuA, how many moles of H+ will be generated? Looks like AFTER you have answered this question and determined the concentration of protons, you may want to square it in calculating Kc. I hope this helps. --Scray (talk) 22:10, 15 April 2009 (UTC)[reply]
As Scray said. Rethink the meaning of the formula: [H+]2 does not mean you have to square [H+] to figure out the concentration, but the other way round: [H+] is the concentration and it needs to be squared to obtain the equilibrium constant. See here. Steipe (talk) 20:23, 23 April 2009 (UTC)[reply]

Hearling loss from high frequencies

Is it possible to have hearing damage if you listen to imperceptibely high frequencies (20 khz~ish) at high volumes? 206.176.119.180 (talk) 18:58, 15 April 2009 (UTC)[reply]

I can't answer your question, but I just wanted to help you understand the relationship between volume and intensity. Volume is a human phenomemon and the correct word is called loudness and is measured in the decibel system while you are asking a question that will probably get answered using the terminology of sound intensity which is power per square meter and is related to the strength of a sound wave. A very intense sound wave is, for example, has a very high value of watts per square meter and vice versa. Even if you can't hear the sound wave you speak of in your question, it will still have intensity, although it will not have loudness because if you can't hear a sound, that means it has zero decibels, but not necessarily zero watts per square meter and if I can further request to whoever answers the OP's question: is my understanding of the relationship between intensity and loudness correct? is loudness, or intensity--specifically, what causes hearing damage? The article goes into generalities such as "caused by a wide range of biological and environmental factors... etc...". I suggest read the Hearing impairment article because it gives some good info, but doesn't specifically answer what you are asking--which is a good question, because I imagine those who read the article would like to know as well. 71.55.186.77 (talk) 20:28, 15 April 2009 (UTC)[reply]

It is incorrectly said above that if you can't hear a sound, that means it has zero decibels. Without a specified reference sound pressure, a value expressed in decibels cannot represent a sound pressure level. With any specified reference sound pressure, a sound pressure that is equal to the reference has zero decibels relation to it. The "loudness of inaudible sound", regardless of reference sound pressure, is actually minus infinity decibels. Cuddlyable3 (talk) 21:29, 15 April 2009 (UTC) [reply]

There is evidence from studies of the effects of ultrasound scans on unborn children that this does not harm their hearing - even though the intensities (watt/sq.meter) are quite high and the amniotic fluid (which is mostly water) transmits the sound very efficiently. I assume that this is because the eardrum has inertia and elasticity and at sufficiently high frequencies is simply not moving measurably in the tiny amount of time before the sound wave reverses direction. This (admittedly sketchy) information suggests that the transmission of sound energy from outside of your ears to inside (where high energies could certainly wreck your hearing) must be a function of frequency. That doesn't mean that a super-loud burst of 20kHz wouldn't harm you though...it just says that we can't simply measure the raw intensity and nothing else. SteveBaker (talk) 20:47, 15 April 2009 (UTC)[reply]
Agree. The possibility of damage to hearing depends on the efficiency of the amplification mechanism in the human ear. Quite simply, if the eardrum and the little bones can't transmit the vibrations to the cochlea because they don't respond fast enough, those little hairs (cilia) won't get damaged. Where the exact cutoff line is, I dunno. It's very likely that the human ear has evolved to efficiently filter only the frequencies that matter most. I'd speculate that ~20KHz might still be in the danger zone, but I don't know of any detailed mechanical studies on the amplification and coupling factors in that region. I do know that I cover my ears as soon as I hear feedback rising in frequency, since it quickly becomes very painful. Actually, as a former soundman, I would leap for the volume knobs first. Intense high-frequency sound is double-plus-ungood. Franamax (talk) 21:10, 15 April 2009 (UTC)[reply]
Yes on 20KHz not being "imperceptibely high frequencies"--many people can hear in that range quite well. Of course not all loudspeakers are capable of producing sound in that range. Pfly (talk) 05:25, 17 April 2009 (UTC)[reply]

honey

What is the chemical or chemicals in honey that help reduce the pain of a sore throat? 65.121.141.34 (talk) 20:03, 15 April 2009 (UTC)[reply]

I don't think that is known. [7] Rockpocket 20:10, 15 April 2009 (UTC)[reply]
I don't know either - but I suspect it may not be a chemical effect so much as a lubricant. SteveBaker (talk) 20:41, 15 April 2009 (UTC)[reply]
I don't know what has the analgesic effect, but the antiseptic effect is due to the way the high concentration of sugars absorbs water, killing the bacteria. --Tango (talk) 20:43, 15 April 2009 (UTC)[reply]
Anti-inflammatory compounds in Honey. Live long and prosperous \V/ --Grey Geezer 22:15, 15 April 2009 (UTC) —Preceding unsigned comment added by Grey Geezer (talkcontribs)
But avoid non-pasteurized honey or you might encounter these: Botulism. Particularly dangerous for small children. 76.97.245.5 (talk) 08:39, 20 April 2009 (UTC)[reply]

Microbial growth media

What is the correct term for the class of growth media ingredients that includes yeast extract, peptone, beef extract, casamino acids, tryptone, etc?

Microbiological media ingredients? Amino acids? I'm not convinced there is a specific classification. Rockpocket 20:13, 15 April 2009 (UTC)[reply]
Please look here or do you mean Lysogeny broth (Luria-Bertani Broth)? Cheers --Grey Geezer 21:43, 15 April 2009 (UTC) —Preceding unsigned comment added by Grey Geezer (talkcontribs)
Sorry, that doesn't really answer my question at all. ike9898 (talk) 22:20, 15 April 2009 (UTC)[reply]
D'Oh! I overlooked the word ingredients [don't misguide old men by a "poorly formulated title" of your request ;-)]. Coming from the media you would call them something like non-defined (or poorly defined) protein complements. If you search for them in the Sigma-Aldrich Catalogue (they should have an interest to label this "Class" correctly, right?), they call them "Microbiology, Basic Ingredients, Protein Sources", or "Peptones" (which has another more specific definition). In my view (as suggested above): No generally accepted "Class" designation. Better? ;-) --Grey Geezer 07:30, 16 April 2009 (UTC) —Preceding unsigned comment added by Grey Geezer (talkcontribs)
Yeah. The reason I ask is because not too long ago a microbiologist got all bothered when I referred to this sort of thing as a "nitrogen source". I never got the chance to ask her exactly why she thought that was wrong or what term she would prefer. It is possibly because these ingredients do more than just supply nitrogen, they also contain growth factors and minerals, and provide some energy as well. However, I haven't been able find to good single word to describe these things. Maybe I should come up with one (remember Sniglets?). ike9898 (talk) 13:28, 16 April 2009 (UTC)[reply]

Very small creatures in my fish tank?

I have something very small living (uninvited) in my freshwater tropical aquarium. It is circular/oval, white/grey in colour and about 1mm-2mm across in size. It moves very slowly across the surface of the glass or the leaves of the plants, as if it were a tiny mollusc or similar. There are several of them in the tank and the numbers seem to be steady. They don't appear to be damaging the plants although it's possible they are attacking the fish. Every so often a fish appears to have a twitchy fit for about a day and then dies, but this is rare. Is it likely they are harmless or could they be some kind of parasite?Popcorn II (talk) 20:51, 15 April 2009 (UTC)[reply]

Planaria? --Dr Dima (talk) 21:25, 15 April 2009 (UTC)[reply]

Hmmm. Possibly but I don't think so. These things are about 2mm at their largest. Many of them are less than 1mm. They also seem to move as a whole, solid form without stretching, as a Limpet might.Popcorn II (talk) 22:55, 15 April 2009 (UTC)[reply]

Might be freshwater ostracods.--Eriastrum (talk) 23:07, 15 April 2009 (UTC)[reply]

Possibly but they really don't look like Crustacea. There's no detail on them, just a smooth shape.Popcorn II (talk) 12:49, 16 April 2009 (UTC)[reply]

Ostracods are enclosed in a little shell, and that is all you usually see: they look like little beans, usually whitish. The crustacean-looking parts (jointed legs and mouth parts) are enclosed inside.--Eriastrum (talk) 19:38, 16 April 2009 (UTC)[reply]
A photo would be helpful. ;-) Axl ¤ [Talk] 08:36, 17 April 2009 (UTC)[reply]

Biological Function of Grief in Humans

Feelings (affection, humor, fear, even belief etc.) can be associated with "biological functionality". However, what is the "functionality" of grief (e.g. after the loss of a child, partner, parent)? A functional reaction should be "feeling strong" or even "euphoria" to start up again (to find a new partner, have another child etc.). Griefing for a loss (which will not be returned) does not really make sense, right? If the loss is by accident, predator, or disease, the learning effect (how to avoid) is low: the griefing person is depressed, inactive, sometimes suicidal, has no appetite, which should increase the chance for an accident, falling prey to a predator or even getting weak and sick: So exactly the opposite = grief makes things worse. The question is not: How to overcome grief but "what is its function in the big game"? Any suggestions? --Grey Geezer 22:08, 15 April 2009 (UTC) —Preceding unsigned comment added by Grey Geezer (talkcontribs)

It probably has something to do with the fact that we've evolved in tribal settings with small groups of a dozen or two people. We have evolved a sense of community. Just as we've evolved responses such as pain so that we know that when we've been bitten in the leg by a rampaging marmosette we should avoid letting that happen again - so taking a 'wound' to the community is something we need to try to avoid - so we have a different 'pain' response to that. If there were no emotional consequences to losing tribe members ("friends" and "relations") perhaps we'd be less cohesive as a group? SteveBaker (talk) 22:16, 15 April 2009 (UTC)[reply]
Its very beneficial to be part of a group, from a group of two people (couple relationship) to a family/tribe group, to a polical/national group. The emotions try to make you stay part of that group no matter what. These emotions fire even if the group doesn't physically exist anymore. Remember that the whole idea of being in a group is a mental idea - so its sort of an idea organism that isn't going to know if the group physically exists - for most of your life the group will exist and these emotions will keep you in-line with group which is beneficial biologically. Also the reason can be non-biological. There is no biological reason for a person to kill themselves for the group (that doesnt include their offspring), but the idea organism that permeates all minds in the group has alot to gain from someone doing this - its strenghtens the idea of group, and thus the group is likly to expand and beat other group ideas. So basically you need to understand that once we had evolved a mind with certain capabilities, we also had 'ideas' that could evolve, and these virtual ideas can very often beat out the biological requirements of their hosts (us). It is beneficial for the rest of the group (which is really just an idea of what the group is) if you are seen to endure great hardship from leaving or someone else leaving the group. Likewise people are typicalyl very moved if they see someone else who has lost their partner, often it motivates them to form even strong group connections with there still existing partner, this is how an idea (like marraige) can be a duplicate of groups, an idea that copies itself as small groups everywhere. So while its pointless for us to greive, it helps everyone else with the idea of marraiage, and it likely results in the idea of marraige being further conveyed.--Dacium (talk) 03:28, 16 April 2009 (UTC)[reply]
Like other aversive things, its main function is probably preventative: the knowledge (obtained by observation of other people) of how terrible grief is makes you work very hard to avoid it. Looie496 (talk) 05:21, 16 April 2009 (UTC)[reply]
Thanks for the comments! So grief, which is often used as a "personal problem" could be a "group signal" ("Look how valuable members of this group are to me!". And like "Look, I suffer to have all those piercings / brandings done to show that I am really part of your group!") and this signal would be "more worth" than the "temporary inefficiency" of the grieving group member. Makes kind of sense. However, grieving for pets then seems to be a "side effect". It seems that Inuit or other native groups are very pragmatic (more realistic?) with the dogs who guarantee part of their survival (also very important!), e.g. killing off "weaker" female pups or disposing of the corpses of long-time sleigh dogs, but skinning them and using their fur. --Grey Geezer 07:51, 16 April 2009 (UTC) —Preceding unsigned comment added by Grey Geezer (talkcontribs)
Yes, I think that's true. We've "adopted" pets - they are a part of the "tribe" - they are treated like children and grieved for in the same way. When animals are "working animals" they are more like tools than members of the family - we can be more hard-hearted about the whole thing. When your old car is beat up and rusty - you get a new one - you throw the old one out...unless you can salvage something from it. SteveBaker (talk) 12:56, 16 April 2009 (UTC)[reply]

Grieving is a retreat into a passive state when one finds that previously held assumption(s) about one's safety, posessions or relationships have been broken. Its survival value lies in 1) the retreat from action to allow time for, hopefully, the sufferer to evaluate their changed situation and 2) the person's grief signals can bring aid from other(s). Babies use 2) as feeding strategy. Cuddlyable3 (talk) 23:20, 16 April 2009 (UTC)[reply]

Achilles tendon and stairs

I find it easier to climb flights of stairs if i do not fully relax my Achilles tendons after each step. Why would that be? —Preceding unsigned comment added by 79.67.15.85 (talk) 22:36, 15 April 2009 (UTC)[reply]

Tendons (and all soft tissue in general) has elasticity. If you keep the tendon tight, it will stretch a bit when you apply weight to it and release some of that energy to propel you up to the next step. Of course, this only works if you go up the stairs on the balls of your feet, not the heels. -- kainaw 12:14, 16 April 2009 (UTC)[reply]

Ear canal bandpass

What is the bandpass of the human ear canal in Hz? (not homework) —Preceding unsigned comment added by 79.67.15.85 (talk) 22:41, 15 April 2009 (UTC)[reply]

Is this what you're looking for? ArakunemTalk 22:48, 15 April 2009 (UTC)[reply]
Well no. Im looking for the actual transfer function of the human ear canal (including the pinna), but excluding the ear drum, choclea etc —Preceding unsigned comment added by 79.67.15.85 (talk) 22:53, 15 April 2009 (UTC)[reply]
Does this help? Cuddlyable3 (talk) 22:59, 16 April 2009 (UTC)[reply]
This book has what you want. You will also find a lot of papers on the subject if you search for "head related impulse response" or "HRIR" on google scholar. Here are some from the Acoustical Society of America [8][9][10]. There is also a lot published by the Audio Engineering Society and the IEEE, but sorry, I don't know if they are exactly what you want as I can mostly only read the abstracts. SpinningSpark 13:48, 18 April 2009 (UTC)[reply]


April 16

Ionic detox

Recently, a co-worker met with an alternative health practioneer and participated in an "ionic foot bath." He described all of the ickey things coming out of his feet (including what appeared to be "live" worms. Ever the skeptical one, I did some investigating. Here on Wikipedia, the closest thing I could find was an article on detoxification foot pads. Using Google, I found a plethora of info (all promoting ionic foot baths!). So, here is my question: Since there is no scientific evidence to support the claims of ionic foot baths, what is really happening in the wather that makes it look like things (including worms) are being drawn from the body via the soles of the feet? —Preceding unsigned comment added by 209.161.212.185 (talk) 03:35, 16 April 2009 (UTC)[reply]

With a little googling you can find accounts of people who run the "ionic baths" without putting their feat in and the water still turns murky. It's a trick. It's been mentioned on The Amazing Randi's site several times. here's the most recent mention.
The live worms are an interesting touch, but once you acknowledge that it's a hoax, then it's easy to use slight of hand to add some of those tiny worms that you can buy to feed your Betta fish.
In general the idea that there are numerous common "toxins" build up in your body and must be cleansed is new age nonsense. If you've got a healthy liver and a pair of healthy kidneys, most of your toxins are being taken care of. (And, if not, all the foot baths in the world won't save you.) APL (talk) 04:49, 16 April 2009 (UTC)[reply]
Here are some YouTube videos. Here. On closer examination, this seems to be similar technology that is used in saltwater etching(Scroll down), except instead of removing metal from an electrode to create an artistic pattern, you're removing metal from an electrode so that the waste metal will darken the water and create a "toxic"-looking effect.
I'm also less convinced that there were actual live worms, now that I've read about it more. Perhaps tiny particles of metal held together by magnatism or electric charge would move around in a lifelike way? APL (talk) 05:06, 16 April 2009 (UTC)[reply]
There is absolutely no doubt that this (and those damned stupid foot-pads) are a total scam. The adverts for the foot pads actually make me laugh: "Just as the leaves of a tree draw toxins out of the air and expel them through their roots - so the Kinoke foot-pads...yadda, yadda, yadda"...EXCUSE ME?! Since when do trees absorb toxins through their leaves? Trees don't expel ANYTHING through their roots. Actually, they soak up stuff through their roots and expel it (well, water and CO2) through their leaves! And just because a tree does it (or not) why would that make a difference to humans...we don't even have leaves or roots! Gah! Evil bastard scammers with annoying TV ads. Argh! Anyway - once you accept that it's a scam - you have to ask how those things got into the water. I presume that if you were observant and skeptical during the "treatment" you could spot the moment when these things were dropped into the water by the practitioner - perhaps they were inside the 'healing crystals' or whatever else was placed into the water? Perhaps they were concealed inside the water recirculating mechanism (or whatever it uses). Perhaps the scammer 'palms' them (as a magician palms a playing card or a coin) and slips them into the water at some point. For sure there are no worms in your feet! Ask your co-worker to look at his/her feet and see if the holes from which the worms emerged are visible? No? No holes? Did the worms teleport through the soles of the feet? If the government isn't going to enforce trading standards and medical practitioner licensing laws against these people then it's up to everyone to be super-skeptical of any of these kinds of claims. Mostly they don't stand up to any inspection at all. I really wish I had the time to go to some of these places - take the treatment - spot the scam and sue the evildoers for fraud and practicing medicine without a license. We're gradually sliding backwards into the Dark Ages - and with all we know and understand - that's just ridiculous. SteveBaker (talk) 12:51, 16 April 2009 (UTC)[reply]
I don't think that you could "spot the moment when these things were dropped into the water by the practitioner". I'm pretty sure that the murkiness of the water is actually rust, or other material from the electrodes. They are consumed by the "ionic bath" process and need to be replaced on a regular basis. (This adds a nice 'don't sell the razor, sell the blades' aspect to the scam.)
They sell these machines (at about $1000!) for home use, and they produce the same effect when operated by the end user.
Of course, live worms aren't going to come out of an electrode. So if there really were live worms, they must have been added by slight of hand. A bit over the top, if you ask me. If I were working this con I would have just stuck with "toxins". APL (talk) 15:40, 16 April 2009 (UTC)[reply]
Hmmm - perhaps these "worms" are something like dried out Sea-Monkeys that could be inside whatever stuff they sell you to put into the water? Suppose they sold you a large tablet - like the kind you put into your dishwasher - with a capsule in the center that would dissolve and re-animate the dried worms. They'd probably scare the bejeezus out of the gullible 'mark'! But you could probably make sufficiently convincing "worms" out of all manner of inert materials that would expand and look suitably yukky. SteveBaker (talk) 20:36, 16 April 2009 (UTC)produce[reply]
Well, I suppose you could do that, but there'd be some setup involved. SeaMonkey "instant life" packets take about 24 hours to come to life. I can't find any evidence that it's ordinarily done. None of the videos of the devices (Either the credulous ones, or the debunking ones) show anything that looks like a live worm, though the orange muck seems to eventually coalesce into a few 'floaters' that might look alive if you didn't look too close. These machines involve dunking a pair of electrodes into salt water. The electrodes apparently corrode away, so that explains the gunk in the water. If there were live worms involved, then I'll bet they were put in there by the salesman to try to punch-up the effect of the machine and not part of machine's usual scam. If that makes sense. Like a vacuum-cleaner salesman who isn't content with ordinary dust, so he hides some graphite in his vacuum. APL (talk) 23:07, 16 April 2009 (UTC)[reply]
A good rule of thumb: If someone talks about "toxins" but fails to mention what specific substances they mean, you can be 99 and 44/100 % sure they're full of shit. Friday (talk) 15:52, 16 April 2009 (UTC)[reply]
Yeah. Absolutely. SteveBaker (talk) 20:36, 16 April 2009 (UTC)[reply]


Here's a article on the topic. APL (talk) 23:07, 16 April 2009 (UTC)[reply]
Sadly, while that's certainly what's going on...there was no mention of the worms. These scammers are on to a good deal then...they take one of these foot-spa's (Cost: $15.14 at Walmart) - hook up a couple of electrodes (6" nails ought to do it - that's another $1) and a suitable battery a switch and some wires (maybe another $10) - and sell the thing for $1000. They'd be making a spectacular profit at $100 - at $1000 it's like printing money! I could build 50 of these a day by myself with just the tools in my garage. Then they have the nerve to sell the "refill kits" for $44 plus postage...$55 to you! Pulling this kind of scam on the American public is like shooting fish in a barrel. There are times when I really wish I was not such an honest person! SteveBaker (talk) 00:41, 17 April 2009 (UTC)[reply]
Minor and off-topic correction for SteveBaker: trees do expel lots of stuff through their roots, primarily sugars destined for the commensal organisms helping them to thrive. It's a fascinating topic, especially when you think about how easy it should be to cheat a tree. It's even vaguely possible that the tree would acquire and pump toxins into the cheating fungi, though I've never seen a single word to that effect. Totally agree though on the other commentary on this scam, my only regret is that I haven't dreamed one of these puppies up! Franamax (talk) 00:51, 17 April 2009 (UTC)[reply]

Ideal physical attributes for boxing

In boxing, are smaller or larger fists better (assuming they're on the end of the same arm throwing the same punch)? And what would be the ideal build in terms of height, chest size, reach, muscle mass etc and the relationships between those characteristics? Thanks. 86.8.176.85 (talk) 04:49, 16 April 2009 (UTC)[reply]

The fist part of your question is easy, the second one is not. When someone (constant weight = same punch power) steps on your hand with high heels (small fists, high relative impact) or Gramps slippers (large fists, low relative impact)... what hurts more? --Grey Geezer 08:01, 16 April 2009 (UTC) —Preceding unsigned comment added by Grey Geezer (talkcontribs)
Force equals mass times acceleration - so heavier fists (not necessarily larger ones) with the same exact speed of swing will produce a larger amount of force when they hit something. However, Force equals mass times acceleration - so when you throw that punch, a heavier fist requires more force to get it going fast...or to put it another way...for the same amount of musculature, etc - a heavier fist will end up travelling more slowly than a lighter one - and the benefit of that heavier fist will end up cancelling out. Now, admittedly - as User:Grey Geezer says - the same amount of force over a smaller area produces more pressure - and therefore (presumably) more pain...hence the reason one stabs people with a sharp knife and not (say) a brick! As to muscles...the force a muscle can produce is proportional to the cross-sectional area - so if you have two people with the exact same body proportions - but one is twice as tall/wide as the other - then the larger person has EIGHT TIMES the amount of weight to move around - but only FOUR TIMES the amount of muscle force to move it with. This is why elephants move so slowly compared to (say) dogs...and insects so much faster still. So we're back to that same Force=Mass times Acceleration problem - if you are smaller, you can move (relatively) faster - but with lighter fists, you need to be faster in order to produce the same amount of force. But this is all very superficial reasoning - can a larger person train harder? Can they take more punishment? Can a smaller person simply dodge all of the big guy's punches so it's irrelevent how hard the big guy can hit? All in all, it's a very complicated question. SteveBaker (talk) 12:38, 16 April 2009 (UTC)[reply]
Larger fists would appear to give an advantage to blocking65.121.141.34 (talk) 14:53, 16 April 2009 (UTC)[reply]
Yeah - but they're more of a disadvantage when it comes to the running-away-screaming...which is probably higher on my list!  :-) SteveBaker (talk) 20:28, 16 April 2009 (UTC)[reply]
This is as near "ideal" shape for a man as can be designed. Cuddlyable3 (talk) 22:35, 16 April 2009 (UTC)[reply]
Oh...I feel so inadequate. SteveBaker (talk) 00:28, 17 April 2009 (UTC)[reply]

Trolley-bus routes

I noticed that in some cities, trolley-bus routes may be circular or non-symmetrical (e.g., A-B-C and C-D-A even if there are no one-way streets).

Let's say you own a bus company in a small town having a bus terminal (A) and three stations (B, C and D). Let's say your buses are all diesel powered. There can be two routes (there are no one-way streets):

A-B
| |
D-C
  • Route Clockwise: A-B-C-D-A
  • Route Counterclockwise: A-D-C-B-A

Now if your buses are all trolley-buses. Running these two routes requires that you install TWO SETS OF OVERHEAD WIRES for your entire bus routes.

So you may decide to run one route only and double the frequency (e.g., 10 mins a bus -5 mins a bus).

I guess that if the routes are light capacity (e.g., low population density), trolley-bus companies may have their routes covering more roads one way or circular to increase the number of stations or reduce the length of wires. For example:

A
|
B
|\
C D
|/
E
|
F
  • Route A->F: A-B-C-E-F
  • Route F->A: F-E-D-B-A

On second thought, this logic may not be applicable to railroads. You cannot afford to run a railroad system on a single set of tracks design. If a train is derailed, you can do nothing before the derailed cars are removed.

If you have a subway system, you may want to have at least two sets of tracks because it's much more expensive to dig two smaller tunnels than a big one. One bidirectional underground station is also less expensive than two one-way stations -- Toytoy (talk) 05:26, 16 April 2009 (UTC)[reply]

IS MY HYPOTHESIS CORRECT? OR DID I OVERLOOK ANYTHING? -- Toytoy (talk) 07:04, 16 April 2009 (UTC)[reply]

Sorry, but I cannot find any question :( 122.169.127.46 (talk) 05:35, 16 April 2009 (UTC)[reply]

No matter whether you're talking about a trolleybus line, a streetcar line, a subway, or a railway, it's always cheaper to build a single line than a double one. But it also severely constrains your operational flexibility and capacity if you want to operate vehicles in both directions. Loop routes are indeed a possibility for avoiding this issue while building only a single line, but the trouble is that they produce indirect routes, which aren't as good for attracting riders. Passengers want to go more or less directly to their destination, not detour around the sides of a loop. So when transit systems do use one-directional loops, they're typically only small ones, just a few blocks long or wide, usually at the end of a route. Here's an example on a subway line (trains run anticlockwise around the single-track loop), here's one on a streetcar route, and here's one on a trolleybus line, which also includes some other one-way sections due to one-way streets. These examples are in Paris, New Orleans, and Geneva respectively.

Single-track lines are much more common on railways, where distances are long and frequencies may be relatively low, than on urban transit systems such as trolleybuses and streetcars. It is true that a derailment may block the line completely, but derailments are rare (and for that matter, if one happens it may block both tracks of a double-track line). Capacity limits and dealing with ordinary delays are much more of an issue.

It's not always correct that on a subway system "it's much more expensive to dig two smaller tunnels than a big one". It depends on the construction methods. In London, for example, all of the deep-level tube lines do use two separate tunnels for their tracks, and often even for the larger tunnels at stations.

But generally in urban transit, if the frequency of operation on a route is low enough that a single direction is acceptable, it will be operated by buses, which need no tracks or overhead wires. --Anonymous, 08:44 UTC, April 16, 2009.

In terms of digging tunnels - the amount of dirt you have to dig and the amount of tunnel-walls you have to erect to make two identical one-track tunnels is (obviously) twice the amount for a single one-track tunnel. But because tunnels have to be circular, making one two-track tunnel requires removing FOUR TIMES the amount of dirt and erecting twice the amount of wall compared to a single one-track tunnel. So if the cost of removing dirt is high - then making two small tunnels could easily be around half as expensive as one larger tunnel. Of course there are many other issues involved - such as the size and running costs of your tunnelling machines and the amount of surface disruption caused by your activities. SteveBaker (talk) 12:26, 16 April 2009 (UTC)[reply]
Factors that favour dual- over single- tunnels for two tracks include 1) the possibility to close a tunnel temporarily for maintenance while keeping a reduced single-track service, and 2) in case of an emergency such as a derailment, fire or chemical spill, the parallel tunnel can be used for access and escape if cross connections are provided. That arrangement has been tested by a number of fire incidents in the England-France Channel Tunnel. Cuddlyable3 (talk) 22:24, 16 April 2009 (UTC)[reply]
Tunnels certainly do not have to be circular. Some are oval, some have vertical sides and and an arched roof, some are rectangular or close to it, and so on. Again, it depends on the technology (not necessarily a tunnel-boring machine; less automated techniques were common in the past and are still used for short tunnels, and many transit systems use cut-and-cover tunnels) and on what you're tunneling through.
On a subway system, one advantage of double-track tunnels is that since the tracks are close together, they can lead directly into a station that has separate platforms outside the tracks for the two directions of travel, which gives a convenient separation of passenger flows compared to a center "island" platform. Another is that if crossovers between the two tracks (for turnbacks) do not require additional tunneling. Another is that a single train does not restrict the flow of air through the tunnel significantly, which improves ventilation and also reduces air resistance on the trains. On the other hand, when circular tunnels are in use, a double-track tunnel requires more height clearance around other things that may be in the ground, and then there are the issues mentioned by the last poster. --00:00 UTC, April 17, 2009.
Oval tunnels have one major problem though - you can't use a simple tunnel boring machine to make them because the cutter heads go around in a circle and the tunnel walls have to be lined as the TBM inches forwards. Agreed that oval tunnels are fine when you're doing cut-and-cover though...but then the amount of material you have to remove and replace is large compared to the volume of the tunnel anyway. As for the air flow argument - I'm pretty sure I read somewhere that the progress of trains though the London underground system is designed precisely so that it DOES move air through the tunnels - and that saves them from having to pump fresh air around the system. Those trains go so slowly that air resistance probably isn't a large fraction of their running costs. Streamlining trains is less important than (say) cars and trucks anyway because they only pay the price once for the entire length of the train - compared to each car or truck on a freeway having to pay the energy price again and again. Anyway - we're getting quite a way from the OP's question here. SteveBaker (talk) 00:27, 17 April 2009 (UTC)[reply]
Again with the boring machines! (Pun intended.) Cut-and-cover tunnels are usually rectangular. Oval ones were probably excavated using a shield or by blasting.
It's not correct that trains pushing air through the tunnels provides useful ventilation. The builders of the early tube lines in London HOPED it would, but it turned out they were wrong. It is correct that streamlining is generally less important for trains than for cars and trucks moving at similar speed, and is generally less important at low speeds, but the situation is different when the train is blocking a large fraction of a tunnel's area. The newer tube tunnels in London (Victoria and Jubilee Lines) are generally at least 6 inches larger in diameter than the older ones, and air friction is the main reason.
--Anon, 07:25 UTC, April 17, 2009.

concurrent queues

at the bank I got a 369, there was someone at 365 but also 500's and 900's. However the bank teller next to me surprised me by offering to tell me when I would come up -- she told me I would be 8th in line. So if her computer shows all three queues as mixed into a single queue order, why keep them separated from each other? Thank you! 94.27.231.11 (talk) 10:17, 16 April 2009 (UTC)[reply]

I presume that there are multiple ticket-issuing machines with rolls of tickets pre-printed with consecutive numbers. So if you go to the first machine, you maybe get a ticket that starts with a '3' - at the second machine they start with a '5' and at the third machine, they start with a '9'. The ticket machines tell the computer system when each ticket was issued - irrespective of the number printed on it. Alternatively - sometimes these machines are set up so that the number tells what kind of service you want - so perhaps if you needed a foreign currency transaction, you'd get a ticket starting with a '9' - and then only the desks that have tellers who are qualified to do foreign currency work will accept those tickets - where every teller can take '3' tickets because those are for general check cashing, etc. But without knowing how your bank issues those tickets, it's hard to be certain. SteveBaker (talk) 12:19, 16 April 2009 (UTC)[reply]
I've seen systems that intentionally keep the ticket-numbers non-consecutive (one fast-food place was totally random, some skip around) for no reason other than to keep customers from knowing that they have a long wait ahead for a noticeably higher number than a single linear "now serving #..." queue. DMacks (talk) 14:55, 16 April 2009 (UTC)[reply]
It is probably done to prevent anyone knowing that they will be served next, allowing them to hover ready to pounce immediately the preceeding customer leaves. Cuddlyable3 (talk) 22:05, 16 April 2009 (UTC)[reply]
My question here is: why wasn't that helpful teller helping the first customer in the queue? Do they plan this stuff during coffee break? :) Franamax (talk) 00:30, 17 April 2009 (UTC)[reply]

Paradox in current through vacuum

please visit the following post in the refdesk : [[11]] . Here, some of them are saying that vacuum has zero resistivity. So, if the resistance is zero, for any finite voltage V,

I = V/R so for any value of V, the current would be infinite. It means that infinite number of electrons would be passing through the vacuum at a given time. Please note that the vacuum will cease to be a vacuum if electrons are there but nevertheless, the resistivity will not change as the electrons are not moving at random. But how will the universe contain an infinite number of electrons? --harish (talk) 12:43, 16 April 2009 (UTC)[reply]

No paradox. If you'll noted, the discussion that gives rise to "resistance = 0" is actually saying energy dissipation is zero. No energy lost can be equated to no resistance. In practice, this means that any given voltage can be transmitted at any given current, but this is not a case where division by zero equals infinity. 64.238.255.250 (talk) 13:12, 16 April 2009 (UTC)[reply]
What does any number divided by 0 equal? If there is no current, there is no defined resistance. Ohms law applies to a conductor with a voltage across it and current flowing through it, nothing more.--58.111.134.75 (talk) 13:16, 16 April 2009 (UTC)[reply]
Division by zero means that your mathematical model broke down, and you need a better one if you want to know what happens then. — DanielLC 00:58, 17 April 2009 (UTC)[reply]

No, R would be infinite, not zero. Another thing is that there will still be a tunnel current, and Ohm's law is not in generally valid in the first place. Count Iblis (talk) 14:34, 16 April 2009 (UTC)[reply]

One can "shoot" a charge carrier, such as an electron, through a vacuum. That means a voltage (the shooting force) has caused a certain amount of charge to move from A to B. The charge flow per time is the effective current. The vacuum has neither helped nor hindered the transfer. Because of its non-hindrance to current one may declare the vacuum to have zero resistance but because it does not itself provide any current carrying path for an applied voltage one may declare the vacuum to have infinite resistance. Since the vacuum has not participated, no property of the vacuum, such as resistance, can be deduced from the experiment. Cuddlyable3 (talk) 19:27, 16 April 2009 (UTC)[reply]

When an electron strikes the anode in a radio vacuum tube (or similar device), it loses its kinetic energy. The energy will be converted to heat. So there is a power loss.(Large transmitting tubes have heat dissipating fins or may even be water cooled.) Another effect may be secondary emission in which one or more electrons are knocked loose from the anode by the arriving electron. The energy imnparted to the secondary emission electrons will be another power loss. - GlowWoprm. —Preceding unsigned comment added by 98.16.66.104 (talk) 20:57, 16 April 2009 (UTC)[reply]

For those who care, the article Vacuum permittivity is exactly relevent to this discussion. --Jayron32.talk.contribs 01:22, 17 April 2009 (UTC)[reply]
At the risk of sounding alittle dense, I have to ask how is the vacuum permittivity related to the vacuum resistivity? I don't see the connection. Dauto (talk) 18:20, 17 April 2009 (UTC)[reply]

CO2 production vs Absorbtion

I know there are lists of how much CO2 countries emit, but is there a list of how much CO2 is absorbed (mainly by vegetation) per country? The 'land use' seems to be something completely different. I am trying to make a list of which countries are net CO2 emitters and which are net CO2 absorbers and by how much per capita.--58.111.134.75 (talk) 13:00, 16 April 2009 (UTC)[reply]

This is poorly understood at present. Lots of CO2 absorption is into soil, and it's quite difficult to calculate the rate at which that happens across all the different kinds of soil in a region. Looie496 (talk) 17:21, 16 April 2009 (UTC)[reply]
And it will stay poorly understood for a while longer. The OCO satellite mission couldn't get the door open and returned to Earth in a non-optimal fashion. Franamax (talk) 00:20, 17 April 2009 (UTC)[reply]
Isn't it Algae, i.e. the oceans, which do the really big job? All is one. --Grey Geezer 07:34, 17 April 2009 (UTC) —Preceding unsigned comment added by Grey Geezer (talkcontribs)
A stable and healthy eco-system is usually CO2 neutral, i.e. there is no significant net absorption. Most of the CO2 that is consumed by vegetation is released within a short term (weeks to years) when the vegetation rots or is consumed. Only a few geologic formations permanently sequester CO2 by (eventually) turning it into fossil fuels or carbonate rocks over long periods of time. You can draw some CO2 out of the air by planting previously fallow ground, or, to a certain extend, by harvesting plant matter for long-lived products like books and furniture. You might want to take a look at Carbon cycle. --Stephan Schulz (talk) 12:51, 17 April 2009 (UTC)[reply]
GG, you may be thinking of the oceans physically absorbing the CO2, partial pressure dictates that if you put more of a substance into the gas phase of a system, you will also get more in the liquid phase. Thus the problem of ocean acidification.
Certainly though, while SS is correct in stable natural cycles being generally CO2 neutral, there is a net sequestration of carbon to the ocean floor, mostly I believe from diatoms. I'm not sure exactly how large that flux is though. Franamax (talk) 20:15, 17 April 2009 (UTC)[reply]

Solar Energy

How long will it take for solar heat and light to reach Earth?117.204.96.71 (talk) 16:55, 16 April 2009 (UTC) 8sec or 8 mnts?[reply]

The earth-sun distance is roughly 150,000 million m; the speed of light is about 300 million m/s. Divide the two and you get about 500 seconds. DJ Clayworth (talk) 17:20, 16 April 2009 (UTC)[reply]
Yes, that's the time it takes solar heat and light to travel the last 150,000 Gm from the photosphere to the Earth.
However, the sun#Core article mentions that the "photon travel time" for the first 500 Mm from the core where gamma ray photons are produced until that energy exits the photosphere (mostly as visible light photons) is estimated as between 10,000 and 170,000 years. --68.0.124.33 (talk) 04:33, 20 April 2009 (UTC)[reply]

Lyme disease contraction vs. reporting location

CDC doesn't keep records for location of contraction of Lyme disease, only location of diagnosis. Is there any research between the correlation of the two? With the incubation period averaging 10 days, do many people contract the illness (for example, while hiking on vacation) and then travel a significant distance before being diagnosed? Or do most people contract it at or around home? 71.62.175.234 (talk) 18:12, 16 April 2009 (UTC)[reply]

Well, firstly, Lymes disease isn't tough to diagnose - so I would imagine most people are going to their doctors or some very local hospital emergency room to get their diagnoses - so there probably isn't much error in using the location of diagnosis. Secondly - how would the victim KNOW where (or when) they contracted it? The incubation period is anything from days to YEARS! How the heck would the average patient be able to know which of all the places they visited in the last six months was the one where they were infected?
So, I think this explains why the CDC do what they do - and I very much doubt you'll get anything better than that. SteveBaker (talk) 20:25, 16 April 2009 (UTC)[reply]

Trip to Mars

What type of digital resources could I bring for entertainment on a fictional flight to Mars? —Preceding unsigned comment added by 167.102.160.13 (talk) 18:18, 16 April 2009 (UTC)[reply]

Video games? Movies? eBooks? If you weren't worried about copyright, you could pack a large variety of all of those into a rather small, lightweight package. (A solid state hard-drive, for example.) You could also bring an iPod, of course. APL (talk) 18:31, 16 April 2009 (UTC)[reply]
If the flight is fictional, you can bring whatever you want. That's how fiction works. Dauto (talk) 18:33, 16 April 2009 (UTC)[reply]
If you're headed to Mars, I'm not sure you need to worry much about the copyright-holders coming after you:) Are Martians even party to the Berne Convention? Though the MPAM/RIAM might have a thing or two to say about it... DMacks (talk) 19:45, 16 April 2009 (UTC)[reply]
I would expect the game to be laggy if you wanted to play video games online. 65.121.141.34 (talk) 18:46, 16 April 2009 (UTC)[reply]
Teledildonics? Edison (talk) 18:51, 16 April 2009 (UTC)[reply]
Tourist or first class? Clarityfiend (talk) 18:52, 16 April 2009 (UTC)[reply]
You could make yourself dream that you were a cop in 2008 but sent back to 1973. But only if you wanted a really lame ending. DJ Clayworth (talk) 19:08, 16 April 2009 (UTC)[reply]
Did both series have the same lame ending? —Tamfang (talk) 04:01, 22 April 2009 (UTC)[reply]

How digitally savvy are the Martians whom you plan to entertain? Cuddlyable3 (talk) 18:58, 16 April 2009 (UTC)[reply]

Even with todays' technology, it would be quite possible to bring along a snapshot of the entire Internet and every piece of music and every episode of every recent TV show and every movie made in English over (say) the last 20 years...let's toss in every digital book and every videogame ever written too. But even if not - those things could be downloaded via telemetry as-needed - so long as you were prepared to order movies and as you would from a subscription DVD service like NetFlicks. So there is really no need to fill the ship with hard drives just to do that. As '65 says - playing online games with anyone other than your crewmates would be impossible even after just a day or two of travel...the latency would make that impossible...and I think that probably rules out the otherwise excellent teledildonics idea from Edison too. The speed of light is a harsh mistress. Personally, I think the thing the crew would get most benefit from would be access to email and Internet forums. Using those kinds of inherently non-realtime service - you can have close friendships with people even when you are halfway across the Solar System...and I think the biggest stressor for such long duration flights would be having to see the same half dozen faces day after day. But finding things to do that DON'T involve staring at a screen is by far the hardest problem. SteveBaker (talk) 20:19, 16 April 2009 (UTC)[reply]
Not as harsh as one that takes so many minutes to respond! DMacks (talk) 20:43, 16 April 2009 (UTC)[reply]
Apropos a mistress, don't you like analog resources even a little bit? Cuddlyable3 (talk) 21:59, 16 April 2009 (UTC)[reply]
You could do e-mail chess. Not much difficulty with the lag then. 65.121.141.34 (talk) 18:18, 17 April 2009 (UTC)[reply]

We are in a 6000 degree C space cloud?

That's according to [12]. Then why aren't we burning up?128.163.236.55 (talk) 18:40, 16 April 2009 (UTC)[reply]

The same article says the cloud density is only 0.26 atom per cc. Cuddlyable3 (talk) 18:50, 16 April 2009 (UTC)[reply]
Last phrase from the article you linked: "The Local Interstellar Cloud's potential effects on Earth are prevented by the solar wind and the Sun's magnetic field". The cloud is too thin. It pays to read an article all the way to the end. Dauto (talk) 18:53, 16 April 2009 (UTC)[reply]
Temperature is almost meaningless in such a near-vacuum situation. At reasonable gas densities it's a measure of how fast the molecules are rushing around. But when there are hardly any molecules there, the distinction between a large body of very thin gas that's merely travelling along and an actual hot gas, becomes almost non-existant. What you have to consider is the amount of energy contained in that gas - that's a function of it's density and it's temperature - but because it's practically a vaccuum - it's density is all but negligable - so it's not able to transfer significant energy into nearby planets and such. If it did transfer energy into the earth - then the parts of the cloud nearest us would instantly cool down to the same temperature as the earth and because thermal conductivity of such a thin gas is terrible - it would take millennia for any significant amount of additional heat to make it through to us. So the total energy transferred from the cloud to us is far FAR too small to measure. SteveBaker (talk) 20:06, 16 April 2009 (UTC)[reply]
The temperature of the cloud is shown by the shape of the spectrum of its thermal radiation, not the tiny thermal power that reaches us. Cuddlyable3 (talk) 21:54, 16 April 2009 (UTC)[reply]
Temperature is a measure of the average kinetic energy of the molecules of a substance. Basically, its the speed at which those molecules move. One substance can transfer thermal energy to another substance only when molecules of that substance bump into other molecules and make them go faster. So, there are two important things here: the speed the molecules are moving, AND how often those molecules bump into you. A substance which is so diffuse that a molecule only hits you like every few seconds will not efficiently transfer much energy to you. A single molecule striking you at a temperature of 6000 degrees has such a tiny effect on heating you up that you wouldn't even notice.
So, even though those molecules are whizzing around at a super fast speed, there are not enough of them around to effectively raise your temperature much at all. In fact, if you were in this environment, uninsulated, you would radiate heat energy much faster than it could be replaced by the occasional 6000 degree molecule bumping into you. You wouldn't burn up, you'd freeze to death, not because the temperature of the surrounding molecules isn't higher than your base temperature, but because there just aren't enough of them around to keep you warm, regardless of how fast each molecule is moving. --Jayron32.talk.contribs 01:16, 17 April 2009 (UTC)[reply]

What is the name of the quantity that is represented by the energy function of density and temperature? (What Stevebaker mentioned)199.76.189.187 (talk) 23:13, 17 April 2009 (UTC)[reply]

Well, I think that density X temperature (X volume) gives you the total thermal energy of the system, which is a quantity. Density X temperature per unit volume would be specific energy, since volume is not an absolute term (it depends on the relative scale you measure it with, unlike density and temperature which are absolute scales). Or possibly energy density, unlike several others here I'm not an expert!
But you asked why we're not burning up - that's not a quantity, it's a function, namely the heat transfer function. In this case, there are so few of those really hot molecules hitting us to make a difference. We absorb and emit far more energy through other processes, it's there, but it doesn't make us burn up. Generally, I think Jayron said it best. Franamax (talk) 00:34, 19 April 2009 (UTC)[reply]

Ecology book, online source?

My girlfriend is trying to refrence a refrence from an article, and her instructor won't let her use it. I was wondering if anyone could help me find an online or downloadable copy, preferably a PDF, of the journal. The journal is "Perspectives in Plant Ecology, Evolution and Systematics," the author she's referancing is S.J. Wright. Though, this may present a problem, in that, the author, of the journal, may be referencing another author, I just don't know.

I, personally, don't see how it's logical to disavow a referance of a reference; That's all that research articles and science books are nowadays, right? —Preceding unsigned comment added by HitmanNumber86 (talkcontribs) 22:20, 16 April 2009 (UTC)[reply]

Would the title of the article be The myriad consequences of hunting for vertebrates and plants in tropical forests ? That's the only article I can see by SJ Wright in that journal. It looks like you can only get the full article by buying a downaloable copy, which costs $31.50. If your girlfriend is attending a college or similar institution, she may be able to access it for free, if they have a subscription. The college librarian(s) should be able to point her in the right direction.--Kateshortforbob 22:38, 16 April 2009 (UTC)[reply]
Google Scholar is great for stuff like this, because often you can find downloadable versions of things that aren't easily available from the journal web site. This is such a case: here is a pdf version. Looie496 (talk) 23:45, 16 April 2009 (UTC)[reply]
When I studied for my last degree, this sort of thing was counted as academic misconduct: quoting a reference that was not consulted. The idea was to note down the actual document used, not necessarily where that author got their original idea from. Then you are covered if there is misinterpretation or alteration of the original author's writings. And you are not claiming to have read all those references you did not see. Graeme Bartlett (talk) 22:20, 17 April 2009 (UTC)[reply]
Yes, but reviewers often complain if the original source of an idea is not cited -- especially if the reviewer is the original source. I believe it is pretty common to cite papers that have only a peripheral importance based on a general understanding of their findings rather than a close reading. It's a writer's responsibility to cite papers correctly, but this is not something that people frequently obsess over. Looie496 (talk) 16:41, 19 April 2009 (UTC)[reply]
If it really isn't practical to check the original source, the appropriate course is to cite it as "$ORIGINAL, as summarised/translated/represented/etc in $THINGIACTUALLYREAD". This satisfies both honesty and attribution. IME, second-hand citations are horribly unreliable; they should be avoided when possible and otherwise flagged. --GenericBob (talk) 05:26, 20 April 2009 (UTC)[reply]

Reaction product

When I react ascorbic acid and sodium bicarbonate, it yields sodium ascorbate, water, and what? Thanks, Reywas92Talk 23:18, 16 April 2009 (UTC)[reply]

Is this a homework question? Wisdom89 (T / C) 23:23, 16 April 2009 (UTC)[reply]
Just help me, dangit. I've got C6H806 + NaHCO3 -> H2O + the formula for sodium ascorbate + unknown. I'm thinking carbon dioxide, but I'm not sure. Reywas92Talk 23:32, 16 April 2009 (UTC)[reply]
If it's homework, then making you do it yourself is helping you. We can give you help if you're stuck on specific concepts, but we can't just give you the answer. Why are you assuming a 1:1 ratio for the reactants? What atoms do you have left over? Franamax (talk) 00:01, 17 April 2009 (UTC)[reply]
  1. Do the metathesis (or double replacement reaction) just as you would expect to, and you will get two products. Just swap the positive ions between the two compounds, and write the new formulas, balanced for charge of course, and show what you EXPECT to get.
  2. Read this article or, look at the formulas of the two compounds you got doing this the standard way, and compare to what your homework problem SAYS you can get. It will require you to do a little subtraction problem.
If you do these things, you will get the correct answer. If you just berate people who are trying to help you figure out the right answer, you won't get much more help from us... --Jayron32.talk.contribs 01:07, 17 April 2009 (UTC)[reply]

Black Desert

I would like to find/receive information regarding the Black desert in Egypt.--Infonews101 (talk) 23:59, 16 April 2009 (UTC)[reply]

Searching Google for "black desert egypt" turns up quite a few links. Shockingly, we seem to have no article on the Black Desert, though we do have one on the White Desert just next door. Franamax (talk) 00:13, 17 April 2009 (UTC)[reply]
Libyan Desert might also have been useful, but fails to mention it. 76.97.245.5 (talk) 10:49, 20 April 2009 (UTC)[reply]

April 17

Does anyone know of the best free dicom viewer out there? I want one with all of the features of Osirix, but that runs on windows. Specifically I want the following features:

-A GUI interface. -Installed from an .exe file, or other autoinstaller file. -not web based (although if you find a web based one and you can help me set it up then that's fine) -Viewer supports:

 -Regular tools: cine, zoom, magnifier, pan, windowing
 -Measuring
 -ability to compare two or more images
 -ability to colour enhance the images

- 3D mpr and post processing like OSIRIX.

It doesn't have to be open source. It just has to be free of charge. I do not need a pacs server, just a full featured radiology workstation. Please help me. I've searched google for months and come up with nothing except osirix. If there is a windows version of that i'll take it!!!!!

THANK YOU —Preceding unsigned comment added by Pilotbaxter007 (talkcontribs) 00:26, 17 April 2009 (UTC)[reply]

WTF are you screaming at us? I can't help but someone who can help out may decide not to by your ALL CAPS approach. Feel free to edit your question. -hydnjo (talk) 00:57, 17 April 2009 (UTC)[reply]
Much better, thanks. I hope someone provides a satisfactory response. -hydnjo (talk) 01:44, 17 April 2009 (UTC)[reply]

NIH top 40

Where can I find a list of the top 40 NIH funded medical schools?Tuckerekcut (talk) 00:47, 17 April 2009 (UTC)[reply]

Google? --Jayron32.talk.contribs 01:02, 17 April 2009 (UTC)[reply]

Is there another explanation for Black Holes besides being dense matter from which light cannot escape?

Has anyone considered the possibility that a black hole is simply a phenomenon produced by 2 bodies, such as galaxies, moving away from each other at a combined speed that is greater than the speed of light. For instance, our galaxy may "appear" to be a black hole to inhabitants of a neibouring galaxy which is moving in the opposite direction. Acknowledging that nothing can travel faster than the speed of light it is still possible for 2 bodies to be moving away from each other at a almost twice the speed of light. The light from either body would never reach the other thereby making them invisible to each other. Would this not produce a black hole effect?

--UncleJink (talk) 03:50, 17 April 2009 (UTC)[reply]

Actually, even if two galaxies (on different sides of us) are moving away from us at near the speed of light, an observer in either one would still be able to see the other and would not see the other galaxy as moving faster than the speed of light (see this section on relativistic addition of velocities). Additionally, even if there were a galaxy we couldn't see, it wouldn't explain black holes or look like a black hole at all. It would just be something you couldn't see. It wouldn't explain any of the Black hole related phenomena that have been observed. Someguy1221 (talk) 03:56, 17 April 2009 (UTC)[reply]
We do not "see" distant galaxies because the light is red shifted. But the fact that far off galaxies are moving away from us is observed phenomenon, not just theoretical. We can observe galaxies moving away faster than light provided that they are no so far off that their light has not reached us yet. Galaxies which are too far off are beyond our observable universe. But yes, regardless of speed of our and far off galaxy, the speed of light coming from the distant galaxy will be the same. Black Hole is region of space that is within our range of observation and we cannot see light from it because light cannot escape from Black Hole due to very high gravity. It is nothing to do with speed of observer or that of Black Hole. If light not reaching us is criteria for Black Hole, then we can say we are in enclosed inside of a black hole ( which region of non observable universe) manya (talk) 05:03, 17 April 2009 (UTC)[reply]
From special relativity we know that the speed of light is the same in all frames. Starting with two planets moving away from some central point at near light speed in opposite directions, we can change to a frame where one of the planets is stationary. In this frame, any light emitted towards an observer on the stationary planet from the moving planet will still approach the observer at 'c'. So it's still going to get there, you can't out-run light. (Also, as mentioned by Someguy, the moving planet in this frame would not exceed the speed of light itself.) 129.67.117.21 (talk) 12:27, 17 April 2009 (UTC)[reply]
It is not the case that there were many observations of black holes and a theory needed to be developed to explain them. Actually, it is entirely the other way round. Black holes were predicted to exist (or at least the possibility) from the theory of gravity, and then astronomers went out looking for them. Thus there is no question of there being any alternative explanations. Although many candidate black holes have now been identified (Cygnus X-1 being the first) it would be a very rash astrophysicist who would make the claim that their existence is established beyond all doubt. SpinningSpark 13:04, 17 April 2009 (UTC)[reply]
You don't need to change reference frames to understand why your reasoning is flawed. Say you're going to the "right" at 3/4 c and an object to your "left", that's going left at 3/4 c, emits some light toward you. The light goes to the right at c, which is faster than you're going, so it will eventually reach you. In cosmology, though, what you describe is possible, and the effect is something like a black hole turned inside out—the event horizon is still a sphere, but instead of the inside being invisible from the outside, it's the outside that's invisible from the inside (and we're inside). That's still not the same as a black hole, though—there's no way to confuse the two. -- BenRG (talk) 15:00, 17 April 2009 (UTC)[reply]

Coriolis effect and sniping

In the game Call of Duty 4, there is a part where we have to snipe a guy 896 meters away. Our captain says that we should take 3 things into consideration: 1 Variable Humidity 2 Wind speed 3 The Coriolis effect My question is how does variable humidity and the Coriolis effect effect the bullet? You can read it here http://en.wikiquote.org/wiki/COD4#Cpt._MacMillan under Capt. MacMillan. —Preceding unsigned comment added by 116.71.49.171 (talk) 05:32, 17 April 2009 (UTC)[reply]

Wind is obious - it exerts a force on the bullet pushing it away. The coriolis effect is when the earth spins as the bullet is travelling, so by the time it hits the target, the bullet is now offline, even though it travelled perfectly straight, because the earth rotated relative to the bullet. Humidity reduces the density of the air, which will effect the total force of the wind and humidity.--155.144.40.31 (talk) 05:53, 17 April 2009 (UTC)[reply]
I haven't played the game, but I have had military training in long-distance shooting. Everyone has to do up to 500 meters and in special schools we go beyond that. Wind and humidity play a factor. The coriolis effect is brought up, but doesn't matter. The sights are set for distance to the target, taking the actual arc of the bullet into consideration. So, as you "click click click" the distance to the target, you are adjusting for the actual arc of the bullet. The main thing you do is adjust for windage (left and right) and then distance (up and down). If it is extremely humid, you add distance. Then, breath, relax, aim, steady, squeeze. In Call of Duty, do you have to adjust your sight left/right and up/down or do they have you just winging it by aiming a little up and a little off to the side? -- kainaw 06:33, 17 April 2009 (UTC)[reply]
I don't know about rifles, but the computerized control systems on long distance artillery (such as naval guns) do routinely make Coriolis corrections. Dragons flight (talk) 06:43, 17 April 2009 (UTC)[reply]
You don't adjust the sights in CoD4, you just aim a little off to the side. (Also, they don't really simulate the coriolis effect there at all, that's just patter. Ditto for the humidity. What matters is the wind (which is indicated by a flag nearby, so you can tell what direction it's blowing in, and how hard); the rest of it is just there to bring atmosphere to the scene. It does the trick, too.) -- Captain Disdain (talk) 10:23, 17 April 2009 (UTC)[reply]
Setting the distance isn't enough to account for the coriolis effect - you need to know what direction you are firing in as well. The maximum acceleration due to coriolis effect is velocity*angular momentum (but rotated 90 degrees). If we take a muzzle velocity of 1000m/s (and assume it doesn't slow down during travel, which isn't realistic, but nevermind) and the mean angular velocity of the Earth (from Earth's rotation) of 7.2921150 × 10−5 radians per second, we get a coriolis acceleration of about 0.07m/s2. If the target was 1000m away, the bullet would take 1s to get there, resulting in a deflection of 35mm. That's a noticeable amount (although it's an overestimate in most cases), but probably far less significant than the wind. --Tango (talk) 11:15, 17 April 2009 (UTC)[reply]
You also need to know your latitude. Algebraist 11:38, 17 April 2009 (UTC)[reply]
The Coriolis effect does not exist when firing due east or due west. It has maximum effect when firing due north or due south. It has intermediate effect when firing at directions in between. – GlowWorm.
GlowWorm, that's not right. To eliminate the coriolis effect you would have to shoot paralel to the earth's axis. That would be firing along the north-south direction with an upward (or downward) inclination identical to the local latitude. If you forget about the vertical component of the coriolis force (which gets masked by gravity anyways), the coriolis effect is identical in all horizontal directions. Dauto (talk) 19:02, 17 April 2009 (UTC)[reply]
I don't see that, Dauto. If a gun is north of the equator and fires due north at a target due north of it, the gun and the target are both on the same line of longitude. During the flight of the shell the target moves to the east. So does the line of longitude. So the shell will land west of the target. This is tricky to think about. Am I wrong? – GlowWorm. —Preceding unsigned comment added by 98.16.66.104 (talk) 01:23, 18 April 2009 (UTC)[reply]
Yes, you are wrong. In the nothern hemisphere the coriolis force will deflect the bullet towards its right. If you shoot towards the north the deflection will be eastward. That happens because as it moves towards the north it is actually approachingthe earth's axis. from the equation for the speed of an object in circular motion we see that the bullet is actually moving eastward faster than the target (larger ) and that's why it appears to be deflected eastward. Dauto (talk) 05:11, 18 April 2009 (UTC)[reply]
Wouldn't it also have an effect on an east-west shot? Not because it's going off target to one side or the other but because the bullet is traveling slower or faster relative to the target. So if you're shooting eastward, the target is coming at the bullet as the bullet is moving towards the target. Therefore, you don't need to aim as high. Right? Dismas|(talk) 05:19, 18 April 2009 (UTC)[reply]
Yes, there's a second effect but your description of it is also incorrect. Due to the earth's rotation, in a referential attached to the earth's surface there is a centrifugal force. We don't notice that force because it gets included into our perception of the gravitational force as the earth itself deforms under the influence of that force becoming a little flat (Yes a small fraction of what we perceive as gravity is actually centrifugal force). If you now shoot a bullet eastward, that bullet will be rotating around the earth's axis faster than objects attached to the earth's surface, creating an extra centrifugal force. That extra centrifugal force is not included in the local gravity force and must be perceived as a new force pushing the bullet away from the earth's axis. This force has both a vertical component upwards (that is an effect in the opposite direction to the one you incorrectly described) as well as a horizontal component southward. If you shoot the bullet westwards, you get the opposite effect and end up having an extra force with components downward and northward. This effect I describe now together with the one I described earlier are included under one umbrella as the coriolis effect. If you ignore the vertical component of the coriolis effect, you will see that between the two effects there is always a force towards the right of the direction of the movement of the bullet (in the nothern hemisphere). Moreover, this horizontal component of the coriolis force turn out to have the same magnitude independently of which direction you shoot the bullet. Neat uh? Dauto (talk) 06:40, 18 April 2009 (UTC)[reply]
The vertical component of the Coriolis effect is also known as the Eötvös effect. There is a discussion of various factors affecting the ballistics of projectiles in our article on external ballistics. Gandalf61 (talk) 11:32, 18 April 2009 (UTC)[reply]

How can dumb electrons be so clever?

The behavious of things like for example electrons are described by complicated maths formulas, that require a high intelligence and a good education to understand. So how come a little electron knows how to behave? They cannot have computers hidden away inside them. 78.146.249.32 (talk) 11:12, 17 April 2009 (UTC)[reply]

They don't have to think how to behave. Their behavior is guided automatically according to the laws of physics, which we are still trying to decipher. It is like we have to calculate with what velocity and initial angle an object has to be launched to reach a certain distance according to the laws of physics, but when the object is actually launched, it (the object itself) doesn't calculate in the beginning of launch how far it has to go. It has no other choice but to reach that distance because it has no control over itself, it cannot think and react. It has to obey the laws of physics. For the projectile motion, those laws are simple. For describing electrons, they become complicated and require sophisticated mathematics and high intelligence. But the analogy still holds and electron having no other choice and no control over itself has to behave the way the laws of nature demand it to. - DSachan (talk) 11:22, 17 April 2009 (UTC)[reply]
And what's the problem with saying that? Your question is like asking 'when I let go of a stone, how does it know it's supposed to fall?' It just do. Dauto (talk) 14:05, 17 April 2009 (UTC)[reply]

Thats like saying "they just do". 78.146.249.32 (talk) 12:03, 17 April 2009 (UTC)[reply]

Yes, that's exactly what he meant. You've got two alternative explanations here. 1) They just do 2) They were designed that way by someone or something that created them. But whichever it is, electrons don't need to be intelligent to do this, any more than a rock needs to be intelligent to know that it should roll downhill. DJ Clayworth (talk) 14:01, 17 April 2009 (UTC)[reply]
Actually, the behavior of an individual electron is not all that complicated, the problem with the calculation of their movement comes in when you have many subatomic particles acting together and influencing each other in a non-linear fashion. Truthforitsownsake (talk) 14:08, 17 April 2009 (UTC)[reply]
Some very complex-appearing behavior can arise from very simple rules. For example, Conway's Game of Life consists solely of rules simple enough that an 8-year-old can follow them. Yet from these rules you can get crazily complex behavior which would be difficult for people to explain. That's not unique to the GoL. Other simple rulesets show similarly complicated results (see Cellular automaton). Now, I'm not claiming that the behavior of electrons is the result of emergent behavior: what I'm saying is that electrons are following what are to them very simple rules (e.g. "move to a lower energy state"), but because of how these rules are constructed and how they interact with the environment, the overall behavior of the electrons appears very complex to us and requires complex math to represent because we don't "see" the world the same way an electron does. -- 128.104.112.117 (talk) 23:47, 17 April 2009 (UTC)[reply]
Also, the electron doesn't "know" how to behave. The OP is putting the cart before the horse here. We invent mathematical equations to model the behavior of electrons. But the equations don't tell the electron how to behave, they tell US how the electron will behave. The electron would continue to do what it did even if we didn't try to figure it out... --Jayron32.talk.contribs 02:19, 18 April 2009 (UTC)[reply]
As for the cellular automata, you might also be interested in Calculating Space and digital physics. These are theories that describe our universe as similar to a cellular automaton and thereby try to explain your problem of the intelligent dumb matter. 84.174.85.135 (talk) 11:04, 18 April 2009 (UTC)[reply]
78.146, how do you behave at any given moment? You have a set of forces pulling you through life and at each instant you respond to those forces. Sometimes certain forces are stronger than others. Sometimes you do dumb things (so there goes your argument of having intelligence) - but you do those dumb things because local forces caused you to act that way. Other times you do smart things - but only because other forces drew you that way.
You are an electron, and an electron is you. You both respond to a complex set of inputs and neither of you have a "computer" (well, you have a brain, but it uses electrons to act). On a semi-related topic, see The Tao of Physics. Franamax (talk) 23:46, 18 April 2009 (UTC)[reply]

bodybuilding

hi everybody and thanks for reading this .. my question is : what are the negative results I can get from bodybuilding ? Does it have any negative effects on hormones and sperms ? —Preceding unsigned comment added by Sha9law0 (talkcontribs) 11:43, 17 April 2009 (UTC)[reply]

You should talk to an expert about that, rather than random people on the internet. If you are a member of a gym, they will probably have personal trainers that can advise you. We have an article, Overtraining, which might answer some of your questions, but don't rely on us when your health is at risk. --Tango (talk) 12:01, 17 April 2009 (UTC)[reply]
A normal and healthy human can do heavy exercise -- including bodybuilding work -- with no adverse effect on hormones or sperms. Unfortunately, many professional-level bodybuilders have cheated with anabolic steroids, which can have serious negative effects on both hormones and sperms. --Sean 14:22, 17 April 2009 (UTC)[reply]
I'd like to see some sources if you're going to make such claims. I don't deny of course that anabolic steroids seriously screw up your body and are one of the reasons many body builders suffer serious adverse effects. However while the human body is very good at compensating, it seems a fairly extreme claim to me that there will definitely no adverse effects on hormones or sperm from body building. Presuming we're talking about quite hefty body building, the exercise and diet can itself have some fairly major effects and it seems unlikely to me that these will have no effect (positive or negative) on your hormones or sperm (or for that matter other bodily functions). This doesn't mean I believe your going to go infertile from body building, simply that there's a difference between saying it will have no adverse effects and saying that it's unlikely to have extremely detrimental effects Nil Einne (talk) 22:23, 17 April 2009 (UTC)[reply]
I agree that I should have qualified what I said. I was thinking of a sensible amount of bodybuilding, but I agree that an absurd amount of bodybuilding could very well affect the sperms, for some definitions of "sensible" and "absurd". --Sean 22:47, 18 April 2009 (UTC)[reply]
Consult an exercise physiologist. Wisdom89 (T / C) 16:46, 17 April 2009 (UTC)[reply]
If you take steroids, obviously there are tremendous negative effects. If you don't, the major negative effects probably come from the huge amounts of food you have to eat to do top-level bodybuilding, and the heavy weighting of the food toward protein. There's also a problem in that bodybuilders are sometimes so musclebound that they have difficulty doing ordinary activities. Looie496 (talk) 18:19, 17 April 2009 (UTC)[reply]
You should also consider what else you would be doing with all your time that you would be spending in the gym. Will it stop you working or ruin your social life, or stop you having time to edit Wikipedia? There are also injuries resulting from improper use of equipment. Graeme Bartlett (talk) 22:01, 17 April 2009 (UTC)[reply]

Questions on cell size and multicellularity

What limits the size of a living cell? Why don't we see (for e.g.) cells the size of a dog?

Do prokaryotic cells show multicellularity? I know they form colonies, but do individual cells show division of labor/specialization? If not, why not? —Preceding unsigned comment added by 121.241.167.100 (talk) 12:56, 17 April 2009 (UTC)[reply]

One thing that limits cell size is the way that cells function. Cells require a large surface area to volume ratio. Now while technically, you could still have very massive cells with the same surface area to volume ratio, the structure would be such that the cell would pull itself apart from gravity. 65.121.141.34 (talk) 13:25, 17 April 2009 (UTC)[reply]
¡Grande!
File:TAQUITO CHIHUAHUA.jpg
¡Pequeño!
Cells can get bigger than a dog! A Hypselosaurus egg (which, assuming it's like modern eggs, is a single cell) could reach a foot in length, larger than the dog at right. You could probably find an ostrich egg bigger than some dogs. And even the biggest dog has a nerve cell that runs from the tip of its tail to its brain, which is pretty long, if not "big". Also, see the discussion regarding division of labor here. --Sean 14:38, 17 April 2009 (UTC)[reply]
I do not believe that large eggs are just one cell. I think thats a myth. For one thing, they would have terrible problems with the celluar equivalent of respiration, as their volume to surface area ratio would be very much different from a normal cell. 78.149.194.28 (talk) 18:26, 17 April 2009 (UTC)[reply]
To clarify the common misconception. Ova, that is the female gamete colloquially known as "eggs" are a single cell. Actual eggs, certainly are NOT. No ova I know of are larger than microscopic for any species. The largest cells are still likely nerve cells, as mentioned, there have been some documented to be several feet long; even given their thinness, that still makes them quite sizable. --Jayron32.talk.contribs 19:34, 17 April 2009 (UTC)[reply]
That's interesting, Jayron32. So, you're saying that our article, which you cited, propagates this misconception in saying, "The ovum is the largest cell in the human body, typically visible to the naked eye without the aid of a microscope or other magnification device". Maybe you could clarify? --Scray (talk) 20:40, 17 April 2009 (UTC)[reply]
Also, from our Egg (biology) article, "The 1.5 kg ostrich egg contains the largest existing single cell currently known...". Seems we really have a contradiction here. --Scray (talk) 20:44, 17 April 2009 (UTC)[reply]
Yeah, OK, so the human ova is a bit larger than microscopic. But really, only a bit. And the canard that an egg is a single cell misrepresnts what a cell is. It doesn't have organelles, it doesn't contain a cell nucleus or anything like that. Its basically a big sac of amniotic fluid and surrounding a smaller sac of nutrients to provide fuel for the growing embryo. Our article is actually probably wrong here. It calls avian and reptilian eggs "zygotes" That makes no sense. If the zygote develops into the embryo, then why is the egg still here?!? So yes, I will buy that the ovum is the largest cell in the body. But a chicken egg is still not a "cell", unless you redefine what a cell is to mean "everything that a cell is, and eggs too". --Jayron32.talk.contribs 02:15, 18 April 2009 (UTC)[reply]
So are you saying you believe that an egg lacks organelles and a nucleus? --Scray (talk) 03:48, 18 April 2009 (UTC)[reply]
I just re-read your comment, and realize that you're specifically referring to the avian/reptilian yolk and albumin as lacking a nucleus and organelles. All right, I'll agree with that as a generalization. BTW, "ova" is plural, "ovum" is singular. --Scray (talk) 03:57, 18 April 2009 (UTC)[reply]
Another note: Caulerpa may be the largest cell by surface area; it's multinucleated. By longest dimension, the largest cell may be the neuron of the giant squid. --Scray (talk) 04:19, 18 April 2009 (UTC)[reply]
One more note: the Squid giant axon (not specific to giant squid - it's just a giant axon) can be 1 mm in diameter - a diameter easily visible to the naked eye; with a length that can be measured in meters, this is a huge cell. --Scray (talk) 16:40, 18 April 2009 (UTC)[reply]
The article correctly states not that an egg is a single cell, but instead it states that the ostrich egg contains the largest known single cell. So, tell me Jayron, If the yolk of the egg is not that cell, where's the cell? Dauto (talk) 04:55, 18 April 2009 (UTC)[reply]
To clarify I interpreted big as weight, and dog as a standard medium sized dog. 65.121.141.34 (talk) 16:08, 17 April 2009 (UTC)[reply]
As to division of labour, there is the awesome dictyostelium - but it's a eukaryote. :( As far as the "why not" bit goes, I'm not sure that lack of a nuclear membrane is the defining factor in how cell specialization works, it comes down in the end to appropriate communication. Perhaps prokaryotes don't adopt this strategy because they've never had to? There are many paths to evolutionary success. I can't think of any definitive (published) reason why a prokaryote couldn't do the same thing. Interesting question though - there's my whole afternoon shot now! :) Franamax (talk) 20:38, 17 April 2009 (UTC)[reply]
Myxobacteria form "swarms" that travel together. This is the closest that prokaryotes come to "division of labour". Axl ¤ [Talk] 17:30, 18 April 2009 (UTC)[reply]
From McGraw-Hill's Encyclopedia of Science & Technology: "The largest known single cells of a living animal are the mature ova of the ostrich and the shark Chlamydoselache, which are about 3 in. (8 cm) in diameter." Axl ¤ [Talk]

Bullet into wood

Suppose I shoot a bullet into a block of wood, knowing the velocity and mass of that bullet. How could I estimate the final resting position of the bullet? What other information would I need to know? 58.6.129.205 (talk) 13:42, 17 April 2009 (UTC)[reply]

I don't know the exact answer to your question, I'll leave that to our resident physicists, but I think you'd need to know the distance of the wood from the initial position of the bullet (in the gun), and the strength of the wood involved. Both these factors would have a large role to play in how far the bullet can penetrate the wood. Regards, --—Cyclonenim | Chat  13:54, 17 April 2009 (UTC)[reply]
(ec)Our article on Penetration (weapons) sadly doesn't provide any information. This webpage [13] gives an equation by the 19th century French engineer Jean-Victor Poncelet, and a calculator to plug your own numbers into, which may be of use (Note that one of the users of the page spotted an error in the Java script for the calculator, hard to tell if it's been fixed). I presume that the various constants quoted would vary depending on the type of wood, orientation of the grain etc. Mikenorton (talk) 14:05, 17 April 2009 (UTC)[reply]
Looking again at those constants, C0 looks like compressive strength and C1 must be the density of the material. Mikenorton (talk) 14:25, 17 April 2009 (UTC)[reply]
One questionably-useful data point from personal experience. A plain old lead round-nose .22LR round will penetrate a 2x4 of plain old lumber (I think probably pine, but I'm not sure.) However a 2x4 of weather treated wood will stop the same bullet. Clearly, the density and strength of the wood is a factor. Also consider bullet shape and cross-section. A smaller, more pointed bullet will penetrate most materials better than a bigger, blunter bullet. This sounds like one for Mythbusters- I imagine trying to estimate this is more complicated than simply performing some tests. Friday (talk) 14:09, 17 April 2009 (UTC)[reply]
PS. (I mean penetrate the short way, not the long way. I don't remember trying it the other direction.) And the poster above brings up a good point about grain orientation- if a bullet can slip in between things, this is going to take less energy out of it than having to blast through things. Friday (talk) 14:11, 17 April 2009 (UTC)[reply]
Pressure-treated lumber is often a lot wetter than non-PT stuff, which I imagine accounts for the difference you saw. --Sean 14:48, 17 April 2009 (UTC)[reply]
My dad said (WP:OR) that a 22 long would go an inch and a half into the end of a 2x4, farther than I would have supposed. I know that a 22 LR will penetrate seasoned 1x lumber (something over 3/4 inch thick: very old boards) and richochet around inside the building a couple of times, from shooting at an unoccupied farm outbuilding I owned, so it did not have the velocity to penetrate the 1x board on the far side of the building. Milage may vary depending on muzzle velocity, distance to target, mass and jacketing of bullet, and type of wood, grain orientation,angle of impact, and seasoning of the wood. Snipers, assassins, CSIs and police must have some empirical data. Edison (talk) 16:06, 17 April 2009 (UTC)[reply]

Paradox

One of my friends asked me this, and i just can't figure it out. Somebody please help me!! Proof that unicorns exist
For me to prove that unicorns exist, it is enough if i prove that there exists at least a single existing unicorn(possibly a more stronger statement). so there are 2 cases

  1. There exists an existing unicorn
  2. There exists no existing unicorn

But how can an existing unicorn not exist ? So statement 2. is false, so statement one has to be true. Therefore, unicorns exist. —Preceding unsigned comment added by Rkr1991 (talkcontribs) 15:19, 17 April 2009 (UTC)[reply]

I don't think there's a paradox so much as a silly phrase. What is the distinction between "unicorn" and "existing unicorn"? If you strip that word, then the choices of "at least one unicorn exists" and "no unicorns exist" is quite clear. In this specific case, it looks like "existing" is used to mean living. Again, though, swap the word and there's no contradiction. — Lomn 15:25, 17 April 2009 (UTC)[reply]
You almost convinced me! Nice. Isn't the joke based on a shift from "there exists no existing unicorn" to "there exists a non-existing unicorn", that is not exaclty the same statement. pma (talk) 15:42, 17 April 2009 (UTC)[reply]
Listen here (for your friend). A contradictory object does not exist, but also, any object that does not exist may well have a contradictory property: this can be harmlessly assumed, for it does not exist. A contradictory property for a non-existent object is, of course, existence. Therefore a non-existent object enjoys the property of existence, so it exists pma (talk) 16:23, 17 April 2009 (UTC)[reply]
Assuming you don't just have a fetish for horses with strange frontal growths, you chose to discuss unicorns because they are objects which by definition do not exist. Any proposition in which they exist is wrong before it leaves the unicorn paddock. --Sean 16:42, 17 April 2009 (UTC)[reply]
Take it from me. If unicorns existed, my wife would own one. Paradox resolved.--UncleJink (talk) 16:43, 17 April 2009 (UTC)[reply]
A silly play with words does not a paradox make. Said that, who is to say that unicorns do not exist? Any animal with one horn fits the bill. Dauto (talk) 17:19, 17 April 2009 (UTC)[reply]
It's pretty nonsensical. "Existing" is just an adjective - it's like "Blue" or "Pointy" - if I put the phrase "existing unicorn" in quotes then either there is an "existing unicorn" or there is no "existing unicorn" - which is clearly a true statement - just like either there is a "blue unicorn" or there is no "blue unicorn". But in any case - even with your interpretation - the two sentences are only "obviously true" when you interpret "existing" like you'd say "blue". Otherwise I can make paradoxes all day long by saying things like:
Either:
  1. There are pink unicorns ...or...
  2. There are green dragons
That's not a paradox - it's a set of statements that are simply false. So if you insist on this rather crazy interpretation of "existing unicorn" then you simply invalidated the entire statement by turning it into "Either unicorns exist or unicorns exist"...which is false. SteveBaker (talk) 17:41, 17 April 2009 (UTC)[reply]

This sounds a little like a play on the Black Swan Problem? Though I could be wrong as i don't fully understand why their statement is supposed to be a paradox. I could sort of read it similar to the Rumsfeld statement about there being known knowns and known unknowns, and unknown unknowns? (http://www.brainyquote.com/quotes/quotes/d/donaldrums148142.html). But at least Rumfeld's made a bit of sense. 17:55, 17 April 2009 (UTC)

I would say this is clearly a play on the ontological argument for the existence of God. It runs: Define God as a being that is best in every way. Clearly it is better for God to exist than not to exist. Therefore God exists. Vast tracts of philosophy have been written to argue about the validity of this argument. Looie496 (talk) 18:14, 17 April 2009 (UTC)[reply]
Me too, I thought to the logic-ontologic trick, but in fact it's not exactly the same... It seems to me that this one is more linguistic than logic.--pma (talk) 20:02, 17 April 2009 (UTC) As to Rumsfeld I thi(self censored).[reply]
Rumsfeld was a monst(self censored) but I think he made an very perceptive point and got screwed over for it. There really are known-unkowns and unknown-unknowns! Now, the context of his pontification, that was incredibly (self censored) and dismissive of (self-censored) and repugnant to every (self-censored). I do appreciate that contribution to world thought though, flawed as its presentation may have been. Franamax (talk) 20:55, 17 April 2009 (UTC)[reply]
To offer a poor paraphrasing of Kant: "existence is not a predicate". There's a discussion of this kind of argument in a religious setting here[14]. 163.1.176.253 (talk) 23:17, 17 April 2009 (UTC)[reply]
Sorry, of course this has been mentioned before by Looie496. Right here. 163.1.176.253 (talk) 23:26, 17 April 2009 (UTC)[reply]
This is discussed in one of the Smullyan books but I'm too lazy to look up which one even though I have them at home. Sorry. – b_jonas 18:30, 18 April 2009 (UTC)[reply]

positron behavior

dear sir

what will happen when we put a positron in an electrostatic field?

does positron behave like an electron or as proton?

if we put an electron and positron together. do they attract each other or repel?

please give confirm answer.

thank you

anant ranjan —Preceding unsigned comment added by Ranjan.anant (talkcontribs) 18:08, 17 April 2009 (UTC)[reply]

Positrons have positive electric charge (hence the name) and will therefore attract electrons. Dauto (talk) 18:15, 17 April 2009 (UTC)[reply]
In an electrostatic field, a positron will not move in the same way as either an electron or a proton. It will feel the same force as a proton, but because it's a lot lighter than a proton, it will move along a different path. Looie496 (talk) 18:59, 17 April 2009 (UTC)[reply]
And as the antiparticle counterpart to an electron, a positron will annihilate when it encounters an electron (as does the electron). So you get a (comparatively) big boom. — Lomn 19:01, 17 April 2009 (UTC)[reply]

Super long leg hair

File:SmartseLeg hair.jpg
A close up shot of the hair in question with normal hairs for comparison

I have one 10cm long leg hair (not quite a world record, see here) why has this one hair grown so long? It's a lot thinner and whiter than my other leg hairs. Any ideas? Smartse (talk) 19:26, 17 April 2009 (UTC)[reply]

It's possible that it was ingrown at one point. In order to pierce the skin, it added more keratin.130.127.99.54 (talk) 20:11, 17 April 2009 (UTC)[reply]
I've added a photo in the hope it helps. Smartse (talk) 16:29, 18 April 2009 (UTC)[reply]
I've got a very fine, long, white hair on the side of my forehead, which I doubt was ingrown at any time. It grows back after plucking. --Kjoonlee 17:13, 18 April 2009 (UTC)[reply]

Redox reactions and Lewis Acids & Bases

If an oxidizing agent gains electrons, does that make it a Lewis Acid? Similarly, if a Lewis Base donates an electrons, is it also a reducing agent?130.127.99.54 (talk) 20:16, 17 April 2009 (UTC)[reply]

Not really. A Lewis Acid-Base reaction is a covalent bond formation reaction. And its a specific kind of covalent bond formation reaction. Remember that a chemical bond consists of two electrons. Under most reactions, one usually thinks of each atom on either side of the bond donating one electron to make a two-electron bond. Kind like this:
X• + •Y → X—Y
However, in a Lewis Acid-Base reaction, in the formation of the new bond, BOTH electrons come from one of the reactants. Thus, a Lewis Acid-Base reaction always looks like this:
A + :B → A—B
In this case, the A is the "acid" and the B is the "base". A Bronsted-Lowery acid-base reaction is thus a special case of the Lewis reaction, where A is ALWAYS the H+ ion. But the more general Lewis Acid-Base reaction encompases a whole lot of other scenarios, such as the following reaction:
F3B + :NCl3 → F3B—NCl3
In this case, the boron compound is "electron deficient", that is it has an empty orbital, or if you prefer, less than a full octet, which makes it an ideal "lewis acid". The nitrogen compound has a spare, unbonded pair of electrons, which makes it an ideal "lewis base". The thing about this is, unlike other acid-base theories, Lewis theory doesn't even deal with "H+" ions at all.
Now, redox reactions involve the transfer of electrons from one element to another, without necessarily forming any new bonds. For example, if you add say a piece of magnesium metal to a solution containing Copper (II) ions, you will get the following redox reaction to occur spontaneously, forming magnesium ions and copper metal:
Mg(s) + Cu2+(aq) → Mg2+(aq) + Cu(s)
No new covalent bonds have formed. What happened here is that two electrons were transfered from the magnesium to the copper. So you see, though both types of reactions involve the rearangement of electrons in some way, they are very different sorts of reactions. Actually, EVERY chemical reaction involves the rearrangement of electrons in some way, so these two types of reactions are not all that unique in that regard. I hope this makes a little more sense. You may want to read Lewis acids and bases and Redox for more on these processes in general. --Jayron32.talk.contribs 02:07, 18 April 2009 (UTC)[reply]

Nephilim

Is this real or is it photoshopped?

http://www.luisescobarblog.com/wp-content/uploads/2008/06/nephilim-skeleton.jpg

--Threebears2000 (talk) 23:22, 17 April 2009 (UTC)[reply]

It's a well-known fake. Looie496 (talk) 23:48, 17 April 2009 (UTC)[reply]
And a poorly done one too. Look at where the light is falling on the skull - only the front part is bright, the top and sides are in shadow. Now look at the shadow of the guy right next to it - the sun is falling on his back - so the skull should be brightly lit from it's right side...and it's not. SteveBaker (talk) 20:04, 19 April 2009 (UTC)[reply]

April 18

Coriolis force on a river in the northern hemisphere

"A straight river flows with speed v in a direction α degrees East of North. Show that the effect of the Coriolis force is to undermine the right bank. Does the magnitude of the effect depend on α?"

My brain has shut off, I don't even know where to start! Thanks for any help,

Mathmos6 (talk) 00:15, 18 April 2009 (UTC)[reply]

This seem's to be a homework. Is that the case? I hope it isn't because the statement "the effect of the Coriolis force is to undermine the right bank." is actually incorrect. Dauto (talk) 01:01, 18 April 2009 (UTC)[reply]

WP:DYOH Smartse (talk) 01:16, 18 April 2009 (UTC)[reply]

Oh dear, it is part of my set work, yes! Perhaps that explains why I've spent so long trying to understand why and getting nowhere - what does happen then? I've read through the wiki article on the effect but I'm still not completely certain, gah! Mathmos6 (talk) 01:41, 18 April 2009 (UTC)[reply]

I think it should be true in the northern hemisphere. The rotation of the earth about its axis gives the water angular momentum. As the water moves north, it gets closer to the axis, so conservation of angular momentum means it tries to rotate more rapidly about the axis, like an ice skater drawing in her arms to spin -- this pushes it against the right bank. Looie496 (talk) 01:51, 18 April 2009 (UTC)[reply]

Okay, so assuming this is talking about the northern hemisphere, does the angle east of north affect the magnitude? Or equivalently, I suppose, will you have a different effect if the magnitude of α is 0/90 degrees? You should have no effect along the hemisphere and 100% of the effect northwards, right? Mathmos6 (talk) 01:58, 18 April 2009 (UTC)[reply]

The effect doesn't depend on the angle . The coriolis force will indeed push the water rightward. The problem with the statement is that this rightward push will very lightly raise the water level on the rightside but that does not undermine the riverbank. To undermine the riverbank you would need an effect that speeds up the flow of the river on its right side. Coriolis effect doesn't do that. Dauto (talk) 02:10, 18 April 2009 (UTC)[reply]

Actually, a flowing river, being a fluid, would also be deflected rightwards as it flows. I am not certain the effect would be great enough to cause undermining of the river bank, but it may do things in the large scale like cause subtle differences in the overall way rivers flow across open plains and the like... --Jayron32.talk.contribs 21:31, 18 April 2009 (UTC)[reply]
The effect makes the river level rise slightly on its right side. That doesn't undermine the river bank. Dauto (talk) 19:51, 19 April 2009 (UTC)[reply]

Recycling efficiency - plastics vs steel

Plastics & steel are 2 key components of modern industry. Both can be recycled, both *are* widely recycled. I'm wondering which is more "efficient" ?

I place that word in quotes because I can think of two methods of comparison (and perhaps you can think of more?):

Resource cost to recycle 1 ton of product into raw materials for plastic vs steel

OR

Recycling cost : original manufacturing cost ratio for plastics vs steel

What do you think? 61.189.63.224 (talk) 05:17, 18 April 2009 (UTC)[reply]

I'll try a qualitative answer here: steel is more efficient to recycle overall. I base this statement on the fact that steel is more widely recycled.
One factor in this is that it's more easy to collect and sort the steel. Crushing a car is easier than sorting six million pieces of plastic, and a bridge or a ship can pretty much be just thrown into a mini-mill. Plastics come in a wide variety of types (look at the number inside the recycling symbol on plastics, it goes up to at least nine) and are difficult to segregate since they need humans or spectroscopic/laser methods to sort. Steels by contrast can be sorted by weight and magnetic properties.
Another factor is that steel is much more dense than plastic. This comes into play when considering the cost of transporting the material back to the recycling facility, since steel "weighs out" before it "cubes out". This means that the trucks/trains/ships are carrying their maximum load, which is usually more fuel-efficient. Put another way, you probably need fewer trucks to ship the equivalent amounts of steel and plastic for their strength-weight-volume ratios.
And steel has the big advantage in thermodynamics that you just have to remelt it, you don't have to reform the molecular structure. With steel, you just make a melt, check its properties, then throw in a little more of what you need to get the exact metal alloy you're trying to produce. With plastics, you have to deal with thermal dissociation of the plastic molecules and more severe negative properties of the contaminants. Steelmaking is a metallurgical process, plastic is a chemical process.
As I said, those are qualitative impressions. If you had a million steel computer cases sitting beside a million plastic cases just outside your recycling facility, I'm not sure which one would win. I'd put my money on the steel though. The economics of recycling are very complex. But since people will come over to your house and tow your old car away for free, whereas they won't come over to get your plastic bags for free - again I'm betting on steel. Franamax (talk) 23:16, 18 April 2009 (UTC)[reply]
Recycling economics is complex; the deciding factor however in determining if it is economically feasible to recycle a material is in having uses FOR the recycled material. Take rubber tires as an example. They are a complete nuisance as a waste product (take up a lot of space, never degrade, etc. etc.) and so should be an ideal candidate for recycling them. However, they are completely worthless in that regard, as there are so few uses for recycled tire rubber. There are a few weird esoteric uses, such as paving running tracks and things like that, but really you can't even use it to make new tires out of, its a worthless material, so there is really no good reason to recycle it. The chemistry of plastics is similar. With steel cans, you melt down the steel cans and make more steel cans. Its simple and easy and means that you will always have enough demand to make steel recycling economically feasible. With plastics, it just doesn't work that way. You can't melt down plastic milk jugs and make more plastic milk jugs; the chemistry just does not support that sort of application. Sometimes, you can use the recycled plastics for alternate applications, but like with the rubber tires example, the demand created by these alternate applications is just not enough to make it as econimical as metal recycling. --Jayron32.talk.contribs 04:56, 19 April 2009 (UTC)[reply]
I thought they fed milk jugs back to the cows who made them! And I'm pretty sure we ship all our old tires to Springfield, that fire's been burning for 20 years now. :) Franamax (talk) 20:13, 19 April 2009 (UTC)[reply]

Power production from sea water by electrolysis process.

Would it be efficient to seperate hydrogen from sea-water by electrolysis process and then to produce electrocity by using the hydrogen in a gas turbine? —Preceding unsigned comment added by Shamiul (talkcontribs) 07:59, 18 April 2009 (UTC)[reply]

No. See First law of thermodynamics for why you cannot win, and Second law of thermodynamics for why you cannot break even. Splitting a water molecule requires exactly the same amount of energy that is returned when you recombine it. But none of the processes are 100% efficient - i.e. you lose energy. As a rough ballpark figure, in you configuration I would expect the loss to be around 60%. --Stephan Schulz (talk) 08:46, 18 April 2009 (UTC)[reply]
Certainly that isn't going to work in terms of getting free energy (but then there is no possible way to get free energy - period - because of the laws of thermodynamics). But there may be reasons to do it anyway. For example - it's proving very difficult to design batteries to make electric cars a particularly viable option - so using the electricity to make hydrogen and putting the hydrogen into the car makes some sort of sense. Also, if you were to supply power from wind turbines or nuclear power exclusively - then there would be times of day when you'd be producing more electricity than you need - and turning that energy into hydrogen, which you'd store and burn later when the need for electricity is higher. So there are reasons to do it - but none of them relate to getting energy for free - which is completely impossible by any means. I think Stephan's estimate of a 60% loss is an under-estimate. There aren't many gas turbines that are more than 60% efficient - and electrolysis is at best only about 80% - there are bound to be other losses. I'd be surprised if you got back 40% of what you put in. Probably, a gas turbine would be a bad way to get electricity from hydrogen - hydrogen fuel cells seem like a better bet. SteveBaker (talk) 20:02, 19 April 2009 (UTC)[reply]

Sleep

Last night, I had approximately five hours of sleep and not suprisingly, I am unable to think about mathematics today. Similarly, with regards to learning vocabulary (from another language), my short term memory did not seem to be up to its uaual standard. I also seem to be consistently thinking about sleep the whole day, unable to do much. I have thoroughly read the article on sleep but could not find the answer to this question - thinking in which areas of mathematics is affected most by a lack of sleep (and which areas are affected the least) and which aspects of thinking in general, are affected by a lack of sleep? I think this reference desk is the best place to post but hopefully I will get answers from the Wikipedians at the other reference desks. Also forgive any errors in my writing. :) --PST 10:55, 18 April 2009 (UTC)[reply]

Have you read Sleep deprivation? I wouldn't expect getting only 5 hours sleep for one night to have a significant effect, though. Plenty of people sleep that little routinely (studies vary in their conclusions about whether that is a good idea or not). I expect your problem is not that you are too tired, but rather that you are too sleepy. You need to do something to wake yourself up - I find sugar and light exercise to be the best way to do that. Once you've woken up and got going, you should be fine. --Tango (talk) 12:49, 18 April 2009 (UTC)[reply]
Thanks for your advice and comments. The other question I have is about sleeping too much. If I were to sleep in, although having slept for 10-12 hours, I feel sleepy 2 hours after having woken up. Describing it as sleepy might be inaccurate - rather it is a temporary drowsy sensation. Is it better for thinking, to sleep for around 10-12 hours or more, or a moderate amount? For me, it is usually around 8-10 hours of sleep that is the best for logical thinking. --PST 14:27, 18 April 2009 (UTC)[reply]
I know what you mean - I find sleeping too long leaves me kind of fuzzy. Again, exercise seems to help (just 10 minutes brisk walk in the fresh air is often enough). I think a typical amount of time to sleep is 6-8 hours, although opinions vary on what is best (I remember a study that showed people that sleep an average of 6 hours a night live longer than those than sleep 8, which is the opposite of conventional wisdom). --Tango (talk) 14:59, 18 April 2009 (UTC)[reply]
Having a short (20 minute) midday nap can help immensely if you aren't sleeping enough at night. Our article Sleep is full of useful advice and data. SteveBaker (talk) 19:52, 19 April 2009 (UTC)[reply]
I agree - it's amazing how much better I feel after a short nap when I haven't been sleeping properly. However, it seems from the OP's question that the lack of sleep was a one-off, so hopefully they slept properly the next night and all is well now. --Tango (talk) 21:58, 19 April 2009 (UTC)[reply]

hiv transmission

can hiv be transmitted through kissing?

and

which is more likey a way of hiv transmission, swallowing cum or bareback sex?

 —Preceding unsigned comment added by 71.198.192.120 (talk) 14:57, 18 April 2009 (UTC)[reply] 
I believe HIV can be transmitted through kissing, but it is quite unlikely. Cuts, sores, bleeding gums, etc. can increase the risk, though. Unprotected vaginal or anal sex is almost certainly a greater risk than oral sex, although cuts, sores and bleeding gums do, again, increase the risk. I remember an excellent table being linked to in a previous ref desk question showing the risks of various sexually transmitted infections being transmitted is different ways. I'll try and find it. --Tango (talk) 15:04, 18 April 2009 (UTC)[reply]
Found it. It doesn't specify its source, though, so no guarantee of accuracy. --Tango (talk) 15:07, 18 April 2009 (UTC)[reply]
The only was HIV could be transmitted through kissing if the receiver has some form of cut in their digestive tract before the stomach. If not, the virus will be broken down in the stomach. It's pretty rare to catch HIV this way.
As for your second question, the former could only result in transmission if the above provision still applies. The latter, however, is much more likely as anal sex is, quite frankly, done in an area not designed for such activities and can result in more cuts within the anal canal. If semen enters a cut, it can transmit the virus to the other person. Regards, --—Cyclonenim | Chat  16:06, 18 April 2009 (UTC)[reply]
Unprotected oral sex is NOT safe sex. There's plenty of HIV-susceptible tissue in the tonsils. The exceedingly low risk of mouth-to-mouth kissing (see page 3 of the PDF) is probably related to how unlikely it is that saliva will contain infectious HIV. --Scray (talk) 16:22, 18 April 2009 (UTC)[reply]
There have not been any documented cases of HIV transmission by French kissing. This paper discusses reasons for the low HIV content in saliva. Despite this, the CDC cautions against French kissing with HIV positive people due to theoretical risk of transmission. Axl ¤ [Talk] 16:56, 18 April 2009 (UTC)[reply]

Figuring out when a star does heliacal rising

Hello Science Deskers! (a) If I know the right acension and declination of any star, how do I do the calculation to figure out when that star has its heliacal rising? (I'm hoping there is a way to do some math and get an answer such as "19 days after the summer solstice", that kind of thing.) (b) Once I figure out the answer, how would I adjust it to figure out when that star would have had its helical rising for any year in the past (such as 5 or 10 or 15 thousand years ago)? Thank you, WikiJedits (talk) 16:00, 18 April 2009 (UTC)[reply]

What you need is a planisphere. Make sure you get one that is for the latitude where you live. You can also get free ones off the internet you can print yourself or software you can run, just try googling for it. If you want to do the trigonometry yourself, you will need a book, this one iss very old (1834) but because it is out of copyright you can view it online or print it out. I don't think trigonometry has changed much since then. SpinningSpark 17:12, 18 April 2009 (UTC)[reply]
Thank you SpinningSpark. I have found a planisphere but can't figure out how to read a heliacal rising from it. For example, I put Spica over where it says Eastern Horizon, but the earliest time it gives is 5 am and the month over there is November. I know sunrise is way after 5 am in November. Is there a way to work it out by doing a calculation instead? I have looked at the book but it is way beyond me. Is there a method a beginner could use? (Also, do you or anyone else know the answer to my question b?) Thanks again, WikiJedits (talk) 01:52, 19 April 2009 (UTC)[reply]
My Planisphere is the Philips' one like the picture in the article so if you look at that you will understand what I am talking about. You are right that the first step is to put Spica on the Eastern horizon. The moving part of the planisphere has a time scale, and the fixed part has a calendar. At any particular date, this will tell you the time Spica will rise. If you look for 19th April, you will find that Spica is rising at about 7pm in the Northern hemisphere. This depends on your latitude which is why it is important to get a planisphere for your own location. Ok, so 7pm tells you that Spica is rising in the night sky (or before) at the moment and is nowhere near its heliacal rising. Next you look around the scale till you find a date at which the timescale is showing the time of sunrise for that date (again dependant on your location). 6am is at 30th October on the planisphere, so if sunrise is at 6am on that date, then that is the also the heliacal rising of Spica.
If you cannot deal with the trigonometry in that book, then you are not going to be able to do the calcualtion by hand. I suggest you get some software to do it for you, or else go and learn the maths first.
For 5 or 10 years in the past, there is virtually no difference to the date of the heliacal rising. For thousands of years in the past, you must take into account the precession of the earth. To do this you must first locate the position of the North Celestial Pole in the epoch which you are interested in. You can get that from this diagram. Next, you must move the pivot point of your planisphere to the location of the ancient North Celestial Pole (don't destroy your good one, print a throwaway one off the internet). Then you just proceed as before.
Hope that's all you needed to know. SpinningSpark 10:07, 19 April 2009 (UTC)[reply]
Thank you, I really appreciate the time you've taken. Think I get it now - I need a table of sunrise times and to compare it to the times and dates shown on the planisphere. Got one off timeanddate.com and doing that for Spica again (at 30N) gives me a best match with October 17, 5:58 am. Additional question 1 - the heliacal rising isn't exactly at sunrise, is it? Do I need to allow extra time? Would it be better to pick July 3 when sunrise is at 4:58 am? Cos that's quite a difference.
Additional question 2 - About the past calculation, I must be doing something wrong? I drew the north celestial pole circle on the rotating part of the planisphere and put the pin in a different place along the circle. But this didn't seem to make much difference (other than the planisphere not fitting together so well). For example, I tried it for halfway around the circle, or about 10,000 years ago, and putting Spica on the eastern horizon still has the fall months aligning with the morning times, when I would expect it to be the spring months. Do you know what I'm missing? Much thanks for all your help, WikiJedits (talk) 19:42, 19 April 2009 (UTC)[reply]

Turning radius of a vehicle

The turning radii of all four or more wheels of a vehicle are always different.What does the mean turning radius given in their specification mean? —Preceding unsigned comment added by 202.70.64.16 (talk) 17:10, 18 April 2009 (UTC)[reply]

Mean turning radius is the radius through which the centre-line of the vehicle turns. It is usually defined at the front wheels, the mean radius is different along the length of the vehicle in general. SpinningSpark 17:38, 18 April 2009 (UTC)[reply]
Car manufacturers sometimes talk about the "turning circle" which is the smallest circle within which the car can do a complete 360 turn...that's a more useful measure - and refers not to the wheels but to the parts of the vehicle that stick out the most. For a car that has lots of "stuff" sticking out behind the rear wheels (like a US-style school bus), that can be a significantly larger number than the radii of the circles made by the wheels. For my car, the number in the handbook specifically excludes the door mirrors! SteveBaker (talk) 19:46, 19 April 2009 (UTC)[reply]

Sedimentary rock, evaporites: How does gypsum end forming a stratum .50m thick?

Where can I learn about the processes by which gypsum -probably originally sedimented in the form of thin little crystals- end up forming strata half a meter thick and spanning kms?

This question applies (in my curiosity) to all kind of evaporites.

In the articles about evaporites and sedimentary rocks, and in the books I've read, I don't see anything explaining the processes that undergo the evaporites once sedimented. Jorgemelis (talk) 17:17, 18 April 2009 (UTC)[reply]

See Dead Sea#Natural history. SpinningSpark 17:51, 18 April 2009 (UTC)[reply]
See also Sabkha (a supratidal salt flat), which is an environment in which thick gypsum deposits are often formed, sometimes in the form of desert roses. Evaporation at the land surface causes the rise of brines by capillary action and as these evaporate they deposit salt at the surface and gypsum and aragonite in the subsurface. In deeper water areas gypsum layers may be formed by crystallization at the air-water interface followed by sinking of the crystals to the bottom. Mikenorton (talk) 19:31, 18 April 2009 (UTC)[reply]

Lazy smokers

Which of these would improve the overall health of an inactive (no exercise) smoker with a typically bad, high-fat diet more?

  • Stop smoking
  • Regular exercise
  • Improved diet

Cheers. As before, if it's considered a medical advice issue, I have no problem with it being taken down. Vimescarrot (talk) 19:59, 18 April 2009 (UTC)[reply]

I doubt there is an answer that applies in all cases. People have lived to be over 100 while smoking 20 a day, so stopping smoking won't necessary improve anything at all (although it most likely will). Exercise and diet are complicated things, there aren't simply "good diet" and "bad diet", "exercise" and "no exercise", so more information would be required. Also, it's necessary to know in what way the person is unhealthy - if they are suffering from Emphysema, stopping smoking will almost certainly improve their health more than the other two, for example. --Tango (talk) 20:26, 18 April 2009 (UTC)[reply]
I wondered if I might get that. Since I'm just curious in general, not looking at anyone or any particular set of circumstances in particular, I probably can't get a real answer as such - though anyone is welcome to come in with more input and opinions. Vimescarrot (talk) 20:37, 18 April 2009 (UTC)[reply]
Of course, health is highly individualized. One can speak in general terms for what works in averages, but you will always find anecdotes of 110 year old men who sit on their ass all day chain smoking unfiltered Camels and eating 5 pounds of bacon a day and washing it all down with Thunderbird wine. And you can not smoke, not drink, run 10 miles a day, and live by eating nothing but tofu, broccoli, and wicker and die of a heart attack at 45. Behavioral effects on health are measurable across large populations, and you should probably do whatever you can to improve your own health, but you can do nothing at all to guarantee that you will live longer only because you do everything you are "supposed" to do. --Jayron32.talk.contribs 21:27, 18 April 2009 (UTC)[reply]
I understand this. Sadly, my mother uses this as justification to continue smoking while immobile in a chair. Vimescarrot (talk) 22:04, 18 April 2009 (UTC)[reply]
Yes - but it's a matter of probability. One person in a thousand who does all of those terrible things to their body will survive - but the other 999 will die an early death. Sure, there are anecdotes of individuals who smoked their entire lives and through sheer, blind luck survived. On the other hand, you have people like my father who tragically died in his early 50's from smoking just 10 cigarettes a day - or like my mother who suffered from most of the bllod pressure and heart disease symptoms of a 10-a-day smoker despite never having smoked in her life because of the second-hand smoke. That my father died so young is probably the only thing that saved my mother from doing the same. Picking the one person in a thousand who survived and hoping that you'll also be that lucky one person in a thousand and not one of the 999 more typical cases is exactly like saying "I'm going to give up working and assume that I'll win the lottery" - nobody in their right mind does that yet it's the exact same ridiculous thinking pattern. SteveBaker (talk) 19:29, 19 April 2009 (UTC)[reply]
It kind of depends on your exit-strategy. Do you want to live with maximal indulgence and go out with a bang? Suffer through lingering health problems? Exercise will improve your cardiac state, help your lung capacity to a certain extent and improve your feeling of general wellbeing - but there's all that dicomfort and sweating. Improved diet will lessen your risk of atherosclerosis, stroke and metabolic syndrome/diabetes - but there's the factor of being hungry all the time. Quitting smoking improves almost every risk factor there is (not diabetes) - but that could just mean you spend a long life wishing every minute of the day that you could have another smoke.
If quitting the evil weed is off the table, then a little of both of the other two. A half-hour brisk walk every day is a good start and eatcher veggies. But really, anything other than continuing to sit in a chair smoking and eating pork rinds is probably going to be an improvement. In the end though, everyone chooses their own way out the door.
Maybe you need to engage with your mother more - offer to cook some neat recipes and ask her to go out on regular walks with you. That way you get an equal benefit. Franamax (talk) 22:24, 18 April 2009 (UTC)[reply]
Exercise should be enjoyable if done correctly (endorphins and all that), and a healthy diet shouldn't leave you feeling hungry unless you are trying to lose weight (which should only by a short term issue). --Tango (talk) 01:02, 19 April 2009 (UTC)[reply]
For a heavy smoker, the answer is very likely stopping smoking. The other things make a difference, but unless the person is an inert obese blob, not as huge a difference as smoking. Looie496 (talk) 02:22, 19 April 2009 (UTC)[reply]
A mixture of OR, unsourced and read from sources but not cited:
@Tango, granted on the endorphin reward, but they take time to develop hence the initial discomfort. There's also the social factor, which is overall important. I tried working out at a gym once and didn't return. OTOH I will go for multihour walks with my friends or family anywhere and always. And despite the obvious handicap of being a smoker, I love playing squash, just because I'm competing against myself as much as the other player. Plus it's among the most exertive of sports, so you get the good stuff that much faster I suppose. :)
And the unsourced part is to feeling hunger pangs. Perhaps I should call them cravings. Long-term exposure to high-sugar and/or high-fat diets set up feedback rhythyms which can result in habitual behaviour. Our reward systems are treated independently in our wiki articles - since I'm in the "unsourced/OR" bit here, I'll point out the large number of neurons comprising the "gut brain" concerned exclusively with feeding.
@Looie - oh yes, undoubtedly quitting smoking is much more than the "likely" answer. The way the OP posed it as a simple choice, it wins hands down. I'm pretty sure the various physiologists who attend this page will agree with me on the risk factors. Trouble is that life is a little more complicated than that. You have to address the risk factors that are easiest to change (IMO) first. To use your terminology, the OP was in a way describing an "inert obese blob" they are very concerned about. They wish to modify someone else's lifestyle and outlook. There's a lot of factors there beyond just presenting "what the people on Wikipedia said". But yes of course, quit smoking! Easier said than done. :( Franamax (talk) 03:24, 19 April 2009 (UTC)[reply]

These activities complement each other nicely. When a heavy-smoking friend of mine started exercising, he found his desire for cigarettes decreasing. Conversely, if one stops smoking, then one breathes better, and might feel more like getting up and exercising! -GTBacchus(talk) 02:44, 19 April 2009 (UTC)[reply]

The classic studies are MRFIT (Multiple Risk Factor Intervention Trial) and the Framingham Heart Study, although I don't have access to those studies at the moment. This study included 84,129 women. Relative risks of coronary events:-

Relative risk of coronary events over 14 years in the Nurses' Health Study
Risk factor Increase in relative risk
Poor diet (quintile 1) vs. good diet (quintile 5) 1.90
Smoking ≥ 15 cigarettes per day vs. ex-smoker 3.5
Exercise < 1 hour per week vs. > 5.5 hours per week 1.41

However this study was a cohort study, not an intervention trial. Axl ¤ [Talk] 18:08, 19 April 2009 (UTC)[reply]

Give up smoking - by a very large margin. Our article Smoking cessation gives a TON of useful information - what techniques for giving up work best, etc. I'll quote this for you because I think it's inspirational for the person doing the quitting:
The immediate effects of smoking cessation include:
  • Within 20 minutes blood pressure returns to its normal level
  • After 8 hours oxygen levels return to normal
  • After 24 hours carbon monoxide levels in the lungs return to those of a non-smoker and the mucus begins to clear
  • After 48 hours nicotine leaves the body and tastebuds are improved
  • After 72 hours breathing becomes easier
  • After 2–12 weeks, circulation improves
  • After 5 years, the risk of heart attack falls to about half that of a smoker
  • After 10 years, the chance of lung cancer is almost the same as a non-smoker.
If the person doing the quitting can understand on a day by day basis what's getting better - they'll be more inclined to stick with it.
I don't think you'll get such immediate or profound benefits from either diet or exercise. However, it's very hard to be specific without a lot more information - and in any case, we're not allowed to give that kind of specific medical advice. Probably the very best advice we can give is "Go see a doctor" - they can weigh the pro's and con's with full knowledge of the state of the person involved. SteveBaker (talk) 19:19, 19 April 2009 (UTC)[reply]
Having quit once for 8 years, I can testify that the effects are immediate and profound. Not all of them beneficial, like searching the ashtrays in the basement to find stale old butts and light them up for two more drags, right down to the cellulose acetate in the filter. Overall, yes of course it's good to quit smoking (although I didn't have one-half a heart attack in those years, so I can't confirm your statistic ;). But the little trap in my brain circuitry was still poised, it never went away - 8 years later when a friend offered me a $200 Cohiba at a wedding (I just had to destroy that value, c'mon), the trap was sprung.
My point is that all risk factors exist, but some can be mitigated more easily than others. It's not a trinary choice, it's a matter of what can most easily be achieved in the circumstances. GTB puts it well, an overall approach yields synergy. That's not what the OP asked originally, but they later revealed their overall concern - so I suggest an overall approach. It's what I do with my own mum, she's made her own decision to keep smoking - but she smokes nothing when we go hiking. :) PS: I understand that this is not a scientific response Franamax (talk) 19:50, 19 April 2009 (UTC)[reply]
Actually, my overall concern is not with my mother. I was genuinely just curious. If I try any sort of logical approach with my mother, depending on her mood, she'll either look at me like I'm a joke, look at me like I'm stupid, or look at me like she's never seen me before. Oh - and thank you, everyone, for your epic input. Uhm...I apologise if you were only doing it because you thought it would be in some way functional. :( Vimescarrot (talk) 23:11, 19 April 2009 (UTC)[reply]

April 19

Electric field therapy

Has any one every heard of it? roscoe_x (talk) 07:49, 19 April 2009 (UTC)[reply]

Mr. Google has. 88.112.62.225 (talk) 08:01, 19 April 2009 (UTC)[reply]
O yes, I've googled it first. But many of the links are selling products. I found this one [15].What I want to know is how common it is? And its affect to health. roscoe_x (talk) 08:10, 19 April 2009 (UTC)[reply]
It depends. It will affect your health in two and only two ways. First, it will consume as much of your available income as you are willing to give the peddlers. If you otherwise spend this on Hamburgers and cigarettes, it will improve your health. If the alternative is to pay for and go to a gym, it will usually be detrimental. For the second way it may affect you, see Placebo effect. In other words, it's complete bunk, but should be fairly harmless by itself. --Stephan Schulz (talk) 09:15, 19 April 2009 (UTC)[reply]
See also Electrotherapy but it sounds like a cranky idea to me. 89.242.147.172 (talk) 10:42, 19 April 2009 (UTC)[reply]
Have a look at this Quackology site,[16]. They don't rate it very highly. Richard Avery (talk) 11:42, 19 April 2009 (UTC)[reply]
There are a bunch of different type of electric field therapy and little evidence for effectiveness for any of them. It's not scientifically implausible that something like this could work, but there's hardly any actual positive evidence at this time. Looie496 (talk) 16:52, 19 April 2009 (UTC)[reply]
Of course, it's a scam. At best, it doesn't work, at worst it does some subtle and as-yet-not-understood harm. <sigh> SteveBaker (talk) 18:27, 19 April 2009 (UTC)[reply]
(I've got to stop looking at the websites for these things...it only makes me angry). [17] claims to explain how this works. Aside from the fact that this page is one large image - so you can't cut and paste the text (grrrr) and it's riddled with typo's - the actual "science" there is just a bunch of random jargon stringed end-to-end to look impressive. In the section "How the Electric Field Therapy works", we have:
  • "EFT is a simple, non-invasive technique that uses static electricity...". OK - fact #1 - it uses static electricity.
  • "The theory is that when an electric filed with high voltage AC is applied on a human body..." - well, firstly it's a "FIELD", not a "FILED" - but fact #1 said they use STATIC electricity. You can't have AC static electricity! Plus, if you apply high voltage AC to the human body (eg by sticking your fingers into an electrical outlet, it tends to DIE - not get better!
  • "...cells metabolism is stimulated by supplementing ions and the Acid-Base Balance of Electrolyte in the blood results adjusted." ...that's not even a grammatically correct sentence. But what makes you think the pH of your blood is likely to be wrong? There is no science saying that any of the conditions it claims to fix is caused by an error in the pH of your blood - and even if it were - passing AC current at high voltages isn't going to change the pH. Do you really want to take a machine made by people who can't put together a single coherent sentence and have it try to electrocute you? I really don't think you do.
  • "The H+ flow of ions in the organism can optimally be directed."...er what? Positively charged hydrogen atoms...like in water..."can optimally be directed"...Where? Why? What happens to all of the OH- ions that would be left behind? If you try to adjust the pH by moving the H+'s in one direction than you'll be adjusting the pH wherever the OH-'s would be in the opposite direction. But this is crazy! This is doing electrolysis on your blood! If it actually did this, it would kill you amazingly quickly!
  • "This is important because beside others, the co-ordinates of enzyme, the Ca++ household, the proteins of transportation in the blood and the general acid-base-homeostasis depend on these flows of protons and ions." - OK - the only way to write a sentence with so little meaning is to open a dictionary of scientific terms and write them down at random. There is no way to even try to analyse what this sentence means!
It gets worse, the company claims to have "ISO9001" certification. I've actually been through the certification processes for those standards and they are pretty hard to get - they relate to how products are designed and documented and represent the result of frequent inspections of your processes, etc. However, when you click on the image of the "certificate" - it's not an ISO document - but something awarded by some unheard-of Chinese organization. It's certainly not a proper ISO-9001 certificate.
This is so very bad...it's beyond ridiculous. PLEASE - don't pay them a cent of your money.
SteveBaker (talk) 18:57, 19 April 2009 (UTC)[reply]
Thanks for all the information. My mom tried this. The one that made her belief there is something in this device is it made her feel electrical shock when the person touch her hand. roscoe_x (talk) 05:53, 20 April 2009 (UTC)[reply]

Viral infection

How is a viral infection detected and how is the responsible virus identified? —Preceding unsigned comment added by 173.49.18.189 (talk) 08:57, 19 April 2009 (UTC)[reply]

Do you mean by the body or in a laboratory? In the body it would be some kind of white blood cell which would then present a viral protein to another type of white blood cell which would then produce antibodies against it.
In the lab you can use lots of different ways - e.g. ELISA, polymerase chain reaction and antibody testing kits (the same technology as in pregnancy tests). I can't find a link to the last method. These all assume that you already know what the virus is. For unknown viruses it's a bit more difficult - I guess that genome sequencing would be used. Smartse (talk) 11:16, 19 April 2009 (UTC)[reply]
Thanks. The question is about how viral detection/identification can be performed in a lab context. What if the virus is unknown, or new? Would a clinical lab, say one at a hospital, be equipped to detect and confirm a new form of viral infection? Or is that only possible at a very well-equipped research lab? 173.49.18.189 (talk) 14:52, 19 April 2009 (UTC)[reply]
Hospital labs are equipped to detect known pathogens. Novel viruses would not routinely be screened for. The detection of novel viruses is a huge topic - there is no simple way to do it. Some viruses have been detected by looking for cytopathic effect in tissue culture. The hepatitis C virus was known to exist (as "non A, non B viral hepatitis") for more than a decade before Michael Houghton's group at Chiron discovered it by screening over a million cDNA clones in a phage library using serum from infected people to detect viral proteins. Today, techniques like random-primed PCR (with random-primed reverse transcription to detect RNA viruses) are available, but far from perfect. Of course once you've detected a virus, showing that it causes a specific disease is another challenge - we'll all loaded with viruses. --Scray (talk) 18:39, 19 April 2009 (UTC)[reply]

Photovoltaics and thermodynamics; ie, efficiency vs 'wasted' energy

Some considerable pondering hasn't cleared this up, and neither have I been able to find reasonable info in any of the articles I looked into, nor any of the archived reference desk questions some searches turned up.

Imagine a photo cell exposed to monochromatic photons are at the precise energy (ie, wavelength) which exactly matches the optimum energy for absorption in the cell; this is, I understand the band gap energy in traditional photo cells (eg, silicon). We will thus, be generating no phonons carrying 'excess' energy from the absorbed photons, there being an exact 'fit'. Phonon generation (energy therein will be wasted w/regard to electricity output) will be at least minimized. As nearly as I can make out, absorption and emission of photons being quantum mechanical events, there is no energy "leak" which eventually ends up in increased temperature of the photocell or attached structure.

It seems to me to be an exception to the usual energy conversion stricture which inherently includes some losses to the energy uber despot, entropy.

What have I missed, if anything? ww (talk) 10:07, 19 April 2009 (UTC)[reply]

One thing that you've missed is recombination. You might absorb every incident photon with no losses, but the resulting charge carrier pair might not make it to the external circuit. See Quantum efficiency of a solar cell. I don't know if this is the main method by which entropy gets its revenge, but it's certainly one of them. There is also the fact that "no real source of electromagnetic radiation is purely monochromatic" (according to Monochrome). --Heron (talk) 11:30, 19 April 2009 (UTC)[reply]
(ec) I am not sure that the second law of thermodynamics prevents 100% conversion of energy from one form to another. What it does do is prevent 100% conversion to useful work. This will certainly come into play once you start trying to extract current from your cell; if through nothing else the joule heating of the circuit. In any case 100% conversion of photons is not practically possible. Some of them will hit the crystal lattice and become phonons, some will be re-emitted before the electron can be made to do anything useful, all real crystals have flaws which will lose some of the photons etc etc. SpinningSpark 11:41, 19 April 2009 (UTC)[reply]
It does not violate thermodynamics because a pure plane wave of zero frequency width has zero radiation entropy, thus, there is no maximum possible conversion efficiency below 100%. The opposite extreme would be blackbody radiation at the same temperature as the cell itself, having maximum radiation entropy, none of the energy could be used to create work. Radiation obeys the laws of thermodynamics just as matter does. 129.2.43.42 (talk) 16:06, 19 April 2009 (UTC)Nightvid[reply]
My thanks all. It may be that the blinders of ignorance have loosened their hold slightly. My thought experiment was deliberately constructed to evade most practical issues and focus (ahemm..) on what seemed to be the key question. So the objection that there can't be a monochrome source is skew to the hypo; and likewise inefficiencies (ohmic and otherwise) during use of the electricity generated to something helpful (like running my automated beer fetching and opening robot, say). I was attempting to exclude such issues, mostly because they're as obiter dicta and so obscurantist. Recombination does indeed seem be something I'd not considered. Easy to miss, not being one at home in the labyrinthine complications of doped and dopey micro lattice structures, so I don't feel as terminally thick as I probably should. The anon contribution by 129.2... may be the very answer I was seeking, but alas I fell off the wagon as it made the turn into pure plane wave territory... In English this historian type can cope with, please...? ww (talk) 18:06, 19 April 2009 (UTC)[reply]

Energy Saving Lightbulbs

What is the technology in energy saving lightbulbs? Are they simply minature versions of a flourescent tube, or are they something else? Do they flicker like flourescents? An example of what I mean is here http://www.amazon.co.uk/Philips-Energy-Saving-Lightbulb-Stick/dp/B001BYUIT2/ref=sr_1_3?ie=UTF8&s=kitchen&qid=1240136836&sr=1-3 I have only been able to find articles on either incandescent or flourescent lighting. 89.242.147.172 (talk) 10:32, 19 April 2009 (UTC)[reply]

It would be great if they really were "energy saving" but I think that would contradict the second law of thermodynamics. They are compact fluorescent lamps - I think this should help. Incidentally DYK that it's best to leave them on if you leave the room for less than 15 minutes because switching them on/off wears them out and could end up making them less efficient that incandescent light bulbs. Smartse (talk) 11:04, 19 April 2009 (UTC)[reply]
They are energy saving as you get the equivalent of 60w worth of light from an incandescent bulb for only 11w. The packaging of the 11w (230-240v) bulb I have says it gives 600 lumen, and is supposed to last 10000 hours. 89.242.147.172 (talk) 11:23, 19 April 2009 (UTC)[reply]
No, they don't flicker, but they are dimmer when first switched on and achieve maximum brightness after a minute or two. Richard Avery (talk) 11:35, 19 April 2009 (UTC)[reply]
I don't believe CFLs are likely to be less efficient then incandescent bulbs if turned off in less 15 minutes. They do wear out a lot that way however and so in overall energy consumption and pollution terms as well as cost to the consumer terms, leaving them on is sometimes better then turning them off. See [18] for example Nil Einne (talk) 13:22, 19 April 2009 (UTC)[reply]
It's important to be clear on this: CFL's have a MUCH longer lifespan than incandescents when used in "normal" ways. They are vastly more efficient because they generate much less heat - so the laws of thermodynamics are not violated! They are more expensive - but pay for themselves in a very short amount of time because of the energy savings and the increased lifespan. The only significant drawback is the mercury they contain - which is a nasty pollutant. However, they use a lot less mercury than old-fashioned 'striplight' flourescents - and right now, properly recycling the darned things - or cleaning up mercury pollution afterwards are a LOT easier than dealing with global warming caused by the wasted energy from incandescent lights. SteveBaker (talk) 18:24, 19 April 2009 (UTC)[reply]
Steve, we must disagree on what "normal" use is. To me, it's switch it on when you need it, then switch the damn thing off when you don't. I put a CFL in my bathroom and it was toast within about four months. The incandescent in the fixture beside it is still working two years later. It operates maybe 30 minutes a day, but gets cycled regularly (I work at home and drink a lot of water). Do the math on that and consider that it takes 9 cubic metres of landfill soil to neutralize the mercury emitted if you throw the bulb in the trash (which I don't, but many people do). Furthermore, during heating season, six months of the year, I save NO energy by having a more efficient light. All that happens is that the other energy source comes on. If that happens to be fuel oil in a furnace vs. hydroelectricity, I'm net-subtracting from combatting global warming. (Mercury emitted from new dam reservoirs is a wash vs. CFL bulbs).
If your definition of "normal" is leaving a light on all day in an air-conditioned house, unquestionably a CFL is better. Whereas I call that aberrant behaviour! :) Franamax (talk) 20:46, 19 April 2009 (UTC)[reply]
No matter ho much water you drink, that CFL should have been ok. You cannot generalize from a single fluke. Mine are warranted for 3 years minimum, and I have so far (3.5 years since I switched) had one dud after 3 weeks - and that was replaced without any discussion. As most electronics, CFL failure will tend to follow a bathtub curve, so a few early failures are to be expected. --Stephan Schulz (talk) 21:06, 19 April 2009 (UTC)[reply]
In contrast to the CFL sitting beside my couch, which has operated reliably for long durations for the last two years? I don't think it was a dud, although it's certainly possible that when you open a two-pack of bulbs one will fall into the bathtub :) I disagree, current switching is a known factor which shortens the life of CFLs compared to incandescents. My habit, ingrained in childhood, is to switch off a light when you don't need it. I find the suggestion that I should leave the light on, consuming energy, in order to save energy - paradoxical.
You're right in a way, I should have kept my receipt, gone back to the store, complained. They would have replaced the bulb and thrown the old one into their garbage. I've researched this a lot - switching and enclosed fixtures are failure modes for CFLs, or at least were. The various commentary available covers several generations of bulb, so I'm not sure. What I do know is that I use CFLs everywhere I find them appropriate. In the bathroom, where they are switched 10-15 times a day - no, I don't see the math that tells me I should spend more for no net energy/CO2 saving and use mercury in the process. The one in the living room, definitely. The bathroom, nope! Franamax (talk) 21:33, 19 April 2009 (UTC)[reply]
Well, our article claims (with sources) that a 5 minute on/off cycle reduces CFL lifetime to about that of an incandescent lamp, or alternatively, that one on/off cycle is equivalent to about 6 minutes of lifetime. With a rated lifetime of 6000h (the lower limit) they should be good for 60000 cycles, or about 10 years - half that if you also actually leave them burning 6 minutes (which also factors into their lifetime, of course). Even if the numbers are somewhat optimistic, it looks very much as if you hit the front side of the bath tub.... --Stephan Schulz (talk) 22:20, 19 April 2009 (UTC)[reply]
Not sure how to phrase this. :) Umm - we either need the bathroom light on for much less than five minutes or much more. Myself in particular. ;) If I need to operate the lamp for more than four times what I need the actual light for just for the privilege of paying way more to get the same lifetime as an incandescent, it is costing me and the entire planet. As I say, the living-room lamps are fine, they're on for hours at a time. No way I'm going to put the bathroom light on all day and night though. I can spend less money and emit less CO2 using incandescent lamps. So that's what I'll do. But yeah, I'll put another CFL in, track the lifetime and do the math. See you in four months! :) Franamax (talk) 23:43, 19 April 2009 (UTC)[reply]
But isn't that precisely the point? The lifetime will obviously be negatively affected by the times you turn it on briefly (but the energy savings won't be by much), however since it doesn't have a continous 5 minute cycle the lifetime will almost definitely be better then a incandescent and as already mentioned, the every savings remains there and costwise they could easily work out better. Also as I already mentioned elsewhere the idea you should leave the lights on is bull, few people who actually know what they're talking about recommend, I'm someone who nearly always switches off my lights even if leaving my room for only several minutes and the lifecycle on my CFLs have been fine. Indeed those that do seem to have had short lifecycles have been ones that are usually left on for a long while! Indeed my biggest problem has been incandescent lightbulbs breaking when I turn off a light then realise I still needed it or whatever, turn it back on and 'poof' there goes the filament. Really the best thing to do is to stop worrying about 'keeping the light on' and just use the light like normal Nil Einne (talk) 01:39, 20 April 2009 (UTC)[reply]
I've yet to see any reliable data indicating that oft made heat claim has any merit and simple basic physics suggests it's probably bull so I'm going to call it that. Heat travels upwards. We all know that (or should if we're going to be making claims about using lightbulbs for heating). That's one of the reasons why we usually install heaters on the ground. If you've ever touched a incandescent you know they get very hot (so do flourescents of course but I digress) and so those the air around them (even more so if your bulb is in a fairly or completely fixture) and probably even the ceiling etc. So guess what? A (probably substanial) portion of the heat you generate with your lightbulbs is very likely not aiding you in heating your home in a useful way at all. You obviously do gain something, from the infrared and from the heat that does radiate or convect but it's very likely only small proportion of what you're generating and whatever the case is definitely not all the wasted energy even if you happen to have a fan installed on your lightbulbs to try and cycle all that heat arpimd (so your claim that you save no energy is surely wrong). It gets even worse if you're using something more efficient like a heatpump or a gas heater or whatever. The simple fact is if you want to generate heat, the best thing to do is to use something designed to generate heat (we call them heaters), not try and use something who's sole purpose is to generate light (which surprise, surprise, is what lightbulbs are for). Also do you really heat your home for 6 months of the year? P.S. If you really don't believe me that it's not simply generating the heat but actually having the heat radiate in a meaningful way, then there's this anecdotal story. I bought an oil column heater, these are common in NZ. This was a real cheapie. Several other people have similar oil column heaters of the same power (1000W). I've found the oil column heater is actually fairly useless at heating my room despite having the same power (well this is rated, I've never tested it to see if it is really what it claims) as these better brand oil column heaters. Clearly the better ones are much better at radiating heat. Obviously these would use more power overall in general since my heater is likely to get hot and switch off whereas the other heaters won't switch off as much but I still feel confident enough to say that these more expensive heater make more efficient use of the power they do use in most circumstances. P.P.S. If you have data indicating that using CFLs during heating season doesn't save energy (hopefully taking in to account any potential confounding factors) then I'm perfectly willing to eat my hat but I doubt it. Nil Einne (talk) 01:39, 20 April 2009 (UTC)[reply]

I have the feeling that they do flicker, even if I can only detect it subconciously. Although I use them they do not seem as nice as incandescent bulbs, but do feel like flourescent lighting which I do not like. I'd like it if someone could provide the evidence to prove me wrong. 89.242.147.172 (talk) 11:48, 19 April 2009 (UTC)[reply]

There are many different kinds of CFCs, with widely varying properties and prices. I use the Phillips Softtone, which after warming up looks much like an incandescent to me. It did cost me around EUR 6 or 7 per bulb, IIRC. Cheaper bulbs will quite possibly have worse performance, both in light colour and in flickering. --Stephan Schulz (talk) 12:39, 19 April 2009 (UTC)[reply]
Note it is important to consider what you mean by 'worse performance'. For example, everything else being equal, cool coloured fluorescents (cool daylight for example) generally have better colour rendition (see colour rendition index) then warm ones. (The phosphors uses makes a big difference however whatever the colour temperatures). However in Western cultures, warm coloured lights which are closer to incandescent lights are generally preferred particularly for living spaces and are sometimes considered more natural (even though cooler lights are far closer to the epitome of natural light, the sun and bright daylight). From sources I've read and anecdotal evidence these preferences don't hold for Asian cultures however where preferences are more varied and often tend to the cooler end of the spectrum. If you can only detect something 'subconsciously' it could easily be simply that you want flourescents to be bad so they are. Nil Einne (talk) 13:42, 19 April 2009 (UTC)[reply]
That's all very true - but since you can buy CFL's that match either incandescent light's color or natural daylight color - there is no way that 'color' is a good excuse for not buying them. The fact that you get the choice is a huge plus. SteveBaker (talk) 18:24, 19 April 2009 (UTC)[reply]
I agree and apologise if my response distracted from the main point. It's simply a pet peeve of mine that people commonly suggest warm lighting is better or more natural since it seems to be a cultural thing and the more 'natural' concept is IMHO flawed (as with most claims that something is more natural). An interesting thing of course is that flourescents (not necessarily CFLs) have already taken significant hold in a number of Asian countries, whether this is because of the difference in colour temperature preferences (since I believe most early flourescents had a fairly high colour temperature) or is partly the cause of the different preference of a combination of both I don't know. Nil Einne (talk) 01:08, 20 April 2009 (UTC)[reply]

I seem to get a peculiar feeling on my skin when in a room illuminated by flourescent light - as if it were something to do with static electricity perhaps. Perhaps its just my imagination. 89.242.147.172 (talk) 14:06, 19 April 2009 (UTC)[reply]

Well if you get a peculiar feeling on your skin, then that's easy to test in a double blind fashion. You leave the room and someone throws a coin, or uses a better random method to decide whether to put an incandescent or CFL in the room. The person then leaves the room. 5 minutes later (well a preset interval so there is no need for contact between the two of you), you come in to the room well blindfolded so you can't possible see what light is present. Repeat multiple times (I would say at least 20.) If you are able to detect reliably whether CFLs or incandescents are present, then we know you are somehow detecting CFLs, perhaps because of a peculiar feeling on your skin. Nil Einne (talk) 14:11, 19 April 2009 (UTC)[reply]
The trouble is that it may be the color rather than flicker or (the extremely unlikely) skin tingle that would clue the person in to which is which. CFL's exist that are a pretty good match for incandescent color - but they aren't 100% perfect. Controlling for color in your experiment is much harder. SteveBaker (talk) 18:24, 19 April 2009 (UTC)[reply]
Well 89 said that he/she believed there was a skin tingle. IMHO it would be useful for 89 to test this if it continues to influence his/her thinking. I agree it's rather difficult to control for CFLs vs incandescents if you can actually see the light whether that's because of colour or flicker or simply because you believe CFLs are bad (and you can easily tell by the colour and other factors) hence the reason I didn't suggest any double blind tests until the OP mentioned a skin tingle Nil Einne (talk) 01:08, 20 April 2009 (UTC)[reply]
One last comment here - I regard CFL's as an emergency measure. Incandescents are such a huge problem for global warming that we simply MUST get rid of them as quickly as we can. In many places around the world, laws are being passed to outlaw these ridiculously inefficient devices...and incandescent light bulb factories are gradually switching production to CFL's. They simply have to go - whether you like it or not. However, LED lighting is the true way of the future. LED lights have an amazingly long lifetime - they are yet more efficient than CFL's - you can dim them nicely (tricky with CFL's) and you can even get them where you can adjust the color they produce to whatever your heart desires (I have one of these). Because they last so long, whatever pollution they create when they eventualy DO die is relatively unimportant because we won't be throwing them away so often. The only limitation on them right now is that the really bright ones are pretty expensive. We're seeing them appear in things like car tail-lights and traffic signals where the consequences of failure and the cost of replacement are high. This year, I think there were more LED Xmas tree lights on sale than incandescents in local stores here in Texas. I have a really fancy tri-color LED "bulb" in my home-office area (you can actually turn it on and off and set the color remotely from your computer using X10 protocol over the power lines!) - it was really just a hideously expensive gimmick - but it does work. It's plenty bright enough - and you can adjust it to produce "warm" lighting or "cold" lighting depending on your mood (you can also make it flash in various colors in time with music on your PC if you're into 1970's disco lights!) But as soon as they get cheap enough (and they undoubtedly will), I plan on switching over my home to slightly saner LED lights completely. SteveBaker (talk) 18:24, 19 April 2009 (UTC)[reply]
I've noticed (at least here in NZ) halogen bulbs that are compatible with ordinary sockets are now available. I believe these are being promoted because they are considered sufficiently energy saving that they will generally be allowed in places where incandescents will be banned. (Unfortunately this isn't going to be NZ since the new government abandoned that policy although given that nearly everyone else including China and our closest neighbour Australia are banning them I expect it's just a 'feel good' reversal since they'll probably still largely disappear.) They don't have the lifespan or anywhere near the energy savings of CFLs but if you absolutely refuse to use CFLs then IMHO you should use them. They are also dimmable although if used in dimmers they may not last as long (since they don't as hot as they should) nor save so much energy although it's my understanding they will always last at least as long as ordinary incandescents and will never use more energy. You're right about LEDs of course. They're also starting to get quite common in torches and other battery powered lights where the power savings (and longevity) is important and flourescents aren't possible. Nil Einne (talk) 01:08, 20 April 2009 (UTC)[reply]

I look forward to getting something similar to sunlight rather than the yellow gloom we are used to - although on the other hand it does signal the evening and hence probably helps you prepare to sleep. 78.146.75.232 (talk) 19:54, 19 April 2009 (UTC)[reply]

You may be interested in [19]. Also do note that cool daylight CFLs (and flourescents) with colour temparatures of around 6500K (lower are available if that's too high for you) are already vastly (IMHO and also based on colour temperatures) more similar to bright daylight then the yellow gloom and are available now (and in fact were the earliest flourescents I believe) Nil Einne (talk) 01:08, 20 April 2009 (UTC)[reply]

The great flaw of CFLs is that it costs about $1 (U.S) to recycle a CFL. An incandescent bulb can be purchased for less than 50 cents (US). A CFL can be purchased for about $3 or less with occasional rebates. Most consumers will not pay $1 to recycle a CFL, so they go into landfills and the mercury pollutes the environment. This is a neglected externality of the "bulb of the future." Edison (talk) 21:23, 19 April 2009 (UTC)[reply]

You're repeating some common, misleading arguments. There are organizations in many areas that recycle CFLs free of charge, including The Home Depot. You can also get mercury-free ones if that's for some reason not an option. If you get your power from a coal plant, the reduced power requirement saves more mercury from entering the environment than is actually contained in the light. Decent CFLs last longer than incandescents and obviously take less power, so your point on price is weak. -- Consumed Crustacean (talk) 21:30, 19 April 2009 (UTC)[reply]
Yes - that's true. I'd also argue against the "bulb of the future" moniker. CFL's are the bulb of today - they've been around for many years now. LED's (or perhaps, OLED's) are the bulb of the future. But at 10x to 30x the price of a CFL and 200x the price of an incandescent - it's hard to convince people to buy them until the price drops. But LED lights are good for 50,000 hours and use even less power than CFL's - so even at $50 to $100 a pop - they'll still pay for themselves over the very long term. 50,000 hours is a lot - if you put LED lights into a new house - the odds are good that most of them will still be in the house and working fine on the day it's demolished. So the issues of what toxic effects they might have in landfills is rather academic. You can flip them on and off as much as you want - they aren't as fragile as incandescents and CFL's so they work well in places like refrigerators and ceiling fan fittings and you can have them in any color you like - or even have RGB LED lights where you can adjust the color to fit your mood on any given day. Economies of scale will push the price of LED lamps down to more acceptable levels in not too many more years...so CFL's should be regarded as a stop-gap solution. SteveBaker (talk) 03:13, 20 April 2009 (UTC)[reply]
So, how would you solve a problem like lava lamps in the future? :) Mine goes through incandescent bulbs like shit though a goose. --Kurt Shaped Box (talk) 03:20, 20 April 2009 (UTC)[reply]
I have wondered the same thing. If incandescent bulbs become impossible to find we're going to have to come up with some sort of heating element/LED combo device to retrofit all of our lava lamps. (Also EZ Bake ovens if you're into that.) APL (talk) 03:54, 20 April 2009 (UTC)[reply]

How long do American electric kettles take to boil?

I'm used to 220volts, but in the US you only have 110volts. Does that mean that American electric kettles take an extremely long time to boil water? And are electric room heaters unknown in the US? I was reading the BS 1363 article and its discussion. Edit: I've just tried boiling one litre of tap water in a 240v electric kettle, and it took 2m 45s to boil, and exactly three minutes before it turned itself off automatically. 89.242.147.172 (talk) 11:30, 19 April 2009 (UTC)[reply]

In principle, you can get the same amount of energy independent of the voltage - you just draw a higher current. I would expect US equipment to be made to the relevant specifications. In other words, a simple heating device designed for 230 volts will work in the US, but will be much less effective. A device designed for 110 volts will likely fail, possibly in spectacular and dangerous ways, if connected to 230 volts. More complex devices will run into trouble either way, although many modern electronic devices are built to accept 100-240 volts, 50-60 Hertz, to be usable off nearly any mains supply in the world. --Stephan Schulz (talk) 11:58, 19 April 2009 (UTC)[reply]
Electronics - yes, but I have never seen a universal supply kettle or anything else that takes considerable power. SpinningSpark 12:16, 19 April 2009 (UTC)[reply]
American plugs are rated at 15 amps so the maximum power that can be extracted is 1650 watts. Poking around on US eBay their kettles seem to be generally in the 1000 to 1500 watt range, although I did find one that claimed 1750 watts and can boil seven cups in 5 minutes. By contrast, British plugs are rated at 13 amp and kettles are usually in the 2000 to 3000 watt range which is considerably more. Before anyone points it out, I know that 220 volts x 13 amps is less than 3kW but the British supply is not really 220 volts. It used ot be 240 volts, when we signed up to European harmonization at 230 volts (edited) the only thing that changed was the specified allowed variation in voltage! Don't believe everything you read on labels. So in summary, American kettles will boil slower, but not extremely slower as you state. SpinningSpark 12:16, 19 April 2009 (UTC)[reply]
In NZ, the maximum current allowed by our normal plugs is 10 amps. Looking at my kettle which is the cordfree kind, it's rated at 2200-2300W. Nil Einne (talk) 13:18, 19 April 2009 (UTC)[reply]
I'd also point out that electric kettles are relatively rare in the US. When we wanted to buy one, we had quite a hard time of it. They aren't things that are ubiquitous in every household...and indeed the one we have has a low capacity and is incredibly slow compared to the ones we've owned in the UK. Most of the time, if we want boiling water, we use either the microwave or a pan on the stove - the former is faster for small quantities - the latter is faster for large quantities because the kettle is so small. There are similar issues with many other appliances. Mains-electric lawnmowers, for example, are relatively pathetic affairs compared to the ones we had in the UK - so relatively few people have them. Whether 110v or 240v was the better choice is a tough one. 110v is undoubtedly much safer around the home than 240v - but the hassle of having to have special multi-phase 220v outlets for running things like washing machines and hot-tubs is a real pain. I've recently been looking to buy an air compressor for running pneumatic tools in my garage - the 110v ones are crap but having to wire a special 220v outlet just to run the darned things is a real pain. I suspect that when electric cars and plug-in hybrids become common - the UK will use three-phase 480v charging outlets - and here in the US, we'll be stuck with 220v...which is going to be painfully slow. SteveBaker (talk) 18:01, 19 April 2009 (UTC)[reply]
That explains (partly) why they dont drink tea in the US - they cannot boil the water. They must just have coffee percolators. Makes me wonder how they "invented" instant coffee. Using an electric stove for cooking must be difficult too. 78.146.75.232 (talk) 19:47, 19 April 2009 (UTC)[reply]
I doubt that 100V is much safer than 240V (or 230V these days, +/- enough that it makes no difference). It's probably a bit safer, but is it really "much safer"? Especially since UK appliances will generally draw less current than their US equivalents (most appliances don't use anywhere near the maximum power available, so the UK and US ones will presumably use the same power) - current can be just as important as voltage when determining risk from electrocution. As for electric cars - UK charge points in public car parks, etc., might use 3-phase, but ones in private garages at people's homes won't - individual houses aren't generally connected to a 3-phase supply. It would probably be easier to increase the current than the voltage - the 13A limit is determined by the wiring inside the house, a separate high-current connection to the mains supply could be made (I don't know what the maximum current that can be drawn by an entire house is, but it is clearly much greater than 13A, although perhaps less than 13A times the number of sockets in the house). --Tango (talk) 19:05, 19 April 2009 (UTC)[reply]

First, the correct nominal North American voltage is 120, not 110 or 100. As to safety, 110-120 V is slightly better than 220-240 V in terms of shock hazard but slightly greater in terms of fire hazard since higher currents are required. However, a factor of about 2 is only going to matter if a shock or overheating incident is right on the boundary between serious and not-serious.

To match the original poster's original research, I just measured a liter of water into a Canadian 120 V electric kettle and boiled it. This kettle is old enough that it doesn't have an automatic shutoff and if it ever had a wattage rating displayed it's illegible now. But it took between 5 and 6 minutes to reach a full boil, and I think you'll find that'd be typical today. Although the circuits are rated for 15 A, nobody makes appliances that draw a full 15 A because one other thing on the circuit would trip the breaker. The tap water going in would have been between 5° and 10°C, so, taking the mid-range of those numbers, the current draw would be 92.5/(120×5.5) kilocalories/volt-minute = just about 10 A. I'd be surprised if you can get a significantly more powerful one in North America. --Anonymous, 19:40 UTC, edited 19:45, April 19, 2009.

Electric space heaters in the U.S. designed to operate on 120 volts only draw about 12.5 amps. so they only put out about 1500 watts of heat, enough for a small room. 240 volt heaters are also available, which can heat a larger room. 120 volt electric kettles in the U.S are sadly underpowered and do an inadequate job of boiling water compared to those running on 220 or 240 volts in the rest of the world. The inadequacy of 120 volts is the result of being the pioneer country in having central station electricity for heating, lighting and power. Poor insulation on the available wiring in the 1880's and a concern for safety from electrocution resulted in a distribution voltage lower than desirable in the light of 2020 hindsight. Edison (talk) 21:18, 19 April 2009 (UTC)[reply]

Here in Canada, (where many of us call electric utility poles "hydro poles") I can roughly confirm Anonymous experience with an electric kettle. I can boil 8 ounces of Canadian water and make a cup of coffee in roughly one Canadian TV commercial break (that's two minutes), so the power output is just fine for me.
The difference between a 120V vs. 240V shock is about three feet farther across the room in my experience. :) Don't try this at home kids. Luckily there was lots of space around the water heater.
And in Canada at least, getting the higher voltage is pretty straightforward. The bulk of the streel-level distribution system runs at 240V, but the step-down transformer has a centre-tap, so it's a three-wire system. In your circuit-breaker box, you tap one or other of the 120V legs for most uses. For the big stuff, you just tie two breakers together by hooking up a different coloured wire to one of them. Voila, double the voltage, no problem at all! Of course, you need to use properly rated wire when you do it. Really big stuff like industrial machinery is a whole different story. Franamax (talk) 00:27, 20 April 2009 (UTC)[reply]
Unless your appliance is the kind that's always on when it's plugged in (as my kettle is), it also needs to be double-insulated if you're going to power it that way. Most devices in both Britain and North America are designed so the power switch is on the "hot" wire and the rest of the internal wiring is connected to the "cold", but when you take 240 V from a 240/120 circuit, both sides are "hot". (This is a safety issue, not a function issue.) --Anonymous, 00:45 UTC, April 20/09.
@SteveBaker: Here in the greater Toronto area, house current (117v @ 60Hz) electric lawnmowers are the norm; gasoline-powered ones are the conspicuous exception. And practically everyone has an electric teakettle in the kitchen. That situation is opposite to my experience growing up in the States, where electric lawnmowers were scarce and electric teakettles practically unknown. Many of the teakettles are garbage, and I have serious qualms about boiling water in a plastic vessel imported from a country well known for falsifying statements of compliance with material specifications and counterfeiting safety approval and certification labels. I therefore tend to boil water in a pot on the stove, or in glass in the µwave oven. But yes, 117v electric teakettles tend to be significantly slower by subjective perception than the 220-240v items with which I've had experience outside North America. —Scheinwerfermann T·C00:54, 20 April 2009 (UTC)[reply]
There aren't many plastics that melt below 100C and I'm pretty sure that they'd have thought of that when selecting which plastic to use! Most - if not all - electric kettles in the UK are plastic 'jug-style'. After all, you don't want a material that's a good conductor of heat OR electricity when you're designing an electric kettle. The ones you find in the UK are pretty highly refined gadgets - they are fast, even when there is a lot of water in there. They have auto-shutoff and a fill level indicator. They also have cords that are curled up like a telephone handset cord so they don't dangle off the edge of the countertop where kids or pets might grab them and pull a quart of boiling water onto themselves. By comparison, the ones I've found in the US date back to the stone age. SteveBaker (talk) 02:56, 20 April 2009 (UTC)[reply]

HIV

I found out recently that in many countries, men who have sex with men are nto allowed to donate blood because they are in a higher risk group for HIV. But I don't see what the problem is; surely as long as you're tested and you're negative then you're fine to donate blood. What else is there to it? Thanks 92.7.20.157 (talk) 11:50, 19 April 2009 (UTC)[reply]

As they are high risk group, the can be infected while donating, and testing sample with conventional methods gives false negative results, while they are truly infected but in a phase called window period where HIV infection is not detected by conventional methods, so by rejecting these donations, we decrease risk of blood borne transmitted HIV Maen. K. A. (talk) 12:03, 19 April 2009 (UTC)[reply]
The problem is simply that the tests aren't 100% reliable (especially in the early stages of infection). There is a risk with any donor, but the risk with men that have had sex with men is greater and someone has determined that the former risk is acceptable and the latter isn't (probably because you can exclude men that have had sex with men without significantly reducing blood stocks, so there is very little cost and a significant benefit). --Tango (talk) 13:23, 19 April 2009 (UTC)[reply]
We have an article MSM blood donor controversy. As has been mentioned no test is 100% effective. In particular many of the older tests had large windows where a HIV would not be detected, however the new tests common in developed countries are better and have a shorter window. The bans vary from country to country but as our article notes, for a number of reasons the bans are sometimes controversal, particularly when it's a lifetime ban even more so when other high risk groups such as sex workers and intravenous drug users don't receive a similar ban. There are also questions raised about whether all MSM should be considered in the high risk group or whether some may be lower risk then people who are currently allowed to donate. [20] [21] [22] Nil Einne (talk) 14:06, 19 April 2009 (UTC)[reply]
Maen. K. A. (Madhero88), the problem lies with false negative results, not false positives. Axl ¤ [Talk] 16:31, 19 April 2009 (UTC)[reply]
Oh... So sorry my mistake, your so right Axl Thank you for correcting me :-) Maen. K. A. (talk) 16:38, 19 April 2009 (UTC)[reply]
The posters above are certainly right about higher-risk groups being excluded for medical and test-accuracy reasons. Another factor to consider is the experience of blood "suppliers" in the last few decades. The onset of HIV was not well handled (to say the least) and resulted in many infections among blood recipients and criminal charges for officials in at least Japan and France. A similar occurence happened with hepatitis C in Canada. If you were the director of a blood-donor clinic and knew you could go to jail if you did the wrong thing, would you go the safe route and bar entire classes of people, or would you say "nah, it'll probably be OK"? In Canada, last time I checked, you are asked if you spent any significant time in the UK in a certain time period and barred if you did, there is some risk that you might be carrying a TSE. You can generally still donate blood for research purposes in any case - but it will not be used in humans. Franamax (talk) 00:52, 20 April 2009 (UTC)[reply]
Ah, but it's nowhere near that simple, for this assumes male potential donors will answer honestly when asked the screening questions. There are those who feel a ban on gay male blood donors is not justified and/or not justifiable, and so will lie. This is not just a public health issue that can be resolved with a simple risk/benefit calculation; it has significant sociopolitical aspects and implications as well. —Scheinwerfermann T·C00:59, 20 April 2009 (UTC)[reply]
Clearly, there are at least these two goals: (i) make the blood supply safe, and (ii) legal protection for those collecting blood. As Franamax so aptly pointed out, the latter is a major driving force, and is served by asking the question of every donor. Lying on the part of the donors does not significantly increase the legal exposure for those who run the blood collection. I don't deny what you're saying about this being a complex web of issues, but the (somewhat simple-minded) thinking behind the screening questions is pretty clear. --Scray (talk) 02:19, 20 April 2009 (UTC)[reply]

The iceberg that sunk the Titanic

Am I likely to have any of the water molecules from this iceberg in me now? How many? In the same way that my next breath is said to contain molecules breathed by Julius Caeser, the dinosaurs, and so on. I've been wondering what volume six billion water molecules woould have, if everyone in the world has one molecule from the iceberg in them. I wantched a tv programme last night (Channel 4, UK) that said the iceberg was ten times the size of the Titanic. Incidently, some of the details given in the programme were different from those given in the Wikipedia articles. 89.242.147.172 (talk) 13:55, 19 April 2009 (UTC)[reply]

  • If one assumes that the Titanic iceberg was only 10 metres on a side (and it was certainly larger than this), it contains 103 metric tonnes of water whereas the total water in the oceans is 1.3 x 1018 tonnes. Assuming this is all now thoroughly mixed we can expect this proportion of Titanic iceberg water to be in an average (say 100kg) human. Dividing by the molecular weight of water (34 u) gets about 1012 (one trillion) molecules per person. SpinningSpark 14:59, 19 April 2009 (UTC)[reply]
So that comes out as one molecule in every 100 people, or thereabouts? Or am I misinterpreting your numbers? --Tango (talk) 15:12, 19 April 2009 (UTC)[reply]
I don't know whether or not you are misinterpreting my numbers, but you are certainly misinterpreting my interpretation of my numbers;
SpinningSpark 15:49, 19 April 2009 (UTC)[reply]
So there were some molecules, or atoms, from the iceberg in the cup of tea I've just drunk. Wow! 78.147.243.82 (talk) 16:42, 19 April 2009 (UTC)[reply]
Actually, there are even more than that, Stephan Schulz has pointed out that the molecular weight of water is 18, not 34, but was kind enough not to write my mistake on the page. SpinningSpark 17:21, 19 April 2009 (UTC)[reply]
Ok, I did misinterpret you. I thought the 1012 was the number of water molecules in a typical person, rather than the number of water molecules from the iceberg in a typical person. (Had I thought about it, I would have realised that was far too small a number...) --Tango (talk) 16:39, 19 April 2009 (UTC)[reply]
  • (ec)Well, 1 g of hydrogen has 6E23 atoms, or 6E14 (600000000000000) per person. For H2O, or water, 18g holds as many molecules. Titanic displaced about 50000 tons. Depending on what "10 times means", the iceberg should have between 500000 tons and 50000000 tons. You do the math ;-). The other way round, 6 billion (US reading, i.e. 6E9 ) water molecules is about 1E-17 mol, or 0.000000000000000018 g (give or take a few zeros if I miscounted). --Stephan Schulz (talk) 15:03, 19 April 2009 (UTC)[reply]
I'm not sure this is a meaningful question - in a body of liquid water the molecules of H2O and constantly dissociating into H+ and OH- ions and recombining (but not necessarily in the same order), so the same molecules don't exist for long. I'm not sure how long a typical water molecule lasts for, but it's probably much less than the time since the Titanic was hit - although actually, you need to measure it from when the iceberg melted, which could have been at least a few years later. --Tango (talk) 15:12, 19 April 2009 (UTC)[reply]
You are a spoilsport ;-). But what about George Washington's axe? Especially since most atoms are indistinguishable... --Stephan Schulz (talk) 15:23, 19 April 2009 (UTC)[reply]
You can just rephrase the Q as "do I have any of the hydrogen or oxygen atoms in me from the Titanic's iceberg ?". StuRat (talk) 15:52, 19 April 2009 (UTC)[reply]
Yes, you could, but that is a different question and could have a different answer. It's also a harder one to answer - I'm not sure how to determine how much oxygen or hydrogen there is in circulation - lots of it is locked up in the crust, mantle and core of the Earth, but the upper layers of the crust are involved in the mixing going on, so it becomes very difficult to work out where to draw the line. --Tango (talk) 16:39, 19 April 2009 (UTC)[reply]
Well, actually, the answer is easy, it's "yes". The difficult question to answer is "how many", that there is at least 1 is easy enough to show by taking extreme estimates. --Tango (talk) 16:52, 19 April 2009 (UTC)[reply]
The really difficult question here (aside from all of the annoying nit-picking) is whether sufficient time has elapsed for the atoms from the iceberg to have mixed sufficiently with those in the rest of the planet to make it reasonable that one or more would have made it into your body. The more recent the event, the less likely that is. So when you pick the more common choices like water that Julius Caesar peed - then the odds are much higher than for someone who was around in the 1990's. The Titanic iceberg only melted something like 100 years ago - so I doubt that it has been completely mixed into the rest of the water on the planet to the degree that Julius Caesars excretions have. But the efficiency of that mixing is undoubtedly the key here - and I'm not aware of any way to measure or even estimate the degree of that. If all the water in the world were not being swirled around by tides and such - I suppose you could use the Maxwell–Boltzmann distribution and the Random walk math to make some kind of measure of the rate at which the molecules would propagate outwards...but that would neglect other water transport mechanisms like tides and clouds being pushed around by the wind...it's a tough thing to attack convincingly. SteveBaker (talk) 17:45, 19 April 2009 (UTC)[reply]
Well, we can make a rough guess. [23] lists the self-diffusion constant of water as 2.2x10-9 m2/s. Using Fick's_law#Example_solution_in_one_dimension:_diffusion_length, with the concentration at position x=0 of 55.5 mol/L (the concentration of pure water), we find that at a range of 20,000 km (half the circumference of the earth) and at 97 years the concentration would be 55.5 mol/L * erfc(3853500). Now erfc(10) is about 2x10-45 (by [24]), and a larger parameter only makes the value less. (Note that extending the time to 2 thousand or 65 million years doesn't appreciably change things. Even at the age of the universe at 14 billion years we're still only at erfc(320).) Working in the other direction, in 97 years diffusion results in a concentration of 1 molecule/L at a distance of 38.9 meters (65 million years is 32 km). - I'd say that diffusion would play only a minor role, and other transport mechanism are much more important. -- 128.104.112.117 (talk) 23:28, 19 April 2009 (UTC)[reply]
Darn! I was hoping that the distance due to this kind of diffusion would be so big that we could neglect other sources. Oh well - so it's basically down to figuring out how much the waves, tides, winds, etc stir things up...that's really tough to figure out. SteveBaker (talk) 02:16, 20 April 2009 (UTC)[reply]
Since a rough calculation shows that the evaporation from the ocean over 100 years couldn't amount to more than 100 meters total, and the average ocean depth is several kilometers, there's a reduction factor of somewhere between 10 and 100 right there. Also, a lot depends on whether the ocean water in the region where the iceberg melted stayed on the surface or sank into the abyss, where it would have stayed for many years.
But in another sense all this is sort of irrelevant. The principles of quantum mechanics say that when water molecules are packed tightly enough so that their wave functions overlap, they do not have individual identities. There is actually no such thing as "the specific water molecules that made up the iceberg." Looie496 (talk) 02:53, 20 April 2009 (UTC)[reply]

History of Motion Detection

I want to know the History of Motion Detection Technique. somebody plz help. —Preceding unsigned comment added by 210.212.183.113 (talk) 14:48, 19 April 2009 (UTC)[reply]

I don't have an answer, but it might help other volunteers if you indicate whether the subject you have in mind is security or psychophysics. SpinningSpark 16:01, 19 April 2009 (UTC)[reply]

Cartographic projections & Symmetry

Is there a cartographic projection such that if you cut it in half along the equator, rotated one half 180 degrees, and overlaid it on the other half, all points would be overlaying those directly opposite them on the globe? I've got a kid here who wants to dig holes through the earth and would appreciate this sort of visual reference.  ;-)

Thanks! -- 208.120.228.214 (talk) 15:08, 19 April 2009 (UTC)[reply]

I am no expert on this subject but I very much doubt that there is a named projection for this. I certainly cannot find anything even remotely like that in my textbook (Steers) on map projections, nor does our article map projection. Of course, such a projection is possible, just about any surface is possible to project, it just hasn't been done. Can I suggest you download Google Earth which may help your youngster to visualise this by manipulating the globe? SpinningSpark 15:39, 19 April 2009 (UTC)[reply]
No, certainly not without some bizarre discontinuity or pathology. This is forbidden on account of parity within the surface. A pure rotation keeps left turns left turns and right turns right turns. Yet if you travel on the surface of the Earth and turn right, a person remaining opposite you will make a left turn. 129.2.43.42 (talk) 15:58, 19 April 2009 (UTC)Nightvid[reply]
Is the map in the Antipodes article what you are looking for? Rmhermen (talk) 17:03, 19 April 2009 (UTC)[reply]
Ah, just beaten to it! I don't think there's a specific projection name for a map that shows antipodes. It's not really a projection per se, but a way of overlaying other projections. --98.217.14.211 (talk) 17:07, 19 April 2009 (UTC)[reply]
After looking on the article about antipodes, I found it amusing that there is almost no overlap of land on land in those superposed maps. Dauto (talk) 22:19, 19 April 2009 (UTC)[reply]
In that case, you'd probably be amused by the Land hemisphere (unless you already clicked through to it). Incidentally, the first external link in the Antipodes article is an entertaining mindscrew to get your head around :) 80.41.73.141 (talk) 23:44, 19 April 2009 (UTC)[reply]

Decay chain

Hi, what are the equations used to calculate how much of each substance there will be at a certain point in time in a decay chain? Thanks, 96.255.93.227 (talk) 19:49, 19 April 2009 (UTC)[reply]

For radioactive ion beams. Axl ¤ [Talk] 20:48, 19 April 2009 (UTC)[reply]
More generally, I think you want the Bateman equations. Axl ¤ [Talk] 07:06, 20 April 2009 (UTC)[reply]

Why does the universe seem to be older than it is?

If I look in one direction using a powerful telescope I can see objects that are 13 billion years old. Now if I look in the opposite direction, I can also see 13 billion years old objects, which means that these objects are separated by 26 billion light years. However the age of the universe is only 13.7 billion years old, so how come the distance between these objects appears to be bigger than the universe itself? Laurent (talk) 21:54, 19 April 2009 (UTC)[reply]

If an explosion travels at lightspeed, its radius will be (lightspeed x time). Its diameter (which is what you're measuring) will be twice that. Vimescarrot (talk) 21:56, 19 April 2009 (UTC)[reply]
That doesn't answer your question. Never mind. It's something to do with relativity and the different perceptions of time and...and...and someone else will come along and correct my mistakes and explain it better, so I'd be better off not trying. Vimescarrot (talk) 21:57, 19 April 2009 (UTC)[reply]
The real reason is the metric expansion of space. Things cannot move faster than the (always constant) vacuum speed of light through space. But it's space itself that is expanding. Consider, as an example, a rubber balloon that doubles in diameter every second. Even if ants living on it are constrained to 5 cm/second, the distance between two given points on the surface will eventually increase much faster, and two ants at these points will move apart with the same speed even if they don't move with respect to the surface at all. For concrete numbers, assume the balloon has a diameter of 10 cm, and the ants are 1 cm (measured along the surface) apart. After 1 second, the balloon will have a diameter of 20 cm and the ants will be 2 cm apart. After 3 seconds, the balloon will have 80 cm, and the ants will be 8 cm apart. After 4 seconds, the balloon will have 160 cm, and the ants will be 16 cm apart - oops! The distance between the ants now has increased 8 cm in one second (and even weirder, they think they have not moved at all).--Stephan Schulz (talk) 22:34, 19 April 2009 (UTC)[reply]
As the last poster has pointed out the distance between objacts in the universe is expanding almost exponentially, so that distant galaxy that emitted some light 13.7 billion years ago that you can see now has been moving away from us all that time at ever increasing rates and is much further away now than 13.7 lightyears. (more like 40 lightyears if memory serves), and twice that distance if you compare galaxies in oposite sides of the sky. Dauto (talk) 22:42, 19 April 2009 (UTC)[reply]
I'm presuming you mean billion light years. SpinningSpark 07:12, 20 April 2009 (UTC)[reply]

April 20

Amino acid properties

are amino acids always solids? It's a question on our daughter's nursing homework and we can't find the answer. thanks. —Preceding unsigned comment added by Marksilv242 (talkcontribs) 01:31, 20 April 2009 (UTC)[reply]

Given the context, I assume you mean the "standard" one most commonly found in proteins? You can look at our Proteinogenic amino acid table and check the melting-point of each. In the more general sense, our amino acid article talks about their structure, and in particular the idea of a zwitterionic form and its physical properties. Is there a "rule of thumb" regarding being an ionic structure and having either generally high or generally low melting point? DMacks (talk) 01:52, 20 April 2009 (UTC)[reply]
Well, they decompose before they melt. But any covalent molecule is going to become a gas at sufficiently low pressure, isn't it? Looie496 (talk) 02:43, 20 April 2009 (UTC)[reply]
True 'nuf. I guess I was thinking "solid at room temp" and ignoring what actual other state or physical change occurs when they the solid falls apart:) A real question is whether they really behave as "covalent molecules" (implying neutral, not covalent ions) or as ions. For example, Glycine is fairly low molecular weight, but it listed as a decomposition (rather than melting) temperature of 233 °C: there are clearly important ionic interactions and intermolecular stabilization. DMacks (talk) 04:24, 20 April 2009 (UTC)[reply]

Are there any (real) animals that can spit/spray corrosive substances as a defence mechanism?

Question as topic. When I say 'corrosive', I mean 'literally corrosive' (as in substances of the type mentioned in the linked article), not your run-of-the-mill toxic/foul-smelling compounds. I'm pretty sure that I once read about a species of spray beetle that has two specialized glands, producing two chemicals which are harmless alone but which when mixed together and concentrated into a high-pressure jet at the point of exit, form an incredibly nasty, highly-caustic substance (though I might be wrong). Thanks. --Kurt Shaped Box (talk) 01:45, 20 April 2009 (UTC)[reply]

Bombardier beetle? --98.217.14.211 (talk) 02:06, 20 April 2009 (UTC)[reply]
That sounds about right, thanks. I'm sure that I saw a similar species on a wildlife show that could mix up and spray a pyrophoric substance too... --Kurt Shaped Box (talk) 02:15, 20 April 2009 (UTC)[reply]

Kinetic energy of gases

Hi, I am doing a physics problem, I am given values of , and T for some gas (assumed ideal) (note: it is done by web interface, values change every time) and asked to find the total translational kinetic energy of the atoms. The answer works out to be every time, and I'm not sure where that extra factor of 1/8 is coming from, could someone please help me? --130.216.30.234 (talk) 08:23, 20 April 2009 (UTC)[reply]

Note: I have looked at the article on kinetic theory, and am still not getting it. --130.216.30.234 (talk) 08:25, 20 April 2009 (UTC)[reply]
Also: related question (the one immediately following) is I am told that the balloon in the first part of the question (the one with of gas at and T) has been filled from some container containing of the same gas at and T, and asked how many of those same balloons could be filled using all the gas. The correct answer is always , or . There's that 1/8 again!! I have no idea where it's coming from! --130.216.30.234 (talk) 08:46, 20 April 2009 (UTC)[reply]

Azimuth sweep angle?

In the game Call of Duty 4, in one mission we control the guns on a C-130 Spectre Gunship. A guy on the plane says a couple of times: "Recalibrate azimuth sweep angle, adjust elevation scan." What does it mean? And yes I'm the guy who asked the Coriolis effect and sniping question . Just a big fan of COD4. And thanks for your answers. —Preceding unsigned comment added by 116.71.54.217 (talk) 08:52, 20 April 2009 (UTC)[reply]